Anda di halaman 1dari 182

Cmo construir preguntas de Seleccin Mltiple para Ciencias Bsicas y Ciencias Clnicas

Versin en espaol de la

Tercera Edicin
(Revisada) Autores de la versin original en ingls:

Susan M. Case David B. Swanson

Traduccin del ingls


Director del proyecto: Alberto Galofr, MD, MEd (1, 2, 3) Colaboradores principales: Ana C. Wright (1); George E. Swaneck, MD; Daniel Moraga, PhD (2) Colaboradores: Ximena Trivio, MD (1); Lus Nez, MD (8); Pedro Herskovic, MD (4); Flavia Garbin MD (7); Vernica Hering MD (6); Marisol Sirhan, MD (1); Pilar Ibez (5). Pontificia Universidad Catlica de Chile (Santiago, Chile); 2Universidad Catlica del Norte (Coquimbo, Chile); 3Saint Louis University (Missouri, USA); 4Universidad de Chile (Santiago, Chile); 5Universidad de Concepcin (Concepcin, Chile); 6 Universidad Austral de Chile (Valdivia, Chile); 7Universidad de Los Andes (Santiago, Chile); 8Universidad Catlica de la Santsima Concepcin (Concepcin, Chile).
1

National Board of Medical Examiners 3750 Market Street Philadelphia, PA 19104 2005

Esta traduccin fue producida por un grupo de profesionales de la educacin mdica de Chile. No es una traduccin oficial de la versin en ingls, pero en todo momento se ha intentado conservar fielmente el texto original. En algunos casos en que se detectaron erratas en el original ingls, stas fueron corregidas. El National Board of Medical Examiners (NBME) tiene planificado realizar su propia traduccin en un futuro prximo. Ofrecemos la presente versin mientras sale a luz la versin del NBME. La versin en ingls contiene la siguiente nota que se traduce aqu: El National Board of Medical Examiners (NBME) no dispone de copias impresas de esta publicacin (Nota: la versin en ingls). Se puede obtener copias adicionales bajando este manual desde el sitio web del NBME (www.nbme.org/about/publications.asp) . Se confiere permiso para duplicar este manual siempre que (1) el copyright y el permiso para publicar el manual aparezca en cada copia, (2) se use este manual solamente para usos cientficos y no comerciales y (3) no se modifique el documente. El NBME se reserva cualquier otro derecho no explcitamente mencionado aqu.

Copyright 1996, 1998 National Board of Medical Examiners (NBME) Copyright 2001, 2002 National Board of Medical Examiners (NBME). All right reserved. Printed in the United States of America

Cmo construir preguntas de Seleccin Mltiple para Ciencias Bsicas y Ciencias Clnicas
Versin en espaol de la

Tercera Edicin
(Revisada) Autores de la versin original en ingls:

Susan M. Case David B. Swanson

Traduccin del ingls


Director del proyecto: Alberto Galofr, MD, MEd (1, 2, 3) Colaboradores principales: Ana C. Wright (1); George E. Swaneck, MD; Daniel Moraga, PhD (2) Colaboradores: Ximena Trivio, MD (1); Lus Nez, MD (8); Pedro Herskovic, MD (4); Flavia Garbin MD (7); Vernica Hering MD (6); Marisol Sirhan, MD (1); Pilar Ibez (5). Pontificia Universidad Catlica de Chile (Santiago, Chile); 2Universidad Catlica del Norte (Coquimbo, Chile); 3Saint Louis University (Missouri, USA); 4Universidad de Chile (Santiago, Chile); 5Universidad de Concepcin (Concepcin, Chile); 6 Universidad Austral de Chile (Valdivia, Chile); 7Universidad de Los Andes (Santiago, Chile); 8Universidad Catlica de la Santsima Concepcin (Concepcin, Chile).
1

National Board of Medical Examiners 3750 Market Street Philadelphia, PA 19104 2005

Comentarios acerca de la traduccin del ingls


Traducir un manual como ste presenta desafos. Por un lado, existen trminos en el idioma ingls de Norteamrica que son difciles de traducir con un equivalente en idioma espaol. Por ejemplo, conceptos como red herring, window dressing no encuentran su equivalente en un vocablo en espaol, por tanto es necesario usar una descripcin aproximada del concepto al traducir este tipo de palabras. Ms complejo es el problema que se presenta al traducir elementos tcnicos relacionados a la evaluacin, que no tienen an un significado universalmente aceptado en espaol. As, Extended-matching items o Pick N testwiseness, non-testwiseness student, high-stakes exam requerirn seleccionar entre varias opciones de traduccin, que no necesariamente dejarn satisfechos a todos los lectores. No se escapa a esta situacin algo tan simple como el plural de tem. Segn la fuente consultada y el uso, puede aceptarse tems, temes o incluso tem como opciones de la forma plural. Otra dificultad es que, al revs del ingls donde ya esta estandarizado el uso de vocablos como por ejemplo stem, multiple-choice item, matching item, en idioma espaol existen tres o ms opciones para cada una de stos. Adems, en muchas ocasiones, el traductor se encuentra ante palabras que pueden aceptar dos o ms acepciones. Es reconocido que en algunos pases de habla hispnica se usen ms comnmente unas acepciones y en otros, unas diferentes. Para aquellos casos en que el trmino traducido, ya sea tcnico o no, podra dejar dudas en el lector, se opt por agregar el original en ingls al pie de pgina, indicado con la numeracin correspondiente despus de la palabra o el concepto en cuestin. As se ver frecuentemente la presencia de la palabra original entre parntesis cuadrados, de esta forma: [ENG: red herring] al pie de pgina. Quiero agradecer a los entusiastas colaboradores que aportaron cientos de sugerencias para ir puliendo sucesivas versiones del texto. En una primera edicin es difcil estar seguro que no se haya escapado algn error que necesite modificarse. Para ello acompao mi correo electrnico para recibir sus comentarios. Alberto Galofr MD, MEd Santiago de Chile, julio de 2005 agalofre@uc.cl

ndice
Seccin I Asuntos relacionados al formato y estructura de preguntas de los exmenes ..............................................................7
Captulo 1. Introduccin .......................................................................................................................................9 La evaluacin: un componente importante de la enseanza..........................................................................9 Asuntos relacionados al muestreo ...............................................................................................................10 Importancia de las consideraciones psicomtricas ......................................................................................11 Captulo 2. Los formatos de temes de Seleccin Mltiple .................................................................................13 Preguntas de Verdadero/Falso versus preguntas de una mejor respuesta ...................................................13 La familia de preguntas de Verdadero/Falso ...............................................................................................14 La familia de preguntas de una mejor respuesta..........................................................................................16 Balance final en cuanto al formato de las preguntas ...................................................................................18 Captulo 3. Defectos tcnicos en los temes........................................................................................................19 Asuntos relacionados a la astucia de los examinandos para responder exmenes...................................19 Asuntos relacionados a dificultades irrelevantes.........................................................................................22 Resumen de los defectos tcnicos de las preguntas.....................................................................................26 Uso de trminos imprecisos en preguntas de exmenes..............................................................................27

Seccin II Como redactar preguntas de una mejor respuesta para las Ciencias Bsicas y Ciencias Clnicas .................................31
Reglas bsicas para los temes de una mejor respuesta ...............................................................................33 Captulo 4. Contenido de los temes: la evaluacin de la aplicacin del conocimiento de ciencias bsicas ....35 Contenido de los temes de ciencias bsicas ...............................................................................................35 Plantillas de temes ......................................................................................................................................38 Plantillas adicionales ...................................................................................................................................39 Tipos de preguntas y ejemplos de preguntas introductorias y listas de opciones........................................40 Redaccin de las opciones: cmo modificar la dificultad de un tem .........................................................41 La forma de un tem.....................................................................................................................................42 Aprendizaje basado en problemas (ABP) y el uso de un grupo de preguntas en base a un caso ................43 Ejemplos de temes para las Ciencias Bsicas.............................................................................................47 Captulo 5. Contenido de las preguntas: evaluacin de la aplicacin del conocimiento de Ciencias Clnicas 51 Mtodos de evaluacin ................................................................................................................................51 Asuntos generales acerca de qu evaluar ....................................................................................................52 Examinar el recuerdo de datos aislados o examinar aplicacin del conocimiento......................................53

Redaccin de preguntas de una mejor respuesta .........................................................................................56 Detalles acerca de los enunciados de las preguntas.....................................................................................57 La verborrea, la informacin innecesaria y las pistas falsas: Mejoran las preguntas de un examen? .......58 Redactar temes relacionados a las tareas que realiza un mdico................................................................61 Redactar temes de tpicos difciles ............................................................................................................66

Seccin III temes de emparejamiento ampliado .......................................................................................................................69


Captulo 6. temes de emparejamiento ampliado (Tipo R).................................................................................71 Como evitar errores al redactar temes de emparejamiento ampliado para sus exmenes..........................72 Ejemplos de declaraciones introductorias y de tpicos para la lista de opciones........................................74 Aspectos adicionales sobre las opciones en los Conjuntos R ..................................................................75 Redaccin de enunciados.............................................................................................................................76 Ejemplos de buenos y malos enunciados usando la misma lista de opciones .............................................77 Resumen de los pasos a seguir al confeccionar temes de emparejamiento ampliado ................................81 Ejemplo de conjuntos de emparejamiento ampliado ...................................................................................82 Pasos para organizar un grupo para redactar preguntas de emparejamiento ampliado ...............................90 Formulario para redactar preguntas de emparejamiento ampliado..............................................................93 Un ejemplo de cdigo para SPSSX para corregir exmenes de seleccin mltiple, incluyendo los temes de emparejamiento ampliado.......................................................................................................94 Comparacin entre temes de formato de cinco opciones y de emparejamiento ampliado.........................96 Desde los A hasta las R y de vuelta.............................................................................................................97 Captulo 7. temes de Seleccione N: una extensin del formato de emparejamiento ampliado ........................99

Seccin IV Temas adicionales ...............................................................................................................................................105


Captulo 8. Interpretacin de los resultados del anlisis de temes .................................................................107 Captulo 9. Cmo establecer un estndar para aprobar o reprobar ...............................................................111 Definiciones y principios bsicos ..............................................................................................................111 Dos mtodos para establecer estndares basados en juicios acerca de los temes.....................................112 Estndares Relativos / Absolutos de acuerdo: el mtodo de Hofstee........................................................114 Captulo 10. Comentarios diversos acerca de tpicos de evaluacin ..............................................................115 Apndice A. El cementerio de los formatos de temes del NBME ....................................................................117 Apndice B. Ejemplos de plantillas para escribir temes, temes, preguntas introductorias y listas de opciones para ciencias bsicas y clnicas ........................................................................................................129

Prefacio a la Tercera Edicin (Ingls)


Este manual fue escrito para que los profesores mejoren la calidad de las preguntas de seleccin mltiple que redactan para sus exmenes. Este manual provee una visin general de los formatos de temes, concentrndose en el formato tradicional de una mejor respuesta y los temes de emparejamiento 1 . Examina los defectos tcnicos de las preguntas, asimismo como aspectos relacionados al contenido de los temes. Tambin provee informacin bsica para ayudar a los profesores con ndices estadsticos obtenidos despus de administrar un examen. Estos se relacionan con la calidad de las preguntas. Tambin se provee una visin general de tcnicas usadas para decidir los estndares que determinan si un alumno aprueba o reprueba un examen. Al mismo tiempo no se hace mencin detallada de la planificacin de un examen. Este manual se concentra casi exclusivamente en aspectos relacionados a las preguntas y dejamos para otra ocasin la planificacin de un examen. Creemos que este manual va a ser de utilidad en especial para profesores que ensean a estudiantes de medicina tanto en ciencias bsicas como en ciencias clnicas. Los ejemplos estn tomados de la educacin mdica de pregrado, aunque la manera de abordar la construccin de preguntas puede servir para evaluar examinandos de otros niveles. Este manual refleja lo aprendido por nosotros al participar en el desarrollo de preguntas y exmenes en los ltimos 20 aos. Durante este perodo nos ha tocado revisar literalmente decenas de miles de preguntas de seleccin mltiple. Adems hemos dirigido talleres de construccin de preguntas en los que han participado miles de profesores que redactaban preguntas para los exmenes nacionales tales como USMLE 2 , NBME 3 , y de especialidad; adems de profesores de ms de 60 escuelas de medicina para la preparacin de exmenes en sus instituciones. Cada participante en los talleres nos ha ayudado a dar forma a nuestras ideas respecto a cmo redactar preguntas de mejor calidad y, a travs de los aos, hemos llegado a ser ms capaces (creemos) para articular las razones de nuestras recomendaciones. Esperamos que este manual logre comunicar estas ideas.

Susan M. Case, PhD David B. Swanson, PhD Enero 1998.

1 2

Tambin conocidos como de trminos pareados, por pares, de correspondencia. USMLE = Unites States Medical Licensing Examination. Es el examen nacional en los Estados Unidos de Norteamrica para autorizar el ejercicio de la profesin de medicina. Consta de tres secciones separadas en el tiempo. 3 NBME = National Board of Medical Examiners. Es tanto la entidad que genera y administra el USMLE como el nombre del examen nacional que antecedi al USMLE.

Seccin I Asuntos relacionados al formato y estructura de preguntas de los exmenes

Esta seccin examina importantes asuntos estructurales para construir preguntas de alta calidad para los exmenes. La seccin siguiente analizar asuntos relacionados al contenido de los temes.

Captulo 1 Introduccin
La evaluacin 1: un componente importante de la enseanza
La evaluacin es un componente crtico de la enseanza; bien usada puede ayudar a lograr objetivos curriculares importantes. No se puede sobrestimar el impacto que tiene el tomar decisiones acerca de cmo y cundo evaluar el conocimiento adquirido y el desempeo 2 de sus estudiantes. Un objetivo importante de las pruebas o exmenes 3 es comunicar qu es lo que se considera importante. La evaluacin asimismo ayuda a llenar brechas educacionales al estimular en los estudiantes la lectura personal. Esta consecuencia 4 de las pruebas es particularmente importante en las rotaciones clnicas 5 , donde el currculo puede variar entre los estudiantes al depender de factores tales como el entorno clnico y la disponibilidad variable de pacientes. Esta consecuencia tambin puede ser importante en algunas metodologas de aprendizaje de ciencias bsicas (por ejemplo, en el aprendizaje basado en problemas) en donde las experiencias educacionales pueden variar entre los estudiantes. Debido a que las pruebas tienen una influencia tan poderosa en el aprendizaje de los estudiantes, es importante elaborar pruebas que promuevan los objetivos educacionales que usted desea alcanzar. Introducir un examen en que los estudiantes tengan que demostrar un conocimiento prctico de habilidades clnicas movilizar a los estudiantes desde la biblioteca hacia la clnica, donde podrn pedir ayuda para practicar sus destrezas en el examen fsico de pacientes. En cambio, el introducir pruebas que examinen predominantemente el recordar hechos aislados dirige a los estudiantes a atosigarse de materia a ltimo momento usando textos de reseas de cursos. Este manual est orientado a aprender a desarrollar preguntas de seleccin mltiple de alta calidad destinadas a evaluar habilidades para interpretar informacin6 y tomar decisiones, caractersticas que creemos son importantes componentes de las habilidades clnicas. El camino que siguen los estudiantes para adquirir sus habilidades ser menos escabroso si reciben una retroalimentacin 7 continua acerca de su progreso.

1 2

[ENG: Assessment] [ENG: performance] 3 [ENG: testing] 4 [ENG: outcome] 5 [ENG: clerkships] 6 [ENG: facts] 7 [ENG: feedback]

Captulo 1. Introduccin

Propsitos de las pruebas


Comunicar a los estudiantes qu materia es importante Motivar a los estudiantes a estudiar Identificar reas de deficiencias en el estudiante que requieren de ms estudio. Determinar la nota final de un curso o tomar decisiones de promocin Identificar aquellas reas con deficiencias en el curso o currculo

Qu deben evaluar las pruebas?


El contenido de los exmenes debe concordar con los objetivos del programa de curso, tanto preclnico como clnico Los contenidos ms importantes deben tener ms relevancia que los contenidos de menor importancia El tiempo dedicado a cada contenido en la prueba debe reflejar la relativa importancia del tpico La muestra de preguntas debe ser representativa de las metas de enseanza.

Asuntos relacionados al muestreo 1


El propsito de toda evaluacin es permitir que se obtengan inferencias acerca de las habilidades de los examinandos 2 ; inferencias que van ms all de los problemas especficos (tanto de los casos como de las preguntas de una prueba) incluidos en el examen, hasta abarcar el dominio ms amplio 3 desde el cual se tom la muestra de los casos (o de las preguntas). Est claro que evaluar toma tiempo. Tambin queda claro que si usted incrementa el tiempo que dedica a una actividad, va a tener que disminuir el tiempo que invierte en otra. Ya sea que usted est desarrollando un plan completo de evaluacin o que est decidiendo qu va a incluir en una sola prueba, usted se enfrenta bsicamente con un problema de muestreo. El desempeo exhibido con la muestra sirve de base para estimar el desempeo en el dominio ms amplio que es el que realmente nos interesa. Cuando se usan preguntas de seleccin mltiple debe decidirse primero qu incluir en una prueba. El grado de atencin dedicado a evaluar algn tema debiera reflejar su importancia relativa. Es necesario elegir una muestra de tpicos y de habilidades (por ejemplo, determinar el diagnstico, decidir un prximo paso en el manejo de un paciente). No se puede preguntar todo. La naturaleza de la muestra determina hasta qu punto la estimacin de la habilidad real es reproducible (confiable, generalizable) y exacta (vlida). Si la muestra no representa adecuadamente el dominio ms amplio que nos interesa (por ejemplo, si se incluye solamente contenido cardiovascular en una prueba de medicina general), los resultados de la prueba sern sesgados y no permitirn estimar el desempeo en el dominio que nos interesa. Si la muestra es muy pequea, los resultados del examen podrn no tener la confiabilidad necesaria (reproducibles, confiables) para asegurar que reflejen la habilidad real del examinando.

1 2

[ENG: Issues related to sampling] [ENG: examinees] 3 [ENG: larger domain]

10

En una prueba de seleccin mltiple existe casi siempre un solo corrector de la prueba 1 (habitualmente un computador) y una serie de preguntas o conjuntos de preguntas. El muestreo consiste en seleccionar un subconjunto de preguntas para incorporar en la prueba. En otros mtodos de evaluacin (ejemplo: exmenes orales basados en casos de pacientes, exmenes con pacientes estandarizados, pruebas de ensayo), este muestreo es mucho ms complicado. Cualquier mtodo de evaluacin que no pueda determinar el puntaje mediante el uso de una mquina, requiere de un muestreo en una segunda dimensin: la del corrector de la prueba. En estos exmenes se est interesado en el desempeo del examinando en un rango de casos y se desea que la calificacin que obtiene el examinando sea independiente del que califica. En estos casos se necesita hacer un muestreo en dos dimensiones: una, en las preguntas o casos a incorporar en el examen y otra, para los que actuarn de jueces o calificadores 2 de los resultados del examen. Es necesario hacer un muestreo en un rango de casos porque el desempeo en un caso puede no predecir adecuadamente el desempeo en otros casos. Es necesario tambin hacer un muestreo usando varios calificadores para reducir el efecto que puedan tener calificadores, ya sea muy estrictos, o su contraparte, demasiado indulgentes. Tambin de esta manera se reduce el efecto de aureola 3 que produce problemas en la consistencia de la calificacin de los resultados de los exmenes cuando varias personas estn calificando. Con un muestreo amplio, se tiende a reducir las variaciones en el desempeo del estudiante y las variaciones entre los calificadores.

Importancia de las consideraciones psicomtricas


El grado de importancia de los aspectos psicomtricos en un mtodo de evaluacin est dado por los propsitos de la prueba y el tipo de decisiones que se tomarn basados en sus resultados. Para pruebas de alta trascendencia 4 (por ejemplo aquellas usadas para decisiones de graduacin, promocin, e incluso para determinar la nota final de un curso), los resultados de una prueba deben ser razonablemente reproducibles (confiables) y exactos (vlidos). Para pruebas de menor trascendencia 5 , las caractersticas psicomtricas son menos importantes y la preocupacin principal debe ser el orientar el aprendizaje del estudiante. Tal como se hizo notar anteriormente, para generar un resultado reproducible es necesario un muestreo amplio (tpicamente una docena de casos, 100 o ms preguntas de seleccin mltiple o de respuesta breve). Las siguientes publicaciones incluyen un mayor detalle acerca de temas de evaluacin en general.
Swanson DB. A measurement framework for performance based tests. In: Hart I, Harden R, eds. Further Development in Assessing Clinical Competence. Montreal: Can-Heal Publications; 1987:13-45. Case SM. Assessments of truths that we hold as self-evident and their implications. In: Scherpbier AJJA, van der Vleuten CPM, Rethans JJ, Van der Steeg AFW, eds. Advances in Medical Education. Dordrecht, The Netherlands: Kluwer Academic Publishers, 1997:2-6.

1 2

[ENG. grader] [ENG. raters] 3 [ENG: halo] 4 [ENG: high-stakes tests] 5 [ENG: low-stakes tests]

Captulo 1. Introduccin

11

Friedman C, de Bliek R, Greer D, Mennin S, Norman G, Sheps C, Swanson DB, Woodward C. (1990) Charting the winds of change: recommendations for evaluation innovative medical curricula. Academic Medicine, 65, 8-14. Swanson DB, Case SM, van der Vleuten CP. (1991:1997/2nd edition) Strategies for student assessment. In Boud, D. and Feletti, G (Eds). The Challenge of Problem-Based Learning. London, Kogan Page Ltd., 269-282.. Newble DI, Dauphinee D, Woolliscroft JO, MacDonald, Mulholland H, Page G, Swanson DB, Thomson, A, van der Vleuten CP. (1994) Guidelines for assessing clinical competence. Teaching and Learning in Medicine, 6:3, 213-220. Swanson, DB, Norman, G, Linn R. (1995) Performance-based assessment: Lessons from the health professions. Educational Researcher, 24:5, pp 5-11,35. Abridged version reprinted in Pedagogue: Perspectives in Health Sciences Education, 6 (Summer 1996), McMaster University Program for Educational Development, 1-7. Swanson DB, Case SM (1997) Assesssment in basic science instruction: Directions for practice and research. Advances in Health Sciences Education: Theory and Practice, 2: 71-84. Case SM, (1997) Assessment of truths we hold as self-evident and their implications. In Scherpbier AJJA, van der Vleuten CPM, Rethans JJ, van der Steeg AFW (Eds), Advances in Medical Education. Dordrecht, The Netherlands: Kluwer Academic Publishers, 2-6. Swanson, DB, Clauser BE, Case SM. (1999) Clinical skills assessment with standardized patients in high-stakes tests. A framework for thinking about score precision, equating, and security. Advances in Health Sciences Education, 4:67-106.

Los siguientes trabajos describen los formatos de temes en mayor detalle.


Case SM, Downing SM. (1989) Performance of various multiple-choice item types on medical specialty examinations: types A, B, C, K, and X. Proceedings of the 28th Annual Conference on Research in Medical Education, 167-172. Swanson DB, Case SM. (1992) Trends in written assessment: a strangely biased perspective. In Harden R, Hart I, Mulholland H (Eds.), Approaches to the Assessment of Clinical Competence: Part 1. Page Brothers, Norwich, England, 38-53. Case SM, Swanson DB. (1993) Extended-matching items: a practical alternative to free-response questions. Teaching and Learning in Medicine, 5(2), 107-115. Swanson DB, Case SM. (1995) Variation in item difficulty and discrimination by item format on Part I (basic sciences) and Part II (clinical sciences) of U.S. licensing examinations. In Rothman A, Cohen R (Eds.), Proceedings of the Sixth Ottawa Conference on Medical Education. University of Toronto Bookstore Custom Publishing, 285-287. Norman G, Swanson DB, Case SM. (1996) Conceptual and methodological issues in studies comparing assessment formats. Teaching and Learning in Medicine, 8(4):208-216.

12

Captulo 2 Los formatos de temes de Seleccin Mltiple1


Para que una pregunta de una prueba sea una buena pregunta, sta debe satisfacer dos criterios bsicos. Primero, la pregunta debe referirse a un contenido importante. Esta es una condicin esencial, la que ser abordada ms adelante en este manual. Obviamente, el contenido del tem es de fundamental importancia pero no es suficiente para garantizar que su pregunta sea considerada una buena pregunta. Los temes que pretenden evaluar rigurosamente materias importantes no pueden hacerlo bien a no ser que estn bien construidos. Es importante evitar defectos en su construccin que beneficien a un examinando astuto en detectar y explotar estas fallas. Igualmente, es necesario evitar introducir dificultades ajenas a la materia que es evaluada. Slo as se podr obtener que las preguntas de una prueba generen puntajes vlidos.

Preguntas de Verdadero/Falso versus preguntas de Una Mejor Respuesta 2


El universo de las preguntas de seleccin mltiple 3 puede dividirse en dos familias de temes: aqullas que requieren que el examinando indique todas las respuestas que son apropiadas (Verdadero/Falso) y aqullas que requieren que el examinando indique solamente una respuesta correcta (una mejor respuesta). Cada familia est representada por varios formatos especficos que se enumeran a continuacin: Formato de temes de Verdadero/Falso que requieren que el examinando seleccione todas las opciones correctas. C (A / B / Ambas / Ninguna) K (Preguntas de Verdadero/Falso complejas) X (Preguntas de Verdadero/Falso simples) Simulaciones del tipo Problemas de Manejo de Pacientes 4 Formato de temes de una mejor respuesta que requieren que el examinando seleccione la nica mejor respuesta. A (4 o ms opciones, tem nico o en grupos) B (4 o 5 opciones en temes de emparejamiento 5 , en grupos 6 de 2 a 5 temes) R (temes de emparejamiento ampliado 7 en grupos de 2 a 20 temes) Las letras usadas para rotular los formatos de temes no tienen un significado intrnseco. Se han asignado ms o menos en forma consecutiva a los nuevos formatos de temes a medida que stos iban siendo creados. (Ver Apndice A).

Otras acepciones: opcin mltiple, mltiple eleccin. Se llaman preguntas de una mejor respuesta porque el tem puede contener ms de una respuesta correcta, pero se trata de responder cul opcin es la mejor. 3 [ENG: multiple-choice questions o MCQ] 4 [ENG: Patient Management Problems o PMPs] 5 [ENG: matching items]. Otras acepciones: trminos pareados, por pares. 6 [ENG: sets] 7 [ENG: Extended-Matching items]. Ver el captulo 6.
2

Captulo 2. Los formatos de temes de Seleccin Mltiple

13

La familia de preguntas de Verdadero/Falso


Las familias de temes de Verdadero/Falso y de una mejor respuesta plantean tareas muy diferentes por parte del examinando. Las preguntas de Verdadero/Falso requieren del examinando seleccionar todas las opciones que son verdaderas. Para estas preguntas el examinando debe decidir donde hacer el corte para decidir que una respuesta es correcta y puede ser rotulada como verdadera. Mientras esta tarea requiere de un juicio adicional (ms all del que es requerido para seleccionar una mejor respuesta), este juicio adicional puede no estar relacionado con la posesin de habilidades clnicas o de un mayor conocimiento. Muy a menudo los examinandos deben adivinar lo que el autor del tem tena en mente al redactar la pregunta, porque las opciones no son completamente falsas ni completamente verdaderas. El siguiente es un ejemplo de una buena pregunta de Verdadero/Falso desde una perspectiva de su construccin. (Siguiendo la tradicin, para las preguntas de Verdadero/Falso las opciones se sealan con nmeros; para las preguntas de una mejor respuesta las opciones se sealan con letras). Ntese que el enunciado es claro y las opciones son absolutamente verdaderas o falsas, sin ambigedades. Cul de la(s) siguiente(s) condicin(es) recesiva(s) est(n) ligada(s) al cromosoma X? 1. Hemofilia A (Hemofilia clsica) 2. Fibrosis qustica 3. Distrofia muscular de Duchenne 4. Enfermedad de Tay-Sachs Las opciones pueden ser diagramadas como sigue: 2 4 Opciones totalmente erradas 1 3 Opciones totalmente correctas

Afirmaciones correctas con respecto a la fibrosis qustica incluyen: 1. La incidencia de fibrosis qustica es de 1:2000. 2. Los nios que padecen de fibrosis qustica generalmente fallecen en la adolescencia. 3. Los pacientes de sexo masculino con fibrosis qustica son estriles. 4. La fibrosis qustica es una enfermedad autosmica recesiva.

La siguiente pregunta de Verdadero/Falso est mal construida. Las opciones 1, 2 y 3 no pueden ser juzgadas como absolutamente verdaderas o absolutamente falsas. Un grupo de expertos no estuvo de acuerdo en las respuestas. Pensando en la opcin 1, note que la incidencia verdadera no es exactamente 1:2000. En esta situacin los expertos requeriran informacin adicional, por ejemplo: A qu pas se refiere esa cifra? La cifra se refiere a todos los grupos tnicos? Se podra intentar modificar la redaccin a aproximadamente 1:2000, pero esta modificacin tampoco ayuda, puesto que no se especifica el lmite superior y el inferior de la aproximacin. El mismo problema surge con las opciones 2 y 3, mientras que la opcin 4 es clara.

14

Aunque redactado en broma (por el segundo autor) el siguiente tem de Verdadero/Falso ilustra un El camino hacia el corazn del hombre es a travs problema comn: preguntas en las que el enunciado de su/sus: no es claro. Dependiendo de la perspectiva del que 1. aorta responde, las opciones 1, 2 y 3 pueden ser 2. arterias pulmonares verdaderas; alternativamente las opciones 1, 2 y 3 3. venas pulmonares pueden ser falsas mientras la 4 es verdadera (ver 1 ). 4. estmago

En el siguiente ejemplo de pregunta de Verdadero/Falso, hay trminos vagos en las En la evaluacin clnica del dolor crnico: 1. la actitud personal del mdico respecto del opciones que le dan pistas al examinando hbil en dolor puede afectar su juicio clnico detectarlas. Por ejemplo, el trmino puede en las 2. una emocin desagradable puede convertirse opciones 1, 2 y 3 sugiere que ellas son verdaderas. en sntoma de dolor En la opcin 4 es difcil de adivinar qu significa el 3. el dolor puede tener un significado simblico trmino generalmente Las investigaciones 4. la apariencia facial o la postura corporal es realizadas han mostrado que los trminos vagos generalmente una clave de la severidad del relacionados con frecuencia no tienen una definicin dolor. clara. Los expertos no estn de acuerdo si la cuarta opcin es verdadera o falsa.

Los defectos en la siguiente pregunta pueden ser Los defectos del tabique interventricular en nios ms sutiles. La dificultad est en que el examinando estn asociados con: debe suponer ciertas condiciones como la severidad 1. soplo sistlico de la enfermedad, la edad del paciente y si la 2. hipertensin pulmonar enfermedad ha sido tratada o no. Diferentes 3. tetraloga de Fallot suposiciones conducen a distintas respuestas, incluso 4. cianosis para expertos. Ntese que en cada ejemplo de pregunta mal construida, el enunciado es poco claro, las opciones contienen trminos vagos o las opciones son parcialmente correctas. En cada instancia un grupo de expertos tuvo dificultades para alcanzar un consenso en la respuesta correcta.

En los Estados Unidos existe un dicho comn en ingls que traducido dira aproximadamente El camino hacia el corazn de un hombre es a travs de su estmago, significando que una buena comida preparada por una mujer facilita un posible romance.

Captulo 2. Los formatos de temes de Seleccin Mltiple

15

Debido a que a los examinandos se les pide seleccionar todas las opciones que son verdaderas, las preguntas de Verdadero/Falso deben satisfacer las siguientes reglas: Los enunciados 1 debe ser claros y sin ambigedad. Deben evitarse frases imprecisas como est asociado con; es til para; es importante y palabras que entregan sugerencias como puede ser; y trminos vagos tales como generalmente o frecuentemente. Las opciones deben ser absolutamente verdaderas o falsas; no est permitida ninguna sombra de duda. Evite frases y palabras sealadas en el prrafo anterior.

La familia de preguntas de una mejor respuesta


En contraste con las preguntas de Verdadero/Falso, las preguntas de una mejor respuesta (tipo A) hacen explcito el nmero de opciones que deben ser seleccionadas. Los temes de tipo A son los ms ampliamente usados del formato de los temes de seleccin mltiple. Consisten en un enunciado (por ejemplo la presentacin de un caso clnico) y una pregunta introductoria 2 , seguida de una serie de cinco opciones, tradicionalmente una respuesta correcta y cuatro distractores. La siguiente pregunta describe una situacin (en este caso, un paciente) y pide al examinando indicar la causa ms probable del problema. Enunciado: Un paciente de sexo masculino de 32 aos de edad presenta una historia de cuatro das de debilidad progresiva en sus extremidades. Ha estado en buen estado de salud, excepto por una infeccin respiratoria alta hace 10 das. Su temperatura es de 37.8 C (10F), la presin arterial es de 130/80 mmHg, el pulso es de 94/min y su frecuencia respiratoria es de 42/min y poco profunda. Presenta una debilidad simtrica de ambos lados de la cara y en los msculos proximales y distales de las extremidades. La sensibilidad es normal. No se pueden obtener reflejos tendneos profundos. Las respuestas plantares son en flexin. Pregunta introductoria: Cul de los siguientes es el diagnstico ms probable? Opciones: A. B. C. D. E. Encefalomielitis aguda diseminada Sndrome de Guillain-Barr Miastenia gravis Poliomielitis Polimiositis

1 2

[ENG: stems]. Otras acepciones: encabezamientos, bases, troncos. [ENG: lead-in question]

16

Ntese que las opciones incorrectas no estn totalmente erradas. Las opciones pueden diagramarse como sigue: D C A E B Menos Ms correcta correcta Aun cuando las respuestas incorrectas no son totalmente errneas, ellas son menos correctas que la respuesta clave 1 . El examinando est informado que debe seleccionar el diagnstico ms probable. Los expertos estaran de acuerdo en que el diagnstico ms probable es B; tambin concordaran en que los otros diagnsticos tambin podran ser algo probables, pero menos probables que B. Las opciones en las preguntas de una mejor respuesta no necesariamente tienen que ser totalmente incorrectas, siempre que las opciones puedan diagramarse en un continuo nico 2 , como el presentado ms arriba. En este caso las opciones pueden diagramarse desde Diagnstico ms probable a Diagnstico menos probable 3 . Este tem presenta defectos en su construccin. Despus de leer el enunciado, los examinandos tienen slo una vaga idea de qu trata la pregunta. En un intento por determinar la mejor respuesta, los examinandos deben decidir si ocurre frecuentemente en la mujer es ms o menos verdadero que rara vez se asocia con dolor en una articulacin. Esta es una comparacin entre manzanas y naranjas. Para ordenar la veracidad relativa de las opciones, stas deben diferir solamente en una dimensin, de lo contrario todas las opciones deben ser absolutamente 100% verdaderas o falsas.

Cul de las siguientes opciones es verdadera acerca de la seudogota? A. Ocurre frecuentemente en la mujer B. Rara vez se asocia con dolor en una articulacin C. Puede asociarse con el hallazgo de condrocalcinosis D. Es claramente hereditaria en la mayora de los casos E. Responde bien al tratamiento con alopurinol

A. Gnero E. Tratamiento D. Herencia

C Asociaciones

El diagrama de estas opciones puede verse como se ilustra en el grfico. Las opciones son heterogneas ya que se refieren a una miscelnea de situaciones diferentes que no pueden ser ordenados a lo largo de una misma dimensin, desde la opcin menos correcta a la ms correcta. Aunque esta pregunta aparenta evaluar el conocimiento desde diferentes aspectos, sus defectos lo impiden. Adems, la pregunta en s no es clara; ya que no puede responderse sin leer las opciones.

Falso . . . . . . . . . . . . . . . . . . . . . . . . . . . . . Verdadero

[ENG: keyed answer] [ENG: single continuum] 3 El continuo nico es probabilidad del diagnstico y no hay otra dimensin ms que esa.
2

Captulo 2. Los formatos de temes de Seleccin Mltiple

17

En contraste a las opciones en el tem de seudogota, las opciones en la pregunta del sndrome de GuillainBarr son homogneas (todos son diagnsticos); los examinandos bien preparados pueden ordenar 1 las opciones en una sola dimensin de acuerdo a lo que se pregunta. Las preguntas del tipo de una mejor respuesta bien construidas deben satisfacer la regla de cubrir las opciones 2 , que se refiere a que la pregunta puede responderse con slo leer el enunciado, sin necesidad de las opciones.

Balance final 3 en cuanto al formato de las preguntas


Recomendamos no usar preguntas de verdadero/falso. Mientras muchos autores de preguntas creen que las preguntas de verdadero/falso son ms fciles de construir que las preguntas de una mejor respuesta, nosotros encontramos que son ms problemticas. El autor de las preguntas tiene algo especfico en su mente cuando redact la pregunta, pero una cuidadosa revisin revela sutiles dificultades que no eran aparentes para el autor. A menudo, la diferencia entre verdadero y falso no es clara y no es raro que en las revisiones siguientes se modifique la respuesta clave. Como resultado, cuando las preguntas de Verdadero/Falso son revisadas, se vuelven a redactar o son descartadas ms frecuentemente que las preguntas desarrolladas usando otros formatos. Algunas ambigedades pueden ser corregidas, otras no. Existe una razn final ms convincente que las anteriores. Nosotros encontramos que, para evitar la ambigedad, los autores de estos temes tienden a producir preguntas que evalan el recuerdo de hechos aislados 4 -- algo que estamos activamente tratando de evitar. Nosotros encontramos que la aplicacin del conocimiento, integracin, sntesis y juicio pueden ser mejor evaluados por medio de las preguntas de una mejor respuesta. Como resultado de lo anterior, el organismo encargado de los exmenes para empezar a ejercer la medicina en Estados Unidos (NBME) no utiliza formatos de Verdadero/Falso en sus exmenes. Recomendamos tambin no usar preguntas del tipo A redactadas en forma negativa. Las ms problemticas son aquellas que tienen la forma Cada una de las siguientes es correcta EXCEPTO o Cul de las opciones NO es correcta? Estas formas sufren del mismo problema que las preguntas de Verdadero/Falso: si las opciones no pueden ser ordenadas en una dimensin comn, los examinandos no podrn determinar ni la respuesta menos correcta ni la ms correcta. Por otra parte, ocasionalmente usamos en algunos exmenes preguntas del tipo A redactadas en forma negativa, cuando las opciones contienen solamente una palabra, como un sustituto (pobre) de los temes que piden al examinando seleccionar ms de una respuesta correcta. Un formato mejor para este propsito, es el formato Seleccione N 5 , en la que los examinandos deben seleccionar N respuestas en cada pregunta. Este formato se presenta ms adelante en este manual. El apndice A ilustra una variedad de formatos que ya no se usan en los exmenes preparados por la NBME.

1 2

[ENG: rank-order] [ENG: cover-the-options] 3 [ENG: bottom line] 4 [ENG: isolated facts] 5 [ENG: Pick N]. Ver captulo 7

18

Captulo 3 Defectos tcnicos en los temes


En esta seccin se describen dos tipos de defectos 1 tcnicos en los temes: la astucia para responder en los exmenes 2 y la dificultad irrelevante 3 . Los defectos relacionados a la astucia para responder exmenes permiten a ciertos estudiantes responder correctamente preguntas basndose exclusivamente en sus habilidades para encontrar pistas en las preguntas 4 , no relacionadas al conocimiento de la materia. Este defecto se encuentra a menudo en temes cuyas opciones no son de la misma rea, es decir estn desenfocados 5 y no satisfacen el criterio de cubrir las opciones. Los defectos debidos a dificultades irrelevantes hacen que la pregunta sea ms difcil por razones no relacionadas al asunto que es el tema de la evaluacin. El propsito de esta seccin es bosquejar algunos defectos comunes y estimularlo a usted para que elimine estos defectos en sus preguntas y provea a todos sus estudiantes, tanto los que tienen como los que no tienen sagacidad o astucia para responder preguntas, de un campo parejo para demostrar sus conocimientos 6 . La probabilidad de responder a una pregunta correctamente debera relacionarse a la competencia que tiene el examinando en el contenido que est siendo evaluado y no debera depender de su habilidad para usar estrategias para rendir pruebas.

Asuntos relacionados a la astucia de los examinandos para responder exmenes


Pistas gramaticales: cuando uno o ms distractores no concuerdan gramaticalmente con el enunciado Debido a que los que desarrollan temes tienden a poner ms atencin en la respuesta correcta que en los distractores, es ms probable que los errores gramaticales ocurran en estos ltimos. En este ejemplo, los estudiantes con astucia para responder preguntas deberan eliminar A y C como opciones ya que no concuerdan gramaticalmente o lgicamente con el enunciado. As, este tipo de estudiantes escoger slo entre B, D, y E. Un hombre de 60 aos es trado al servicio de urgencia por la polica, por haberlo encontrado inconsciente en la vereda 7 . Despus de asegurar la va area permeable, el primer paso a seguir en el manejo de este paciente es la administracin por va intravenosa de: A. examinar el lquido cfalo raqudeo B. glucosa con vitamina B1 (tiamina) C. una tomografa axial computarizada 8 de cabeza D. fenitona E. diazepam

1 2

[ENG: flaws] [ENG: testwiseness] 3 [ENG: irrelevant difficulty] 4 [ENG: test-taking skills] 5 [ENG: unfocused] 6 [ENG: provide a level playing field for the testwise and the not-so-testwise students] 7 [ENG: sidewalk] 8 [ENG: CT scan]

Captulo 3. Defectos tcnicos en los temes.

19

Pistas lgicas: un grupo de las opciones incluye todas las posibilidades 1 . En este tem las opciones A, B, y C incluyen todas las posibilidades. El estudiante con astucia para responder exmenes se da cuenta que la respuesta correcta tiene que estar entre A, B, o C, mientras que aquellos estudiantes sin astucia para responder exmenes 2 pierden tiempo al considerar igualmente D y E como opciones verosmiles. Los que redactan preguntas, con frecuencia agregan las opciones D y E slo porque desean una pregunta con cinco opciones. En esta situacin, los autores no se daran cuenta de los mritos relativos de las opciones D y E. Con frecuencia estas opciones pueden ser parcialmente correctas y dar origen a confusin por parte del examinando porque no pertenecen a la misma dimensin que las opciones A, B, y C. Estos defectos se ven a menudo en temes que contienen opciones con vocablos como aumenta, disminuye o se mantiene constante.

La delincuencia: A. se distribuye por igual entre todas las clases sociales B. es ms frecuente entre los pobres C. es ms frecuente en la clase media y los ms ricos D. es un indicador de desajuste psicosexual E. est llegando a un nivel mximo tolerable en la nacin

Expresiones absolutas: se usan palabras como siempre o nunca en las opciones. En esta pregunta, las opciones A, B y E contienen trminos que son menos absolutos que los de las opciones C y D. El estudiante con astucia para responder exmenes eliminar las opciones C y D como una posibilidad de respuesta correcta porque tienen una menor probabilidad de ser verdaderos en relacin a otras opciones con expresiones menos absolutistas. Ntese que este defecto no estara presente si el enunciado fuera enfocado 3 y las opciones fueran breves. Esto sucede solamente cuando se usan verbos en las opciones en lugar de ponerlos en la pregunta introductoria. El defecto de la memoria en pacientes con una demencia avanzada tipo Alzheimer: A. puede tratarse adecuadamente con fosfatidilcolina (lecitina) B. podra ser una secuela de un Parkinson precoz C. se asocia a una ausencia de nudos neurofibrilares en la autopsia D. nunca es grave E. es posible que comprometa al sistema colinrgico

1 2

[ENG: Logical cues: a subset of the options are collectively exhaustive] [ENG: non-testwise student] 3 [ENG: focused]

20

Respuesta correcta ms larga que las dems opciones: la respuesta correcta es de mayor longitud, ms especfica o ms completa que las otras opciones. En esta pregunta, la opcin C es de mayor longitud que las otras opciones. Adems es la nica que contiene dos elementos. Los autores de preguntas tienden a poner mayor cuidado en la respuesta correcta que en los distractores. Quizs por una tendencia de los autores a ser didcticos, agregan material adicional, informacin de importancia secundaria, advertencias, etc. A veces esto se lleva a la exageracin cuando la respuesta correcta tiene la longitud de un prrafo mientras las opciones contienen una sola palabra. La ganancia secundaria 1 es: A. sinnimo de fingir estar enfermo 2 B. frecuente en los trastornos obsesivocompulsivos C. una complicacin de una variedad de enfermedades y adems tiende a prolongarlas D. improbable en el dao orgnico cerebral

Repeticin de palabras: una palabra o frase que est en el enunciado se repite entre las opciones. Esta pregunta tiene la palabra irreal en el enunciado y desrealizacin en la respuesta correcta. A veces una palabra se repite slo en un sentido metafrico, por ejemplo, si en el enunciado se menciona dolor seo y la respuesta correcta contiene una palabra con el prefijo osteo- Un paciente de 58 aos de sexo masculino, con una historia de consumo excesivo de alcohol y una hospitalizacin previa por motivos psiquitricos se presenta con confusin y agitacin. l dice que encuentra que el mundo es irreal 3 . Este sntoma se llama: A. despersonalizacin 4 B. descarrilamiento 5 C. desrealizacin 6 * D. dficit focalizado de memoria E. ansiedad de seal 7

1 2

[ENG: secondary gain] [ENG: malingering] 3 [ENG: he speaks of experiencing the world as unreal] 4 [ENG: depersonalization] 5 [ENG: derailment] 6 [ENG: derealization] 7 [ENG: signal anxiety]

Captulo 3. Defectos tcnicos en los temes.

21

Estrategia de convergencia: la respuesta correcta contiene el mayor nmero de elementos comunes con las otras opciones. Este defecto es menos obvio que los anteriores, pero ocurre con frecuencia y es preciso considerarlo. Se manifiesta en mltiples formas. La premisa fundamental es que la respuesta correcta contiene el mayor nmero de elementos comunes con las otras opciones. En este caso es poco probable que esta opcin sea una opcin sin la oportunidad de ser 1 la respuesta correcta. Por ejemplo, en opciones que contienen nmeros, la respuesta correcta se ubica ms a menudo en el centro que en las cifras extremas. En las opciones con ms de un elemento, es ms probable que la respuesta correcta sea la opcin que tiene ms elementos en comn con las otras opciones. Veamos un ejemplo: si las opciones fueran lpiz y lapicera, lpiz y destacador (o marcador) 2 , lpiz y lpiz de cera 3 , lapicera y rotulador 4 (o plumn). En este caso es probable que la respuesta correcta es lpiz y lapicera (lpiz aparece en tres ocasiones entre las opciones, lapicera dos veces y los otros elementos slo una). Aunque este ejemplo aparezca ridculo, este defecto se produce porque los autores de preguntas comienzan con la respuesta correcta y luego redactan permutaciones de la respuesta correcta como distractores. Por lo tanto es probable que la respuesta correcta termine teniendo ms elementos comunes con las otras opciones. Por otro lado, los distractores que se redactan al final en una pregunta tienen ms probabilidad de ser menos relevantes que los otros por las dificultades que se encuentra para generar varios buenos distractores 5 . En el ejemplo de al lado, el estudiante con astucia para responder exmenes eliminara forma aninica Los anestsicos locales son ms efectivos en la: A. forma aninica, actuando desde dentro de por poco probable porque forma aninica la membrana del nervio aparece solamente una vez. Este estudiante B. forma catinica, actuando desde dentro de adems excluira fuera de la membrana del la membrana del nervio nervio porque fuera aparece con menor C. forma catinica, actuando desde fuera de frecuencia que dentro. Por lo tanto, el estudiante la membrana del nervio tendra que elegir slo entre las opciones de B y D. D. forma sin carga, actuando desde dentro de Como tres de las cinco opciones mencionan una la membrana del nervio forma con carga, el estudiante optara por elegir la E. forma sin carga, actuando desde fuera de opcin B. la membrana del nervio

Asuntos relacionados a dificultades irrelevantes


Las opciones son largas, complicadas, o dobles El siguiente tem demuestra un defecto bastante comn. El enunciado presenta palabras irrelevantes, pero lo ms importante es que las opciones son demasiado largas y complicadas. Tomara un buen tiempo leer las opciones porque contienen varios elementos en su construccin. Esto hace que se corra el peligro de transformar lo que se est tratando de evaluar en un proceso de medicin de velocidad de lectura. Hgase notar que este defecto slo corresponde a las opciones, ya que hay buenas preguntas que tienen un enunciado relativamente largo. Si el propsito de la pregunta es evaluar si el estudiante es capaz de interpretar y

1 2

[ENG: outlier] [ENG: highlighter] 3 [ENG: crayon] 4 [ENG: marker] 5 [ENG: the incorrect answers are more likely to be outliers as the item writer has difficulty generating viable distractors]

22

sintetizar informacin para, por ejemplo, determinar el diagnstico ms probable, entonces se acepta que el enunciado incluya una descripcin bastante completa de la situacin.

Los comits de revisin por pares en los sistemas de salud administrada 1 pueden tomar acciones adversas contra un mdico impidindole ejercer su profesin dentro de ese sistema. Existe adems un requerimiento de que el mdico reciba un debido proceso 2 . Un debido proceso debe incluir cul de las siguientes opciones: A. Una notificacin, un foro imparcial, un consejo 3 y una oportunidad para escuchar y rebatir la evidencia acusatoria. B. Una notificacin apropiada, un tribunal con poder de decisin, una oportunidad de confrontar a los testigos, y una oportunidad de presentar evidencia de apoyo a la defensa. C. Una notificacin oportuna y razonable, un panel imparcial con poder de decisin, una oportunidad de escuchar la evidencia acusatoria e interrogar a los testigos, y la oportunidad de presentar evidencia en la defensa.

Las opciones que contienen nmeros no se presentan en forma consistente Cuando se usan opciones numricas, las opciones debieran redactarse en orden numrico y presentarse en un formato similar (ya sea nmeros, porcentajes, o rangos numricos). Se produce confusin cuando se mezclan formatos y cuando las opciones se presentan en un orden no lgico o en un formato no consistente. En este ejemplo, las opciones A, B, y C se expresan como rangos mientras que las opciones D y E estn en porcentajes especficos. Todas las opciones debieran expresarse como rangos o como porcentajes especficos. No se recomienda mezclarlos. Adems, en esta pregunta el rango de la opcin C incluye las opciones D y E, lo que casi con certeza excluye las opciones D y E como respuesta correcta. Cul es la probabilidad de que una mujer quede infrtil despus de un segundo episodio de infeccin? A. Menos de 20% B. 20 a 30% C. Mayor de 50% D. 90% E. 75%

1 2

[ENG: Peer review committees in HMOs] [ENG: due process] 3 [ENG: council]

Captulo 3. Defectos tcnicos en los temes.

23

Los trminos que expresan frecuencia son vagos (por ejemplo: raramente, habitualmente). Investigaciones realizadas han mostrado que aquellos trminos que expresan frecuencia en forma vaga no son definidos ni interpretados consistentemente, an por los expertos. Una discusin ms completa de los resultados de estas investigaciones se presenta ms adelante (pg. 27). La obesidad severa en la adolescencia temprana A. habitualmente responde en forma dramtica a un rgimen diettico B. a menudo est relacionada con enfermedades endocrinas C. tiene un 75% de probabilidad de que desaparezca espontneamente D. tiene un mal pronstico E. habitualmente responde a la farmacoterapia y psicoterapia intensiva

El lenguaje usado en redactar las opciones no es paralelo; las opciones estn en un orden no lgico. Esta pregunta ilustra un defecto comn en el cual las opciones son largas y el lenguaje hace dificultoso y demoroso el determinar cul es la respuesta ms correcta. En general, este defecto puede corregirse editando la pregunta. En este caso particular la pregunta introductoria en el enunciado podra cambiarse a Por cul de las siguientes razones no puede obtenerse conclusiones de este resultado? Las opciones podran editarse de la siguiente manera: A. No se hizo ningn seguimiento de nios no vacunados, B. El nmero de casos es demasiado pequeo, C. El ensayo incluy slo a nios de sexo masculino, y se podra escribir otra opcin para D. En un ensayo clnico para determinar la efectividad de una vacuna, 200 nios de 2 aos de sexo masculino recibieron una vacuna contra una enfermedad determinada. Se realiz un seguimiento por 5 aos para ver si desarrollaban o no esta enfermedad. De este grupo, el 85% de los casos no la contrajo. Cul de las siguientes opciones en relacin a este resultado es correcta? A. No se puede obtener conclusiones ya que no hubo seguimiento de nios que no recibieron la vacuna. B. El nmero de casos (30 en 5 aos) es demasiado pequeo para obtener resultados estadsticamente significativos. C. No se puede extraer conclusiones porque el ensayo slo se hizo en nios de sexo masculino. D. La eficacia de la vacuna (%) se calcula como 85-15/100.

24

Ninguna de las anteriores se usa como opcin El uso de la frase ninguna de las anteriores como una opcin produce problemas en preguntas en que Cul ciudad est ms cercana a Nueva York? se requiere aplicar una actitud juiciosa y en que las A. Boston opciones no son absolutamente verdaderas o falsas. B. Chicago Si la respuesta correcta es una de las otras opciones, an teniendo el conocimiento del contenido, el que C. Dallas responde se enfrentar a un dilema porque debe D. Los ngeles decidir entre una opcin perfecta y detallada contra E. Ninguna de las anteriores aquella que el autor de la pregunta considera correcta. Y a menudo los que responden las preguntas son capaces de pensar en otra opcin ms Si los estudiantes escogen E, uno no sabe si estn correcta que la propuesta entre las opciones. El usar pensando en Filadelfia (que est cerca) o Londres ninguna de las anteriores convierte la pregunta en (que est bastante lejos). una de verdadero/falso, en que cada opcin tiene que ser evaluada como ms o menos verdadera que el universo de opciones que no estn en la lista de opciones. A menudo es posible corregir esta situacin mediante el reemplazo de ninguna de las anteriores por una opcin que tenga mas o menos el mismo significado, pero es ms especfica. Por ejemplo, en un tem en que se le pide al examinando que especifique una farmacoterapia ms apropiada, se podra reemplazar ninguna de las anteriores por no se debe dar ningn medicamento en este momento lo que eliminara la ambigedad de ninguna de las anteriores. Los enunciados son innecesariamente complicados y engaosos Ordene los padres de hijos con sndrome de Down en orden de mayor a menor riesgo de reaparicin. Para todos los casos la edad de la madre es de 22 aos y el siguiente embarazo ocurre dentro de los 5 aos siguientes. Los cariotipos de las hijas son I. 46, XX, -14, +T(14q21q) pat II. 46, XX, -14, +T(14q21q) de novo III. 46, XX, -14, +T(14q21q) mat IV. 46, XX, -21, +T(14q21q) pat V. 47, XX, -21, +T(21q21q) (no se hizo cariotipo de los padres) A. B. C. D. III, IV, I, V, II IV, III, V, I, II III, I, IV, V, II IV, III, I, V, II III, IV, I, II, V

Ocasionalmente, los autores toman una pregunta fcil y la convierten en algo tan rebuscado que slo los ms obstinados leern la pregunta. Este es un ejemplo de ese tipo de tem. El sistema usado para presentar las opciones desde I a V es complejo. Tener que ordenar los nmeros romanos despus de interpretar este sistema es irrelevante.

E.

Captulo 3. Defectos tcnicos en los temes.

25

Resumen de los defectos tcnicos en las preguntas


Asuntos relacionados a la astucia para responder exmenes Pistas gramaticales uno o ms de los distractores no siguen gramaticalmente al enunciado. Pistas lgicas un subgrupo de las opciones incluyen todas las posibilidades Trminos absolutos - trminos como siempre o nunca estn presentes en algunas opciones. Respuesta correcta de mayor longitud la respuesta correcta es la de mayor longitud, la ms especfica, o la ms completa entre las opciones. Repeticin de palabras una palabra o frase est incluida en el enunciado y en la respuesta correcta. Estrategia de convergencia la respuesta correcta incluye el mayor nmero de elementos comunes con las otras opciones.

Asuntos relacionados a dificultades irrelevantes Opciones demasiado largas, complicadas y con ms de un concepto Opciones con nmeros expresadas en forma inconsistente Algunos trminos en las opciones son vagos (p. ej. raramente, habitualmente). Construccin usando lenguaje no comparable (paralelo) en las opciones Opciones sin ordenamiento lgico Uso de Ninguna de las anteriores como opcin Enunciados engaosos o innecesariamente complicados La respuesta de una pregunta depende de la respuesta a otra pregunta

Pautas generales para construir preguntas Asegrese que el tem puede responderse sin leer las opciones o que las opciones sean 100% verdaderas o falsas. Incluya la mayora del tem en el enunciado; el enunciado debe ser largo y las opciones cortas. Evite incorporar informacin superflua. Evite temes engaosos y demasiado complicados. Redacte opciones que sean gramticamente consistentes y lgicamente compatibles con el enunciado, listarlas en orden lgico o alfabtico. Redacte distractores que sean verosmiles y de la misma longitud que la respuesta correcta. Evite trminos absolutos como siempre, nunca, y todos en las opciones, tambin evite trminos vagos tales como habitualmente y frecuentemente. Evite frases expresadas en trminos negativos (por ejemplo, aquellas que contienen excepto o no en el enunciado). Si cree que debe usar un enunciado negativo, use de preferencia opciones cortas, incluso de una sola palabra.

Y lo ms importante: Concentre sus esfuerzos en conceptos importantes. No pierda el tiempo en examinar asuntos triviales.

26

Uso de trminos imprecisos en preguntas de exmenes


Aunque en el hablar cotidiano y en la escritura se usan trminos imprecisos, stos causan confusin al ser usados en preguntas de exmenes. En un estudio que se realiz en el NBME, 60 miembros de 8 comits encargados de redactar preguntas en varias especialidades revisaron una lista de trminos que se usaron en temes de seleccin mltiple para expresar algn concepto relativo a la frecuencia con que ocurre una situacin y se les pidi que indicaran el porcentaje de tiempo reflejado en cada trmino. Los resultados (que se muestran ms adelante) indican que estos trminos no tienen una definicin operacional compartida por todos por igual, incluso entre los redactores de temes. El valor promedio, ms o menos una desviacin estndar, excedi 50 puntos porcentuales para ms de la mitad de los trminos relativos a la frecuencia. Por ejemplo, los redactores de temes dijeron que el trmino frecuentemente indicaba 70% del tiempo, con la mitad diciendo que estaba entre 45% y 75% del tiempo. Todas las respuestas para este trmino estuvieron en un rango entre 20 a 80%. De mayor importancia es el hecho que los valores encontrados para frecuentemente se sobrepusieron a aquellos obtenidos para el trmino raramente. La consecuencia de estos resultados para la construccin de preguntas en un examen vara segn el formato de pregunta. Los trminos vagos crean mucho ms problemas en los diversos tipos de temes Verdadero/Falso (temes de tipo K, C y X) 1 que en los temes de una mejor respuesta correcta (tipos A y R). Por ejemplo, trminos imprecisos causan problemas importantes en preguntas de Verdadero/Falso como en este ejemplo.

Opciones verdaderas en la seudogota incluyen: 1. ocurre comnmente en el sexo femenino 2. se asocia a menudo con dolor agudo 3. es habitualmente hereditaria 4. los niveles sricos de calcio estn frecuentemente elevados

En preguntas de verdadero/falso el examinando tiene que juzgar si cada opcin es verdadera o falsa. Cuando las opciones no son absolutamente verdaderas o falsas, los examinandos utilizan sus definiciones personales de los trminos ambiguos o intentan adivinar lo que esos trminos significan para el que redact la pregunta. Alternativamente, las respuestas de los examinandos pueden reflejar los estilos personales de formas de responder (la tendencia a responder ya sea verdadero o falso cuando se desconoce la respuesta correcta). Estos factores de estilos de responder pueden tener un efecto mayor que el conocimiento mismo de la materia para determinar si el examinando responde correctamente una pregunta y puede ser parte de la razn que hace que las preguntas de verdadero/falso tienden a producir resultados mediocres 2 .

1 2

Ver Apndice A [ENG: perform poorly]

Captulo 3. Defectos tcnicos en los temes.

27

El problema no se corrige si se reformulan las opciones usando nmeros exactos. Por ejemplo, la afirmacin la incidencia en el sexo femenino es de 1:2000 no sera una rectificacin apropiada para la opcin 1 en el ejemplo anterior. La incidencia no es exactamente 1:2000 y, porque no se especifica una banda, los examinandos van a definir sus propias bandas, de forma ajustada o amplia, presumiblemente de acuerdo a los estilos de respuesta propios de cada examinando. Para temes de verdadero/falso, el manejo adecuado de opciones numricas es, ya sea generar una comparacin (por ejemplo, la incidencia es mayor que la de la osteoartritis) o especificando una banda (por ejemplo, la incidencia es entre 1:1000 y 1:2000). El asunto mencionado anteriormente con temes de verdadero/falso no se produce con la misma intensidad en temes de una mejor respuesta (o sea aquellos que contienen una pregunta clara y opciones homogneas). Por ejemplo, la pregunta siguiente contiene un trmino vago en el enunciado, sin embargo, porque la tarea del examinando es encontrar la mejor respuesta, la pregunta resulta relativamente inequvoca. Cul de los siguientes resultados de laboratorio se encuentra generalmente elevado en pacientes con seudogota?

Se producen dificultades si los temes de una mejor respuesta presentan trminos vagos en las opciones, como en este ejemplo. La nica manera que el tem se puede hacer an ms ambiguo es agregar una quinta opcin ninguna de las anteriores.

El dolor en pacientes con seudogota es A. frecuente B. habitual C. a menudo D. comn

28

El grfico de cajas muestra la distribucin de las respuestas para los trminos relacionados con el concepto de frecuencia. Estos resultados estn basados en las respuestas de 60 miembros de los comits encargados de redactar las preguntas en el NBME. La lnea horizontal en cada caja corresponde a la mediana de las respuestas. Los lmites superior e inferior corresponden al 50% de las respuestas. Las lneas verticales muestran la amplitud de todas las respuestas. Por ejemplo, la mediana de frecuentemente corresponde al 70% de las veces; la mitad de las respuestas estuvieron entre 45% y 75% de las veces y todas las respuestas estuvieron entre 20% y 80% de las veces, lo que muestra que casi se tocaron con las respuestas de raramente.
De: Case SM. (1994) The use of imprecise terms in examination questions: How frequent is frequently? Academic Medicine, 69(suppl):S4-S6

Captulo 3. Defectos tcnicos en los temes.

29

30

Seccin II Cmo redactar preguntas de una mejor respuesta para las Ciencias Bsicas y Ciencias Clnicas

Los captulos precedentes delinearon los aspectos tcnicos relacionados a la construccin de preguntas de seleccin mltiple. Esta seccin est enfocada al contenido de las preguntas.

32

32

Reglas bsicas para los temes de una mejor respuesta


Cada tem debiera enfocar un concepto importante, tpicamente un problema clnico comn o potencialmente catastrfico. No pierda el tiempo con preguntas que evalan asuntos triviales. Cada tem debiera evaluar la aplicacin del conocimiento y no el recuerdo de un hecho aislado 1 . El enunciado de los temes debiera ser relativamente largo y las opciones, cortas. Las vietas 2 clnicas proveen una buena base para desarrollar una pregunta. Para las ciencias clnicas, el enunciado debiera comenzar con un problema que presenta un paciente, seguido de la historia clnica (incluyendo duracin de los signos y sntomas), hallazgos del examen fsico, resultados de los estudios diagnsticos, tratamiento inicial, hallazgos subsiguientes, etc. Las vietas clnicas pueden incluir solamente un subconjunto de esta informacin, pero la informacin debe darse en el orden mencionado anteriormente. Para las ciencias bsicas las vietas pueden ser breves. Las vietas de laboratorio tambin son una alternativa apropiada. El enunciado de los temes debe incluir una pregunta hecha en forma clara y debe ser posible dar la respuesta correcta sin mostrar las opciones. Para determinar si la pregunta est bien enfocada, cubra las opciones y vea si la pregunta es clara y si el examinando es capaz de dar la respuesta correcta sin ver las opciones, basndose solamente en la lectura del enunciado. Proceda a volver a redactar el enunciado si los examinandos no son capaces de hacer esto. Los distractores 3 (las opciones incorrectas) debieran ser homogneos. Debieran estar en la misma categora que la respuesta correcta (ejemplo: slo diagnsticos, tests, tratamientos, pronsticos, decisiones sobre un siguiente paso a tomar 4 . Vuelva a redactar cualquier distractor que no sea similar a la respuesta correcta. Evite usar doble opciones (por ejemplo, haga W y X; haga Y debido a Z) a menos que la respuesta correcta y todos los distractores sean del formato doble opcin. Vuelva a redactar las preguntas que contienen doble opciones para representar un solo punto. Todos los distractores debieran ser verosmiles, gramticamente correctos, lgicamente compatibles y del mismo largo (relativo) que la respuesta correcta. Ordene las opciones lgicamente (por ejemplo, numricamente) o en orden alfabtico. Evite defectos de los temes que beneficien a aquellos examinandos que son astutos en percibir los defectos de construccin y as guiarse hacia la pregunta correcta. Evite asimismo defectos que resultan en preguntas difciles slo por fallas en su construccin.

NO redactar preguntas en la forma Cual de las siguientes opciones es correcta? o Todas las siguientes opciones son correctas EXCEPTO: Estas preguntas no estn bien enfocadas y resultan en opciones heterogneas. Idealmente debiera someterse cada pregunta a los cinco tests mencionados arriba. Si la pregunta sortea con xito los cinco, est probablemente bien enunciada y enfocada en el tpico apropiado.

1 2

[ENG: fact] [ENG: vignettes] 3 [ENG: distractors] 4 [ENG: disposition alternatives]

33

Ver tambin:
Swanson DB, Case SM. Assessment in basic science instruction: directions for practice and research. Advances in Health Sciences Education: Theory and Practice. 1997; 2:71-84.

34

Captulo 4 Contenido de los temes: La evaluacin de la aplicacin del conocimiento de Ciencias Bsicas
Contenido de los temes de Ciencias Bsicas
Tradicionalmente, los temes se clasifican mediante el tipo de proceso cognitivo que se requiere para responder a una pregunta (por ejemplo, recordar, interpretar o solucionar problemas; memoria, comprensin o razonamiento). Se considera que los temes que usan el proceso de recordar examinan el conocimiento de hechos o datos aislados por el examinando. Los temes de interpretacin requieren que los examinandos revisen la informacin dada (a menudo en forma de tablas o grficos) y que lleguen a una conclusin (por ejemplo, un diagnstico). Los temes de resolucin de problemas presentan una situacin y requieren que los examinandos formulen algn tipo de accin (por ejemplo, el siguiente paso en el manejo de un paciente). Se piensa que los temes de interpretacin y de resolucin de problemas involucran habilidades ms elevadas que aqullos que requieren meramente que se memorice informacin de datos. Desafortunadamente, a menudo es difcil determinar cules son los procesos cognitivos que se requieren para responder a un tem porque dependen tanto de los antecedentes 1 del examinando como del contenido del tem. Por ejemplo, un tem concerniente al flujo sanguneo en un paciente con un defecto del tabique interventricular podra requerir simplemente recordar sin mayor esfuerzo por parte de un especialista en cardiologa peditrica o de un fisilogo cardiovascular, pero un alumno con slo unos pocos aos de estudios de medicina podra tener que deducir la respuesta correcta a partir de principios bsicos de hemodinmica. Los procesos cognitivos usados en responder una pregunta van a variar entre los examinandos, lo que hace difcil usar este enfoque taxonmico. Una manera ms simple y objetiva de enfrentar la clasificacin de los temes es considerando la tarea 2 que se le solicita al examinando. Si un tem requiere que el examinando llegue a una conclusin, haga una prediccin o seleccione un curso de accin, el tem debiera clasificarse como un tem de aplicacin del conocimiento. Si un tem examina solamente el recordar hechos aislados (sin requerir su aplicacin), debiera clasificarse como un tem de memoria. Todos los temes debieran requerir la aplicacin de conocimientos, permitiendo que el examinando no slo demuestre su base de conocimientos sino tambin la habilidad para usar la informacin.

1 2

[ENG: background] [ENG: task]

Captulo 4. Contenido de los temes: La evaluacin de la aplicacin del conocimiento de Ciencias Bsicas

35

El siguiente par de enunciados ilustra la diferencia entre una pregunta que evala el recuerdo de un hecho aislado de otra que evala la aplicacin de conocimientos. Enunciado de un tem de memoria en ciencias bsicas: Qu rea es irrigada por la arteria cerebelosa inferior? Enunciado de un tem de aplicacin de conocimientos en ciencias bsicas: Un hombre de 62 aos desarrolla una ataxia de las extremidades del lado derecho, sndrome de Horner, nistagmo, y la prdida de la sensibilidad al dolor y a la temperatura. Qu arteria es la ms probable que est ocluida? Es comn usar vietas clnicas como enunciados de un tem para evaluar la aplicacin del conocimiento de ciencias bsicas en la interpretacin de situaciones clnicas. Por ejemplo, en lugar de preguntar a los examinandos que identifiquen los msculos inervados por un nervio craneano, se les puede entregar un conjunto de hallazgos del examen fsico y luego pedirles que identifiquen el sitio de la lesin. En lugar de preguntar por una descripcin de la acidosis o alcalosis respiratoria se les puede entregar los resultados de un anlisis de gases en sangre arterial (ms otros hallazgos pertinentes segn sea necesario) y pedirles que identifiquen la ms probable explicacin fisiopatolgica. Hay que asegurarse que los examinandos puedan contestar la pregunta con sus conocimientos de ciencias bsicas. La experiencia clnica no debera ser necesaria. Las vietas de laboratorio tambin pueden ser tiles para preparar temes que examinan la aplicacin de conocimientos. Estos temes presentan experimentos de laboratorio y requieren del examinando que utilice su comprensin de los principios de las ciencias bsicas para predecir o explicar los resultados. Las vietas pueden describir experimentos clsicos del rea de ciencias bsicas o tambin situaciones menos conocidas o an hipotticas. Este tipo de temes cambia el foco de la evaluacin desde el conocimiento de hechos aislados hacia el uso de los principios de ciencias bsicas para resolver problemas. El uso de vietas clnicas y de laboratorio para evaluar la aplicacin de conocimientos tiene varias ventajas. Primero, la validez de apariencia 1 del examen aumenta al usar temes de resolucin de problemas. Segundo, es ms probable que los temes se enfoquen en informacin de importancia en lugar de trivialidades 2 . Tercero, sirve para identificar a aquellos examinandos que han memorizado una gran cantidad de datos pero que no pueden utilizar la informacin en forma efectiva.

1 2

[ENG: face validity] [ENG: trivia]

36

Pautas para guiar el contenido de los temes de ciencias bsicas.


Evale la aplicacin de conocimientos usando vietas clnicas y vietas con experimentos de laboratorio. Desarrolle temes sobre conceptos importantes o fundamentales 1 y principios que todos los examinandos debieran comprender (sin la ayuda de referencias). Evale material relevante al aprendizaje relacionado con la prctica clnica de pre y postgrado. Evite temes que solamente requieran el recordar datos aislados. Evite tpicos esotricos o tpicos que, aunque interesantes, no son esenciales. Recomendamos que los temes se

Estos dos temes fueron redactados para evaluar el mismo tpico. desarrollen usando el segundo ejemplo y no el primero.

La porfiria aguda intermitente es el resultado de un defecto en la biosntesis de: A. colgeno B. corticoesteroides C. cidos grasos D. glucosa E. heme * F. tiroxina (T4)

Un hombre de 33 aos que se encontraba en buena salud manifiesta debilidad leve y episodios ocasionales de severo dolor abdominal tipo calambre 2 , sin diarrea. Una ta y un primo han sufrido episodios similares. Durante cada episodio presenta abdomen distendido con disminucin de ruidos intestinales. Al examen neurolgico se encuentra una debilidad leve de las extremidades superiores. Estos hallazgos sugieren un defecto en la biosntesis de: A. colgeno B. corticoesteroides C. cidos grasos D. glucosa E. heme * F. tiroxina (T4)

1 2

[ENG: key concepts] [ENG: cramping]

Captulo 4. Contenido de los temes: La evaluacin de la aplicacin del conocimiento de Ciencias Bsicas

37

Plantillas de temes 1
La estructura general de un tem puede ser representada por una plantilla de tem, pudiendo generarse muchos temes usando la misma plantilla. Por ejemplo, la siguiente plantilla puede usarse para generar una serie de preguntas relacionadas con anatoma macroscpica: Un (descripcin del paciente) no puede (describir la discapacidad funcional) 2 . siguientes opciones representa mejor la lesin en este paciente? Esta es una pregunta que podra ser redactada usando esta plantilla: Cul de las

Un hombre de 65 aos presenta dificultad al levantarse y enderezar su tronco desde la posicin sentada, pero no presenta dificultad para flexionar sus piernas. Cul de los siguientes msculos es el ms probable que est comprometido? * A. Gluteus maximus B. Gluteus minimus C. Posterior del muslo 3 D. Iliopsoas E. Obturator internus

Muchas preguntas de ciencias bsicas pueden ser redactadas en el contexto de una vieta clnica. Las vietas de un paciente pueden incluir alguno o todos de los siguientes componentes: Edad, sexo (por ejemplo: un hombre de 45 aos) Sitio donde se encuentra el paciente (por ejemplo: en el servicio de urgencia) Motivo de consulta (por ejemplo: cefalea) Duracin (por ejemplo: desde hace dos das) Historia (con historia familiar?) Hallazgos del examen fsico +/- Resultados de los estudios diagnsticos +/- Tratamiento inicial, hallazgos subsiguientes, etc.

1 2

[ENG: Item Templates] [ENG: functional disability] 3 [ENG: Hamstrings]

38

Plantillas adicionales
Un (descripcin del paciente) presenta (tipo de lesin y localizacin). Cul de las siguientes estructuras es ms probable que est comprometida? Un (descripcin del paciente) presenta (hallazgos de la historia clnica). Recibe (medicamentos). Cul de los siguientes medicamentos es la causa ms probable de (hallazgo en la historia, examen fsico, o de laboratorio)?. Un (descripcin del paciente) presenta (hallazgos anormales). diagnstico de (enfermedad 1) en vez de (enfermedad 2)? Cul hallazgo [adicional] sugerira el

Un (descripcin del paciente) presenta (sntomas y signos). Estas observaciones sugieren que su enfermedad es el resultado de (una ausencia o presencia) de cul de las/los siguientes (enzimas, mecanismos). Un (descripcin del paciente) cumple con (un determinado rgimen diettico). Cul de las siguientes situaciones es ms probable que ocurra? Un (descripcin del paciente) presenta (sntomas, signos, o una enfermedad especfica) y est recibiendo tratamiento con (medicamento o clase de medicamento). La droga acta mediante la inhibicin de cul de los/las siguientes (funciones, procesos). Un (descripcin del paciente) presenta (hallazgos anormales). Cul de los siguientes (resultados positivos de laboratorio) se esperara? (perodo de tiempo) despus de (un evento tal como un viaje o una comida con ciertos alimentos), un (paciente o una descripcin de un grupo de individuos) presenta (sntomas y signos). Cul de los siguientes (organismos, agentes) es el ms probable que se encuentre en un anlisis de (alimento)? Despus de (procedimiento), un (descripcin del paciente) desarrolla (sntomas y signos). Los hallazgos de laboratorio muestran (hallazgos). Cul de las siguientes causas es la ms probable? Un (descripcin del paciente) fallece a causa de (enfermedad). Cul de los siguientes hallazgos es ms probable que se encuentre en la autopsia? Un paciente presenta (sntomas y signos). Cul de las siguientes es la ms probable explicacin de su (hallazgos)? Un (descripcin del paciente) presenta (sntomas y signos). Una exposicin a cul de los siguientes (agentes txicos) es la causa ms probable? Cul de los siguientes es el mecanismo ms probable del efecto teraputico de esta (clase de medicamento) en pacientes con (enfermedad)? Un paciente presenta (hallazgos normales) y (hallazgos anormales). Cul es el diagnstico ms probable? Ejemplos adicionales se encuentran en el Apndice B. Captulo 4. Contenido de los temes: La evaluacin de la aplicacin del conocimiento de Ciencias Bsicas 39

Tipos de preguntas
Determine el medicamento Determine la exposicin a un agente txico Determine el tipo de dieta Determine el estado de nimo 1 Pronostique 2 los hallazgos del examen fsico Pronostique los hallazgos de laboratorio Pronostique las secuelas Identifique causas o diagnsticos subyacentes Identifique la causa de una respuesta a ciertos medicamentos Indique el medicamento a prescribir

Ejemplos de preguntas introductorias3 y listas de opciones


Cul de lo siguiente presenta (una anormalidad)? El conjunto de opciones podra incluir sitios de lesiones; lista de nervios; lista de msculos; lista de enzimas; lista de hormonas; tipos de clulas; lista de neurotransmisores; lista de toxinas; segmentos espinales. Cul de los siguientes hallazgos es el ms probable? El conjunto de opciones podra incluir lista de resultados de laboratorio; lista de hallazgos adicionales del examen fsico; resultados de la autopsia; resultados del examen microscpico de lquidos corporales, msculo, o tejido de una articulacin; resultados de anlisis de ADN; niveles en el suero. Cul de las siguientes es la causa ms probable? El conjunto de opciones podra incluir una lista de los mecanismos subyacentes de enfermedad; medicamentos que podran producir efectos laterales; medicamentos o clases de medicamentos; agentes txicos; mecanismos hemodinmicos, virus, defectos metablicos. Cul de lo siguiente debera administrarse? El conjunto de opciones podra incluir medicamentos, vitaminas, aminocidos, enzimas, hormonas. Cul de lo siguiente presenta algn defecto? El conjunto de opciones podra incluir una lista de enzimas, mecanismos de retroalimentacin, estructuras endocrinas, elementos de la dieta, vitaminas. Dado un rbol genealgico 4 cul es la probabilidad de que el prximo hijo (especificar sexo) que nazca presentar la misma enfermedad?

1 2

[ENG: mood] [ENG: Predict] 3 [ENG: lead-ins] 4 [ENG: pedigree]

40

Redaccin de las opciones: como modificar la dificultad de un tem.


Las opciones incorrectas en cada pregunta se llaman distractores. Cada distractor en una pregunta debera ser seleccionado al menos por algunos examinandos. Por lo tanto, los distractores deben ser verosmiles y ninguno de ellos debe sobresalir sobre los dems apareciendo como obviamente incorrecto. Una buena fuente de distractores son los errores comunes 1 y fallas en el razonamiento. Los distractores afectan directamente la dificultad de una pregunta. Veamos un ejemplo (a la derecha). Quin fue el principal autor de la Declaracin de la Independencia en los Estados Unidos de Amrica? A. Abraham Lincoln B. Thomas Jefferson C. Franklin Roosevelt D. El Rey Jorge II E. Catalina la Grande

En este ejemplo, las opciones son lo suficientemente divergentes como para identificar fcilmente a Thomas Jefferson como la respuesta correcta. En este caso, incluso alguien quien conozca slo superficialmente la historia de los Estados Unidos respondera a la pregunta correctamente. Ahora consideremos el mismo enunciado con otro conjunto de distractores. En este ejemplo, la pregunta se hace ms difcil, pues todas las opciones son verosmiles para alguien con un conocimiento limitado de la historia de los Estados Unidos. Para algunas materias es posible que el tipo de distractor del primer ejemplo pudiera ser el ms apropiado. En cambio, para otras materias el segundo ejemplo puede servirnos mejor. Quin fue el principal autor de la Declaracin de la Independencia en los Estados Unidos de Amrica? A. George Washington B. Thomas Jefferson C. Alexander Hamilton D. Benjamin Franklin E. James Madison

Al redactar los distractores, es importante recordar que entre sus caractersticas se encuentren el que sean: Homogneos en su contenido (por ejemplo: todos representan un diagnstico), todas son un paso siguiente a seguir en el manejo de un enfermo) Incorrectos, o por menos inferiores a la respuesta correcta Verosmiles y atractivos para los examinandos no bien informados Similares a la respuesta correcta en construccin y longitud Gramaticalmente consistentes y lgicamente compatibles con el enunciado

[ENG: misconceptions]

Captulo 4. Contenido de los temes: La evaluacin de la aplicacin del conocimiento de Ciencias Bsicas

41

La forma de un tem
Un tem bien configurado incluye la mayor cantidad posible del tem en el enunciado. El enunciado debera ser relativamente largo y las opciones relativamente cortas. El enunciado debe contener todos los datos pertinentes. No debe incluirse datos pertinentes adicionales en las opciones. Un tem con una forma correcta se vera as:

Enunciado Largo

A. B. C. Opciones cortas D. E. El siguiente tem presenta una forma deficiente:

Enunciado Corto
A. B. C. Opciones largas D. E.

42

Aprendizaje Basado en Problemas (ABP)1 y el uso de un grupo de preguntas en base a un caso2


Un nmero creciente de escuelas de medicina han adoptado el ABP como metodologa de enseanza en partes del currculo de ciencias bsicas. Aunque la manera de implementar el ABP es relativamente nica en cada escuela, todas stas usan casos escritos de pacientes (problemas) en la metodologa de enseanza de las ciencias bsicas. Los problemas se disean para estimular el aprendizaje del contenido de las ciencias bsicas tradicionales (por ejemplo: anatoma, fisiologa, bioqumica) desde una perspectiva clnica y enfatizan la aplicacin de conceptos de las ciencias bsicas a situaciones clnicas. Este material se logra cubrir por medio del estudio independiente por parte del estudiante y por medio de la discusin en pequeo grupo con un tutor docente. Los cursos y currculos que utilizan ABP enfatizan el proceso de aprendizaje, el cmo aprender, el hacer a los estudiantes responsables de su propio aprendizaje y el prepararlos para un aprendizaje durante toda su vida. Sin embargo existen importantes variaciones entre los programas que tienen consecuencias para la evaluacin. El enfoque mediante el Descubrimiento Abierto 3 enfatiza el proceso de aprendizaje: los estudiantes tienen la responsabilidad de determinar el contenido de lo que van a aprender, as como el cundo y cmo aprender. Se considera como lo ms importante aprender a aplicar principios amplios a situaciones de resolucin de problemas, con una mnima gua de parte de los tutores y una mxima oportunidad de exploracin por parte de los estudiantes. En contraste al Descubrimiento Abierto, el enfoque de Descubrimiento Guiado 4 desarrolla el currculo identificando los objetivos educacionales especficos para cada problema y stos son dados a los instructores que los utilizan para guiar los grupos de discusin y el aprendizaje por parte de los estudiantes. Estos currculos pueden estar considerablemente estructurados con una cuidadosa secuencia de actividades de aprendizaje. Los estudiantes pueden o no estar conscientes de esta estructura y de los objetivos especficos pero la experiencia puede ser muy similar al de los estudiantes de programas con un enfoque de Descubrimiento Abierto. En la prctica los dos enfoques podran considerarse como los dos extremos de un continuo y la totalidad de los programas que usan ABP podran considerarse como presentes en algn punto de este continuo. Los programas varan a lo largo del continuo y, dentro de cada programa, los problemas (y los grupos) tambin varan. La evaluacin en los programas que usan el mtodo de Descubrimiento Abierto se sita a menudo en variables de proceso tales como auto direccin en el estudio, motivacin, esfuerzo, resolucin de problemas y actitudes. La evaluacin de los resultados del aprendizaje es genuinamente difcil, porque en este enfoque se estimula a cada estudiante que siga su propia va de aprendizaje. Particularmente, la aplicacin de exmenes de seleccin mltiple tradicionales, a menudo es vista como inadecuada, porque estimulara a los estudiantes a que estudien para la prueba, desalentndolos en la eleccin de la materia a estudiar y en el proceso de cmo estudiarla. La evaluacin de los resultados del aprendizaje es ms fcil en el enfoque de Descubrimiento Guiado ya que los mismos objetivos que guiaron el desarrollo de los problemas y su uso, tambin puede ser usado para desarrollar una evaluacin. Para lograr una congruencia con las metas del currculo, la evaluacin debera enfocarse hacia la comprensin de los mecanismos bsicos de la salud, enfermedad y tratamiento. Las pruebas usando preguntas de seleccin mltiple bien construidas pueden jugar un papel importante en la evaluacin de ciencias bsicas, siempre que se enfoquen en evaluar la aplicacin del conocimiento de las
1 2

[ENG: PBL = problem-based learning] [ENG: case clusters] 3 [ENG: Open Discovery approach] 4 [ENG: Guided Discovery approach]

Captulo 4. Contenido de los temes: La evaluacin de la aplicacin del conocimiento de Ciencias Bsicas

43

ciencias bsicas al cuidado de pacientes. Las pruebas que usan grupos de preguntas en base a un caso; o sea, un conjunto de preguntas de seleccin mltiple que estn asociadas a un mismo paciente, son particularmente apropiadas para los cursos que usan ABP. Ms adelante se da un ejemplo de un grupo de preguntas en base a un caso. Esta consiste de una breve presentacin de un caso, seguida de tres preguntas de seleccin mltiple. Cada pregunta aborda un aspecto algo distinto del caso, considerando la situacin clnica desde varias perspectivas. Este material, al igual que el uso de ABP, enfatiza el aprendizaje de informacin de ciencias bsicas de manera que sea til en el cuidado de los pacientes. Una mujer de 34 aos presenta diarrea desde hace cuatro das. Dos meses antes padeci de mononucleosis infecciosa. Abusa de drogas por va intravenosa y presenta anticuerpos para HIV en la sangre. El examen fsico muestra deshidratacin y marcada debilidad muscular 1. Es ms probable que los estudios de laboratorio en el suero mostrarn A. K+ disminuido (*) B. Ca2+ disminuido C. HCO3- elevado D. Na+ elevado E. pH aumentado 3. Estudios posteriores para evaluar la infeccin por HIV muestran una relacin de linfocitos T helper a linfocitos T supresores de 0.3. Esto ocurre porque la infeccin por HIV: A. induce la proliferacin de linfocitos T helper B. induce la proliferacin de linfocitos T supresores C. infecta las clulas con receptores CD4 (*) D. infecta a los macrfagos E. estimula la sntesis de leucotrienos

2. Al evaluar la causa de la diarrea, cul de las siguientes opciones es la ms apropiada: A. Biopsia del colon para identificar Giardia lamblia B. Cultivo de la cavidad oral para identificar Candida albicans C. Biopsia duodenal para identificar Entamoeba histolytica D. Aspiracin gstrica para identificar Mycobacterium avium intracelular. E. Obtener una muestra de deposiciones para identificar Cryptosporidium (*)

44

Adems de los principios descritos anteriormente en este manual, hay dos consideraciones adicionales que se requieren al preparar un grupo de preguntas basadas en un caso: las pistas y el efecto de dependencia 1 . Primero, es deseable evitar dar pistas 2 sugerir respuestas a preguntas iniciales en las preguntas subsiguientes. Probablemente, los estudiantes se van a adelantar buscando estas pistas en las preguntas que siguen a continuacin. Por ejemplo, en un grupo de preguntas que describe un paciente con dolor torcico, si la primera pregunta aborda la causa ms probable de este dolor y la segunda pregunta requiere seleccionar el medicamento ms apropiado para su tratamiento, es importante que cada diagnstico en las opciones de la primera pregunta tenga un medicamento correspondiente a esa causa en la segunda (y viceversa). De lo contrario, los estudiantes que tienen habilidad para usar estas pistas pueden eliminar diagnsticos y medicamentos al comparar ambas listas de opciones. Segundo, es deseable evitar el efecto de dependencia 3 --crear preguntas en las que el estudiante debe saber la respuesta a una pregunta para poder responder otras preguntas a no ser que el tpico a ser evaluado sea tan importante que el redactor del tem est dispuesto a que los estudiantes reciban ya sea todos o ninguno de los puntajes asociados al grupo de preguntas. El grupo de preguntas que se presenta en la pgina siguiente, preparado por los Drs. David Felten y Ralph Josefowicz para el examen final de estudiantes de primer ao del curso de Neurociencias en la Universidad de Rochester, ilustra una estrategia para evitar el efecto de dependencia. Cada uno de los tres temes est enfocado a un aspecto diferente del paciente y es probable que los estudiantes respondan correctamente a unos e incorrectamente a otros, por lo que recibirn un crdito parcial por conocimientos parciales. La ltima pregunta presenta un efecto de dependencia leve de los temes precedentes ya que requiere que los estudiantes integren la informacin que contienen para poder responder correctamente. Pero esto parece razonable dada la importancia de este proceso. Puede ser difcil para un docente trabajando solo preparar grupos de preguntas que requieran de informacin proveniente de diversas disciplinas de ciencias bsicas. Esto necesitara de un extenso conocimiento en varias disciplinas. Una estrategia para enfrentar este desafo es trabajar en equipo, usando un mtodo similar al que se usa para preparar material de problemas en los cursos que usan ABP. Por ejemplo, un clnico puede preparar la descripcin del paciente al comienzo del grupo de preguntas y agregar ms adelante las preguntas que incluyan fisiopatologa. Los docentes de las disciplinas de ciencias bsicas afines al caso pueden contribuir con preguntas que aborden varios aspectos del paciente desde la perspectiva de sus respectivas disciplinas. Este tipo de preguntas puede utilizarse no solamente en cursos y currculos de ABP. Es completamente apropiado usarlo en situaciones que acentan la aplicacin clnica de las ciencias bsicas en el proceso de enseanza-aprendizaje y en la evaluacin. En nuestra opinin, esto incluye la mayora de los cursos en ciencias bsicas, an aquellos que se dictan en el primer ao de la carrera. Como lo demuestra el ejemplo de Neurociencias en la pgina siguiente, es posible evaluar conocimientos de anatoma y fisiologa en el contexto de un paciente an en cursos dictados en forma tradicional.

1 2

[ENG: cueing and hinging] [ENG: cueing] 3 [ENG: hinging]

Captulo 4. Contenido de los temes: La evaluacin de la aplicacin del conocimiento de Ciencias Bsicas

45

Una mujer es trada en estado inconsciente a un servicio de urgencia despus que perdi el conocimiento mientras realizaba compras en un centro comercial cercano. La familia informa que la paciente se senta bien esa maana pero desarroll una cefalea progresiva mientras realizaba compras. Ha tenido hipertensin y fibrilacin auricular y toma un medicamento antihipertensivo y un anticoagulante oral. La presin arterial es de 220/130 mmHg y la respiracin alterna entre apnea e hiperpnea. Responde a estmulos dolorosos con respuesta extensora en el brazo y pierna derechos. El examen de fondo de ojo revela edema de papila en el ojo izquierdo. La pupila derecha mide 3 mm de dimetro, mientras que la izquierda mide 7 mm y no responde a la estimulacin de luz. Su mirada tiende a irse hacia la izquierda. Hay hiperreflexia difusa, mayor al lado derecho. Presenta Babinski bilateral. 1. Qu dao estructural explica la pupila izquierda dilatada y sin respuesta a la luz? A. nervio ptico B. tracto ptico C. nervio oculomotor (*) D. ncleo lateral geniculado E. pedculo 1 superior 2. El dao de cul de las siguientes estructuras explica la postura en extensin del lado derecho del cuerpo? A. telencfalo B. diencfalo C. mesencfalo (*) D. puente E. mdula 3. El tipo de respiracin se puede describir como: A. Normal B. Cheyne-Stokes (*) C. Hiperventilacin central neurognica D. Apnustica 2

4. Cul de los siguientes sndromes de herniacin es ms consistente con el cuadro clnico? A. Cingulate gyrus bajo el falx cerebelli B. Uncus del lbulo temporal a travs del tentorium cerebelli (*) C. Diencfalo a travs de la incisura tentorial D. Tronco encfalo a travs de la incisura tentorial E. Amgdalas cerebelosas a travs del foramen magnum

Una discusin adicional de la evaluacin en cursos y currculos de ABP se halla en:


Swanson DB, Case SM, and van der Vleuten CM. Strategies for student assessment, In: Boud, Feletti, eds. The challenge of Problem-Based Learning Second Edition. London. Kogan Pahe Ltd, 1997:269-282.

1 2

[ENG: colliculus] [ENG: Apneustic]

46

Ejemplos de temes para las Ciencias Bsicas


1. Varias clulas contiguas son marcadas con una tincin fluorescente que no cruza la membrana celular. En forma experimental, una de las clulas es blanqueada usando una luz que destruye la tincin, sin embargo la fluorescencia se recobra pronto. Esta recuperacin se explica mejor por la presencia de cul o cules de la(s) siguiente(s) estructura(s) entre la clula blanqueada y sus vecinos fluorescentes? A. Lmina basal B. Desmosomas (maculae adherentes) C. Uniones de brecha 1 (*) D. Glicosaminoglicanos E. Uniones estrechas 2 (zonulae occludentes) 3. Durante una operacin quirrgica se monitorea la PCO2 y el pH de un paciente anestesiado. Se le efecta ventilacin mecnica y los valores iniciales son normales (PCO2 = 40 mmHg, pH = 7.42). Si la ventilacin decreciera, cul de las siguientes opciones es la ms probable que ocurra? PCO2 arterial A. B. C. D. E. F. disminuido disminuido disminuido elevado elevado elevado pH disminuido elevado sin cambio disminuido (*) elevado sin cambio

2. Un hombre de 30 aos de edad presenta prdida de sensacin al dolor y la temperatura desde el cuello hacia abajo, en el lado derecho del cuerpo y en el lado izquierdo de la cara, adems de una parlisis parcial del paladar blando, laringe y faringe en el lado izquierdo y ataxia en el lado izquierdo. Este sndrome se produce probablemente como consecuencia de una trombosis en cul de las siguientes arterias? A. Basilar B. Cerebelosa pstero inferior derecha C. Cerebelosa pstero inferior izquierda (*) D. Cerebelosa superior derecha E. Cerebelosa superior izquierda.

4.

En esta va metablica ramificada, cada paso es catalizado por una enzima diferente. Cul de las enzimas va a ser ms inhibida por el compuesto V? A. B. C. D. E. A B (*) C D E

1 2

Gap juntions Tight junctions

Captulo 4. Contenido de los temes: La evaluacin de la aplicacin del conocimiento de Ciencias Bsicas

47

5. Un paciente con cirrosis posthepatitis desarrolla rpidamente una hepatomegalia acompaada de deterioro en la funcin heptica. Cul de los siguientes componentes del suero es ms probable que est alterado? A. 1-Antitripsina B. Antgeno carcinoembrionario C. Gonadotrofina corinica D. -Fetoprotena (*) E. Gastrina

7. Los exmenes de laboratorio de un hombre de 35 aos de edad que presenta edema de extremidades inferiores, muestran una concentracin srica normal de complemento y un aumento de la cifra de colesterol. En el examen de orina se encuentra: protena 4+, 0-5 eritrocitos por campo, y varios cilindros hialinos. La biopsia renal es ms probable que muestre: A. Glomerulonefritis aguda postestreptoccica (proliferativa) B. Glomerulonefritis membranoproliferativa C. Glomerulonefritis membranosa (*) D. Enfermedad de cambios mnimos (nefrosis lipoidea) E. Glomerulonefritis rpidamente progresiva

6. El primer recin nacido de una madre de 26 aos, Rh-negativa que ha tenido dos abortos previos en el segundo trimestre, nace con hemlisis severa e insuficiencia circulatoria. Esta situacin se pudo haber prevenido tratando a la madre con: A. IgG anti-RhD durante este ltimo embarazo B. IgG anti-Rh inmediatamente despus de terminar los dos embarazos anteriores (*) C. IgM anti-Rh durante este ltimo embarazo D. IgM anti-Rh inmediatamente despus de terminar el primer embarazo

8. Los genes de un cromosoma bacteriano tienen las siguientes uniones en la transferencia conjugada: x e y, el 25% del tiempo; y y z, 50% del tiempo. Si el orden de los genes es x-y-z, aproximadamente qu porcentaje del tiempo x y z sern transferidos juntos? A. 1% del tiempo B. 5% del tiempo C. 13% del tiempo (*) D. 20% del tiempo E. 40% del tiempo

48

9.

El men en un banquete incluye: pollo frito, 11. Un paciente que es visto en el servicio de papas fritas, arvejas, eclairs de chocolate y caf. urgencia no recuerda qu clase de medicamento En menos de dos horas, todos los comensales para el corazn est tomando. Su frecuencia sufren un severo episodio caracterizado por cardaca es de 80/min y los intervalos PR y QRS nuseas, vmitos, y dolor abdominal. Al estn prolongados en el ECG. Adems el analizar la comida contaminada Cul paciente se queja de pitos en los odos. Cul de organismo es el ms probable que se encuentre los siguientes medicamentos es el ms probable en grandes cantidades? que est tomando? A. B. C. D. E. Escherichia coli Proteus mirabilis Salmonella typhimurium Staphylococcus aureus (*) Streptococcus faecalis A. B. C. D. E. Digoxina Lidocana Fenitona Propranolol Quinidina (*)

10. El medicamento Y tienen un volumen de 12. Un nio de 8 aos necesita que se le convenza distribucin (Vd) de 75 L tanto en hombres para ir a la escuela y, a menudo, mientras est en jvenes como ms viejos. En los jvenes, el ella, se queja de cefalea y dolor abdominal clearance es de 15 L/h, del cual el 50% lo hace severos. En ocasiones, su madre debe ir a va heptica y 50% renal. Para los jvenes la buscarlo para llevarlo a la casa debido a sus dosis de mantenimiento es de 100 mg cada 6 sntomas. Durante las noches trata de dormir en horas. Cul de las siguientes indicaciones la cama de sus padres. Cuando ellos le insisten producir esencialmente el mismo nivel en que duerma en su propia cama, l menciona plasmtico estable 1 en un hombre de mayor que tiene monstruos en el closet de su pieza. edad, en el cual el clearance de creatinina est Con cul diagnstico son compatibles estos reducido a la mitad del joven y que posee una hallazgos? funcin heptica normal? A. 75 mg cada 3 horas A. Esquizofrenia infantil B. 75 mg cada 6 horas (*) B. Aspectos normales del perodo escolar C. 75 mg cada 9 horas C. Angustia de separacin (*) D. 100 mg cada 3 horas D. Trastorno de la conducta social 2 E. Psicosis simbitica E. 100 mg cada 6 horas F. 100 mg cada 12 horas

[ENG: steady state concentration] [ENG: Socialized conduct disorder]

Captulo 4. Contenido de los temes: La evaluacin de la aplicacin del conocimiento de Ciencias Bsicas

49

50

Captulo 5 Contenido de las preguntas: Evaluacin de la aplicacin del conocimiento de Ciencias Clnicas
Mtodos de evaluacin
A pesar del debate que se realiza en forma continua en cuanto a cun apropiado es el uso de preguntas de seleccin mltiple, stas son usadas en las tres secciones del examen nacional para autorizar el ejercer la medicina (USMLE) en Estados Unidos de Norteamrica. En su deseo de producir mejores instrumentos de evaluacin, la entidad que desarrolla estos exmenes nacionales (NBME) ha llevado a cabo investigaciones acerca de otras modalidades de preguntas. As, por ms de 25 aos, se ha enfocado al proyecto denominado Examen Basado en Computadores o CBX 1 y que luego se conoci como Simulaciones de Casos Basados en Computadores o CCS 2 que se introdujeron en el ao 1999 como un componente de la tercera seccin del examen USMLE. Desde mediados de la dcada del 70, una segunda rea de investigacin se concentr en los pacientes estandarizados 3 . Tal como sucede en otras formas de evaluacin autntica 4 , los CCS y los exmenes usando pacientes estandarizados parecen tener importantes ventajas en la evaluacin de aspectos de la competencia clnica porque presentan al examinando tareas de una manera parecida a la realidad. Otros proyectos se han enfocado a mejorar el formato de preguntas de seleccin mltiple. Como resultado de las investigaciones realizadas en este campo, las preguntas de seleccin mltiple se ven muy diferentes de aquellas que se usaban tradicionalmente en el pasado. Tanto por motivos psicomtricos como por razones de contenido, los formatos de preguntas de verdadero/falso, tales como los de tipo K (mltiples verdadero/falso) y las del tipo C (A, B, ambas, ninguna) ya no se usan en los exmenes para demostrar capacidad para ejercer la profesin 5 . Aunque la mayora de las preguntas usadas en la seccin 2 del USMLE de los exmenes nacionales de este tipo contienen las tradicionales 5 opciones, tanto las preguntas de tipo A y las de emparejamiento ampliado (ver captulo 6) pueden incluir hasta 26 opciones, lo que coloca al examinando en una situacin parecida a aqulla en que debiera contestar preguntas de respuesta libre 6 . Prcticamente todas las preguntas de la Seccin 2 (USMLE Step 2) presentan una vieta clnica que representa una tarea relevante para un interno recin iniciado, tal como determinar un diagnstico o el siguiente paso a seguir en la atencin de un paciente. Estas preguntas requieren interpretacin y sntesis de la informacin suministrada. Tambin requiere de la aplicacin de sus conocimientos a situaciones conocidas o desconocidas de acuerdo con la experiencia del examinando. Al igual que en CCS y los casos basados en SP, las preguntas de seleccin mltiple enmarcadas en vietas breves de pacientes, presentan a los examinandos, problemas de toma de decisiones mdicas en simulaciones de baja fidelidad 7 .

1 2

[ENG: Computer Based Examination or CBX] [ENG: Computer-based Case Simulations or CCS] 3 [ENG: Standardized Patients o SPs] 4 [ENG: authentic assessment] 5 [ENG: licensure] 6 [ENG: uncued free-response] 7 [ENG: low-fidelity simulations of medical decision-making problems]

Captulo 5. Contenido de las preguntas: Evaluacin de la aplicacin del conocimiento de Ciencias Clnicas

51

Asuntos generales acerca de qu evaluar


Existen varios conflictos 1 que influyen en la construccin de cada Seccin del USMLE y que pueden ser relevantes para usted cuando decide qu incluir en sus pruebas. Las diversas secciones del USMLE estn diseadas para su propsito de ser un examen de autorizacin para practicar la medicina 2 . As, la Seccin 2 de este examen est diseada para evaluar la aplicacin de conocimientos mdicos que se requieren para iniciar un perodo de residencia despus de terminar los estudios mdicos de pregrado y sin importar la especialidad que elija el examinando. El enfocar el examen en un contenido que es necesario para practicar la medicina (si fuera posible definir tal cantidad de informacin) ms que en el contenido de lo que se ense, significa que el examen puede incluir preguntas que evalan conocimientos que no han sido enseados de manera uniforme en las escuelas de medicina. A la inversa, habr tpicos que pueden haberse enseados en las escuelas pero que son omitidos en los exmenes. En la realidad, se produce una correspondencia estrecha entre el contenido enseado y el que se incluye en este tipo de exmenes, pero el examen est diseado para ser una evaluacin independiente acerca del contenido que los estudiantes deben dominar antes de comenzar sus estudios de postgrado, sin importar si este contenido fue enseado o no. Una situacin anloga se produce en las escuelas de medicina que deben decidir si es apropiado incluir contenidos en los exmenes que no fueron enseados en forma explcita. Por ejemplo, se debe decidir hasta qu punto se debe asignar a los estudiantes la responsabilidad de un estudio independiente ms all de la materia cubierta en clase. Un segundo conflicto en la seccin 2 del USMLE resulta de la necesidad de enfocar el examen en los conocimientos necesarios que requiere un mdico general no diferenciado. Cuando se critica este nfasis en un examen general en una poca en que predomina la especializacin, hemos respondido que la licencia para practicar medicina es una licencia general y no una licencia de especialidad. Esto nos hace creer que es necesario producir un examen que tenga caractersticas generales. La tendencia de la medicina en Estados Unidos a fomentar la produccin de mdicos de caractersticas generales nos hace aparecer casi como profticos algo de lo que no se acusa a menudo al NBME. Una situacin anloga que ocurre en escuelas de medicina es hasta qu grado las pasantas clnicas 3 debieran enfocarse solamente en su disciplina, sin realizar un esfuerzo de integracin de varias disciplinas. Adems, los profesores deben decidir hasta qu punto las pasantas clnicas debieran servir como una experiencia de pre internado para los estudiantes o una en que se provee una visin general de temas relevantes para todos los mdicos sin importar su especialidad. El enfocarse en preguntas que son apropiadas para todos los estudiantes en vas de graduarse, en lugar de apuntar a estudiantes que seguirn determinada especialidad, ha tenido una influencia considerable en el contenido de las preguntas de la seccin 2 del USMLE. As, por ejemplo, el comit que redacta las preguntas de ciruga preferir incluir preguntas acerca del manejo de pacientes en situaciones en que se necesita una intervencin inmediata o donde importa realizar un triage y derivar a un paciente hasta el servicio de ciruga en vez de darlo de alta del servicio de urgencia, en lugar de redactar preguntas relacionadas a tcnicas quirrgicas que no se consideran un conocimiento esencial para los que no son cirujanos. El comit encargado de las preguntas de obstetricia y ginecologa probablemente no redactar preguntas acerca del manejo de casos poco comunes en embarazos de alto riesgo (aun cuando la mayora de los estudiantes hayan sido expuesto a este tipo de casos en una pasanta clnica), y se enfocar en el diagnstico del embarazo ectpico algo que todo nuevo interno, sin importar la especialidad ulterior, debiera reconocer. En preguntas de psiquiatra es poco probable que aparezcan preguntas acerca de un paciente psiquitrico hospitalizado, pero ser ms corriente que se encuentren preguntas que evalen la habilidad para diferenciar entre
1 2

[ENG: tensions] [ENG: examination for general licensure] 3 [ENG: clerkships]

52

enfermedades psiquitricas y no psiquitricas en pacientes que presentan un comportamiento extrao. Las preguntas de la Seccin 2 del USMLE deben ser aceptadas por la totalidad de los miembros del comit relacionado a una disciplina especfica, y adems deben ser aceptadas por los miembros del comit interdisciplinario. Esta secuencia en que la revisin por un comit de disciplina especfica es seguido por la revisin del comit interdisciplinario ayuda a lograr un equilibrio en el contenido en cada examen. De igual forma, en su propia escuela sera importante que sus preguntas de un examen fueran revisadas por otros miembros de su departamento, aunque la logstica de llevar a cabo este procedimiento podra ser difcil de realizar.

Examinar el recuerdo de datos aislados 1 o examinar aplicacin del conocimiento


Adems de pensar acerca de los tpicos que son importantes para incluir en un examen, usted debiera pensar en cmo estructurar las preguntas para evaluar ms all del recuerdo de datos aislados. Tradicionalmente, las preguntas en un examen se han clasificado como requiriendo recuerdo, interpretacin, o resolucin de problemas (memoria, comprensin y razonamiento), dependiendo de los procesos cognitivos que se requieren para responder la pregunta. Tpicamente, son preguntas de memoria 2 aquellas que evalan el conocimiento de definiciones y datos aislados. Las preguntas de interpretacin requieren que los examinandos evalen cierta informacin, a menudo en forma tabular o grfica, para llegar a una conclusin (ejemplo, un diagnstico). Las preguntas de resolucin de problemas 3 presentan una situacin y requieren que los examinandos tomen una accin (por ejemplo, el siguiente paso en el manejo de un paciente). El inconveniente de estas clasificaciones es que los procesos cognitivos que se requieren para responder una pregunta dependen tanto de los antecedentes del examinando 4 como del contenido de la pregunta. Los expertos en un rea del conocimiento podran simplemente recordar una respuesta correcta sin mucho esfuerzo consciente, mientras otros tendrn que razonar la respuesta a partir de principios bsicos. Los procesos cognitivos implicados en la respuesta a una pregunta son especficos para cada examinando, lo que hace difcil abordar las preguntas con un enfoque taxonmico. Un enfoque alternativo divide a las preguntas en dos categoras: a) aplicacin del conocimiento y b) recuerdo de datos aislados. Si una pregunta requiere que el examinando llegue a una conclusin, haga una prediccin, o seleccione una accin a seguir, entonces la pregunta se clasifica como de aplicacin del conocimiento. Si una pregunta solamente evala memoria de datos aislados (sin requerir una aplicacin), se clasifica como una pregunta de recuerdo. Los temes que requieren recuerdo de datos aislados a menudo comienzan por citar una Cul de los siguientes hallazgos es ms probable que enfermedad y luego preguntan cules aparezca durante la evolucin postoperatoria de hallazgos clnicos se espera encontrar. Estas pacientes que presentan una embolia pulmonar? preguntas se estructuran en forma similar a los textos de estudio; el examinando podra buscar el prrafo correspondiente en un libro y responder a la pregunta. El defecto de estas preguntas es que aparecen como clnicamente al revs de lo que sucede en la vida real. Los pacientes raramente les comunican a sus mdicos el diagnstico de la enfermedad que padecen y luego le preguntan al mdico cules son sus sntomas y hallazgos del examen fsico.

1 2

[ENG: Testing recall of isolated facts] [ENG: Recall questions] 3 [ENG: Problem-Solving Questions] 4 [ENG: background of the examinee]

Captulo 5. Contenido de las preguntas: Evaluacin de la aplicacin del conocimiento de Ciencias Clnicas

53

Otro tipo de tem que plantea una tarea inapropiada es conocido como un tem de sala de espera 1 . En este tipo de pregunta, se le pide al examinando que seleccione entre cinco pacientes cul es el ms apropiado para realizar un cariotipo fetal. Slo tiene que mirar en la sala de espera y seleccionar el ms apropiado.

Cul de las siguientes constituye una indicacin para estudiar el cariotipo fetal en una mujer de 28 aos de edad? A. Edad del padre: 55 aos B. Higroma qustico fetal en el examen con ultrasonido C. Diagnstico de espina bfida en el hijo mayor D. Aborto previo de un feto con triploida E. Trisoma 21 en un hermano de la mujer

En cambio, este tem describe una paciente y pregunta cul estudio es el ms apropiado, lo que es una tarea ms razonable.

Una profesora de 28 aos de edad, en buenas condiciones de salud, tiene un embarazo de 11 semanas. La historia familiar muestra que dos de sus hermanos padecen un retardo mental severo, su madre falleci a los 55 aos de cncer mamario, y el padre vive separado de la familia. No existen registros con datos acerca de la salud de la familia Cul de los siguientes procedimientos es el ms apropiado? A. Examen en sangre para la deteccin del estado de portador de X frgil B. Examen en sangre para la deteccin del estado de portador de fenilcetonuria C. Muestra de vellosidades corinicas para la deteccin de distrofia muscular de Duchenne D. Muestra de vellosidades corinicas para realizar anlisis cromosmico. E. Amniocentesis para determinar la -fetoprotena

Las preguntas que evalan aplicacin del conocimiento se construyen de una manera clnicamente ms realista al presentar los hallazgos clnicos y luego preguntar al examinando que indique la enfermedad subyacente responsable de esos hallazgos. Tpicamente, los examinandos deberan ser capaces de sintetizar la informacin correspondiente a varias pginas de un texto de estudio para poder responder correctamente este tipo de preguntas.

Un hombre de 62 aos de edad desarrolla disnea aguda y dolor torcico de tipo pleurtico 4 horas despus de una colecistectoma. Cul es el diagnstico ms probable?

[ENG: waiting room item]

54

El uso de preguntas con vietas clnicas en los enunciados presenta varios beneficios. Primero, la validez de apariencia 1 del examen mejora notablemente al requerir que el examinando resuelva problemas clnicos. Segundo, es ms probable que las preguntas se enfoquen en informacin importante en lugar de asuntos triviales. Tercero, estas preguntas sirven para identificar aquellos examinandos que han memorizado una cantidad importante de datos, pero son incapaces de usar esa informacin en forma efectiva en situaciones clnicas. Las preguntas con vietas clnicas son generalmente ms apropiadas para exmenes de niveles ms superiores 2 . Redactar preguntas de aplicacin del conocimiento es relativamente sencillo en medicina. Cuando usted describe a un paciente y hace una pregunta relacionada al paciente, usted est evaluando aplicacin del conocimiento. La nica ocasin en que el uso de una vieta clnica resulta en una pregunta de recuerdo de datos aislados es si la vieta describe un paciente idntico al que el examinando haya tenido conocimiento con anterioridad (por ejemplo, al usar una vieta clnica de un paciente presente en un libro o que se discuti en clase).

Pautas 3 para guiar el contenido de los temes de ciencias clnicas


Evale la aplicacin del conocimiento mediante el uso de vietas clnicas que planteen situaciones de manejo de pacientes en que hay que tomar una decisin mdica. Enfatice las preguntas orientadas a problemas comunes o potencialmente catastrficos o crticos. Evite contenidos o situaciones esotricas o raras 4 . Plantee tareas de decisiones clnicas que se espera debiera saber realizar el examinando. Evite presentar situaciones clnicas que son habitualmente responsabilidad del especialista. Las preguntas deben enfocarse a tareas especficas que un examinando exitoso debiera saber manejar al avanzar a la etapa siguiente de su educacin (por ejemplo, determinar el diagnstico ms probable, indicar los estudios adicionales de laboratorio que debiera solicitar; formular el siguiente paso en el manejo de un paciente; predecir el hallazgo ms probable). En cada tpico, el foco de la pregunta debiera estar en aquellas reas en las que comnmente se cometen errores

1 2

[ENG: face validity] [ENG: higher level examinations] 3 [ENG: guidelines] 4 [ENG: zebras]

Captulo 5. Contenido de las preguntas: Evaluacin de la aplicacin del conocimiento de Ciencias Clnicas

55

Redaccin de preguntas de una mejor respuesta


Construccin del enunciado 1 La gran mayora de las preguntas debieran construirse con una vieta clnica. El enunciado debiera comenzar con un problema que presenta un paciente, seguido de la historia clnica (incluyendo duracin de los sntomas y signos), hallazgos del examen fsico, resultados de estudios diagnsticos, tratamiento inicial, hallazgos posteriores, etc. Las vietas pueden incluir solamente una parte 2 de esta informacin, pero la informacin debiera presentarse en este orden particular. El enunciado debiera consistir de un solo problema, claramente formulado. La pregunta introductoria 3 del enunciado debiera consistir de una pregunta redactada en forma clara de manera que al examinando le sea posible responder sin ver las opciones. Satisfacer la regla de cubrir las opciones es un componente esencial de una buena pregunta. Buen enunciado: Este enunciado provee de suficiente informacin y puede responderse sin referirse a las opciones. Un hombre de 52 aos de edad presenta disnea progresiva y tos productiva con expectoracin purulenta en los dos ltimos das. Ha sido fumador de dos cajetillas de cigarrillos diarios durante los ltimos 30 aos. La temperatura axilar es de 37.2 C (99F). Los ruidos respiratorios son apenas audibles a la auscultacin (distantes) junto a roncus y estertores aislados 4 . El recuento leucocitario es de 9 000/mm3, con recuento diferencial normal. La tincin Gram del esputo muestra numerosos neutrfilos y diplococos gram negativos. La radiografa de trax muestra hiperinsuflacin pulmonar. Cul de los siguientes es el diagnstico ms probable?

Enunciado que pregunta por hechos aislados. El siguiente enunciado contiene informacin insuficiente. Para responder la pregunta el examinando debe usar las opciones como marco de referencia. Cul de las siguientes opciones es verdadera en relacin a la seudogota?

Las vietas de pacientes debieran incluir todos o algunos de los siguientes componentes, en el orden indicado: Edad y sexo (ejemplo: un paciente de sexo masculino de 45 aos de edad) Lugar en que se realiza el acto mdico: (ejemplo: se presenta en la sala de urgencias). Motivo de consulta (ejemplo: por una cefalea). Duracin (ejemplo: de dos das de duracin) Historia, incluyendo Historia Familiar si es necesario Hallazgos del examen fsico +/- Resultados de los estudios diagnsticos +/- Tratamiento inicial, hallazgos posteriores.

1 2

[ENG: stem] [ENG: subset] 3 [ENG: lead-in] 4 [ENG: rhonchi and wheezes]

56

Asegrese que el enunciado: Est enfocado a conceptos importantes ms que datos triviales Puede ser respondido sin ver las opciones Incluye todos los datos pertinentes. No debe proporcionarse informacin adicional en las opciones. No presenta triquiuelas 1 o demasiada complejidad. No est expresado en forma negativa (evitar usar EXCEPTO o NO) en la pregunta introductoria.

Detalles acerca de los enunciados de las preguntas


Uso de pacientes reales. Creemos que es mejor no usar pacientes reales como punto de partida para las preguntas de seleccin mltiple, especialmente en los exmenes para estudiantes. Como regla general los pacientes reales son demasiado complejos, y los elementos que los hacen complejos no son necesariamente aqullos que los hacen importantes para usar en evaluacin. Tal como mencionamos anteriormente, incluimos informacin secundaria 2 pero no incluimos pistas falsas 3 (o sea, informacin que tiene por objeto desviar la atencin de los examinandos de la respuesta correcta). Desafortunadamente, los pacientes reales a menudo presentan pistas falsas entre sus hallazgos. Uso de materiales de referencia. Creemos que es apropiado proporcionar informacin en una pregunta, si en la vida real es probable que alguien consulte una referencia para obtener la informacin. Por ejemplo, en muchos casos, creemos que es apropiado dar una tabla con valores normales de laboratorio o un calendario de inmunizaciones o de chequeos mdicos. Por supuesto que no es necesario que uno formule solamente preguntas que requieran que el examinando consulte la informacin de la tabla. Por ejemplo, se podra preguntar acerca de la inmunizacin de un nio de 6 aos de edad que no ha recibido inmunizaciones previas. Uso de las palabras del mdico o del paciente. En general no creemos conveniente incluir las propias palabras de un paciente, en especial si la tarea a realizar por el examinando consiste en interpretar matices de lenguaje que podran verse afectados por el tono de voz. Por otro lado, podra ser til preguntar al examinando que escoja la respuesta ms apropiada por parte del mdico hacia un paciente, haciendo que el examinando escoja entre opciones redactadas como preguntas abiertas 4 , cerradas 5 , o preguntas que sugieren la respuesta que se desea obtener 6 . Pacientes que mienten. Creemos que todos los pacientes representados en preguntas de seleccin mltiple deben decir la verdad, o en todo caso proveer la interpretacin por parte del mdico de la historia clnica. Los mdicos utilizan mltiples pistas para determinar cun veraz es un paciente y no es posible poner por escrito muchas de estas pistas. As, las preguntas describiran el consumo de alcohol de un paciente como, El paciente bebe 16 onzas (aproximadamente 500 ml) de cerveza cada noche con su comida o La descripcin de su consumo de alcohol por el paciente es contradictoria. No es recomendable escribir en forma ambigua algo como El paciente asegura 7 que bebe slo una botella de cerveza cada noche.

1 2

[ENG: is not tricky] [ENG: incidental findings] 3 [ENG: red herrings] 4 [ENG: open-ended] 5 [ENG: closed] 6 [ENG: leading] 7 [ENG: claims]

Captulo 5. Contenido de las preguntas: Evaluacin de la aplicacin del conocimiento de Ciencias Clnicas

57

La verborrea, la informacin innecesaria y las pistas falsas1 : Mejoran las preguntas de un examen?
La mayora de los educadores subraya la necesidad de redactar preguntas cuyo enunciado sea lo ms breve posible, evitando la verborrea (esto es, incluir palabras extras), informacin innecesaria 2 (incluir material extrao), y pistas falsas (incluir informacin diseada para despistar al examinando). Sin embargo, en oposicin a este consejo, recomendamos el uso de vietas clnicas en la redaccin de preguntas. En el caso de la seccin 2 del USMLE 3 estas vietas consisten en un prrafo con una descripcin clnica de una situacin problema, habitualmente seguida de una pregunta relacionada al diagnstico o el paso siguiente a seguir en el cuidado de un paciente. Estos temes hacen hincapi en la aplicacin del conocimiento al requerir de los examinandos que tomen decisiones clnicas en lugar de simplemente recordar datos aislados. Son diseados para reflejar tareas de la vida real desafiando a los examinandos que primero identifiquen los hallazgos importantes y luego integren esos hallazgos en un diagnstico o en una accin clnica. Estos temes a menudo requieren de mltiples pasos en el proceso mental que lleva a su resolucin. Hemos visto que los temes con vietas tienden a tener menos defectos tcnicos que los tpicos temes sin vietas, presumiblemente porque los temes de vietas siguen una estructura estandarizada y presentan preguntas que son clnicamente naturales. A pesar de estas ventajas, algunos han cuestionado el uso de vietas, al pensar que una vieta hace que un tem se vuelva ms engorroso cuando se envuelve la verdadera pregunta en datos adicionales. Algunos recomiendan evitar el uso de vietas en forma absoluta, otros prefieren usar vietas cortas y expresar concisamente slo datos pertinentes y el resto recomienda usar vietas largas que incluyan informacin ms completa que el examinando debe considerar y sintetizar. Se realizaron varias investigaciones 4 para comparar las caractersticas psicomtricas de temes desarrollados en tres formatos: sin vieta, con vieta corta, y con vieta larga. Se dise esta progresin para requerir niveles crecientes de interpretacin, anlisis, y sntesis de los hallazgos (ver ms adelante ejemplos de estos tres formatos). Tal como se esperaba, los temes se volvan ms difciles al presentar los hallazgos del paciente en una forma menos elaborada, sin embargo las diferencias en discriminacin no fueron estadsticamente significativas. A pesar de estos hallazgos no concluyentes 5 creemos que las vietas son generalmente ms apropiadas porque examinan aplicacin de conocimientos a situaciones de pacientes y plantean desafos clnicos apropiados. Se podra considerar a estos temes como simulaciones clnicas de baja fidelidad que mejoran la validez de contenido 6 de los exmenes. Un tem redactado sin vieta, tpicamente se crea desde una perspectiva de arriba hacia abajo 7 (a saber, dada una enfermedad, sealar los hallazgos asociados a ella). Para un experto, los temes redactados de esta manera pueden aparecer idnticos a los temes redactados con una vieta correspondiente a un paciente.

1 2

[ENG: Verbosity, Window dressing, and Red Herrings] [ENG: window dressing] 3 Segunda fase del examen nacional para autorizar el ejercicio de la medicina en USA 4 De: Case SM, Swanson DB, Becker DF. Verbosity, window dressing, and red herrings: do they make a better test item? Academic Medicine. 1996,71:528-530. 5 [ENG: mixed psychometric results] 6 [ENG: content validity] 7 [ENG: top-down]

58

El siguiente tro de preguntas indica que los temes no son idnticos para el estudiante de bajo rendimiento 1 . Cada tem fue administrado a estudiantes de medicina del ltimo ao de la carrera como parte del examen para autorizar a ejercer la profesin2 . La tabla junto a cada pregunta muestra el porcentaje de estudiantes de alto 3 y bajo 4 rendimiento (definidos como el 20% superior de estudiantes y el 20% inferior de los resultados del examen) que eligieron cada opcin. En el tem sin vieta, prcticamente todos los estudiantes del grupo alto (99%) y el 90% del grupo bajo eligieron la respuesta correcta. En el caso de las preguntas con vietas cortas o largas, los resultados mostraron que no eran particularmente ms difcil para el grupo alto pero s lo eran para el grupo bajo. La respuesta correcta en este ltimo grupo fue seleccionada por el 82% en las preguntas de vieta corta y por el 66% en las preguntas de vieta larga.

Sin vieta A Alto 1 B 0 1 C 99 90 D 0 1 E 0 0

La enfermedad renal ms probable en pacientes peditricos con sndrome nefrtico y funcin renal normal es: A. Glomerulonefritis aguda postestreptoccica B. Sndrome hemoltico-urmico *C. Sndrome nefrtico de cambios mnimos 5 D. Glomeruloesclerosis focal y segmentaria E. Prpura de Schnlein-Henoch con nefritis

Bajo 8

Vieta corta A Alto 0 B 0 2 C 98 82 D 2 8 E 0 1

Un nio de dos aos de edad presenta edema desde hace una semana. Su presin arterial es 100/60 mmHg. El edema es generalizado con presencia de ascitis. Las concentraciones sricas son las siguientes: creatinina 0.4 mg/dL, albmina 1.4 g/dL, y colesterol 569 mg/dL. Hay 4+ de protena en la orina, sin presencia de eritrocitos. El diagnstico ms probable es (sigue la misma lista de opciones de la pregunta anterior).

Bajo 5

1 2

[ENG: low performing] Segunda de tres secciones del examen nacional de medicina en los Estados Unidos. 3 [ENG: Hi] 4 [ENG: Lo] 5 [ENG: Minimal change nephrotic syndrome]

Captulo 5. Contenido de las preguntas: Evaluacin de la aplicacin del conocimiento de Ciencias Clnicas

59

Vieta larga A Alto 0 B 1 9 C 98 66 D 1 10 E 0 5

Un nio de dos aos de edad, de raza negra, desde hace una semana presenta edema de prpados y tobillos. La presin arterial es 100/60 mmHg, el pulso 110/min, con 29 respiraciones por minuto. Adems del edema palpebral, y un edema 2+ de los tobillos, presenta un Las abdomen distendido con signo de la ola positivo 1 . concentraciones sricas son las siguientes: creatinina 0.4 mg/dL, albmina 1.4 g/dL, y colesterol 569 mg/dL. En la orina, hay 4+ de protena, sin presencia de eritrocitos. El diagnstico ms probable es (sigue la misma lista de opciones de la pregunta anterior).

Bajo 10

Aunque este tem se le adjudic la etiqueta de largo, es bastante ms breve que la mayora de los temes presentes en la seccin 2 del USMLE. Creemos que los recin graduados de una escuela de medicina deben ser capaces de evaluar la informacin proveniente del paciente, sintetizar los hallazgos importantes, y llegar a una conclusin. Como resultado, nuestros temes tienden a presentar una mezcla de hallazgos importantes y no importantes. Por otro lado, a veces sintetizamos los hallazgos en frases como la historia familiar no aporta datos importantes.

[ENG: positive fluid wave]

60

Redactar temes relacionados a las tareas que realiza un mdico


Cada tpico requiere ser abordado de una manera levemente diferente al redactar preguntas. Aqu damos algunos ejemplos de preguntas introductorias y otros consejos prcticos para guiar la redaccin de preguntas para cada tarea que realiza un mdico.

Salud y promocin de la salud 1


Los temes en esta rea examinan la habilidad de evaluar factores de riesgo, comprender informacin epidemiolgica y aplicar medidas preventivas. Las preguntas relacionadas a la salud y a la promocin de la salud habitualmente pertenecen a una de las siguientes categoras: 1) exmenes de chequeo mdico 2 ; 2) interferencia constructiva 3 ; 3) inmunizaciones/medicina del viajero o 4) intervencin de urgencia. En general, inicie el tem con una vieta clnica que describe un paciente. Adems de la historia clnica y de los hallazgos del examen fsico, estas vietas pueden incluir informacin acerca de las inmunizaciones, factores de riesgo, y de la historia familiar. Informacin acerca de la comunidad puede ser pertinente y por tanto debera ser incluida, pero las preguntas debieran enfocarse en un paciente individual. Las preguntas NO debieran enfocarse en la evaluacin directa de datos aislados. Por ejemplo, evite preguntar acerca de la causa ms importante de muerte en una subpoblacin; en cambio, concntrese en la aplicacin del conocimiento. Al preguntar sobre inmunizaciones o pruebas de deteccin o chequeo mdico, considere proporcionar al examinando una tabla con los datos pertinentes para evitar la memorizacin de recomendaciones contradictorias. Las siguientes preguntas introductorias son ejemplos que podran usarse en esta categora: Cul de las siguientes inmunizaciones debieran administrarse en este momento? Cul de las siguientes es la prueba de chequeo mdico ms apropiada? Cul de las siguientes pruebas habra pronosticado estos hallazgos? Cul de las siguientes es la intervencin ms apropiada? Para cul de las siguientes condiciones el paciente presenta el mayor riesgo? Cul de las siguientes es la ms probable de haber prevenido esta afeccin? Cul de los siguientes es el paso ms apropiado a seguir para prevenir morbilidad/mortalidad/discapacidad? Cul de lo siguiente debera recomendarse para prevenir una discapacidad por esta lesin/afeccin? Cul de las siguientes opciones de tratamiento precoz habra prevenido la afeccin de este paciente? Cul suplemento entre los siguientes habra probablemente prevenido esta condicin?

1 2

[ENG: health maintenance] [ENG: screening tests] 3 [ENG: constructive interference]

Captulo 5. Contenido de las preguntas: Evaluacin de la aplicacin del conocimiento de Ciencias Clnicas

61

Un nio de 15 aos de edad sufre dos episodios de choque anafilctico despus de picaduras de abejas. Cul de las siguientes es la intervencin ms adecuada? A. Administracin de corticoides durante el verano B. Terapia antihistamnica de larga duracin C. Ropa que lo proteja * D. Desensibilizacin con extracto de veneno de abeja E. Restringirlo a permanecer en casa durante los meses de verano.

Una mujer de 33 aos, gesta 1, para 1, a las 38 semanas de gestacin da a luz un recin nacido de sexo femenino. El recin nacido presenta hepato-esplenomegalia, ducto arterioso persistente y cataratas. A las 8 semanas de embarazo, la madre desarroll un cuadro consistente en un exantema maculopapular, adenopatas cervicales, 1 odinofagia y artralgias que desapareci en una semana. Posteriormente, el curso prenatal fue normal. Cul de los siguientes exmenes durante el embarazo es ms probable que hubiera detectado la anormalidad en el feto? A. Amniocentesis B. Cultivo para el virus herpes simplex * C. Ttulos seriados para rubola D. Examen de orina para Citomegalovirus E. Test de VDRL

A una mujer de 75 aos de edad, en buenas condiciones de salud, se le encuentra en un chequeo de rutina un colesterol srico total de 208 mg/dL y una concentracin de HDL srico de 70 mg/dL. El electrocardiograma no muestra anormalidades. Cul de las siguientes recomendaciones dietticas es ms apropiada? A. Reducir la ingestin de colesterol B. Reducir la ingestin de grasas saturadas C. Reducir ingestin de hidratos de carbono simples D. Aumentar ingestin de fibra * E. Sin cambios en la dieta

A un hombre de 33 aos de edad se le encuentra una presin arterial de 166/112 mmHg. Los electrolitos sricos son normales. Cul de las siguientes enfermedades o patologas ser reducida con mayor probabilidad por un tratamiento antihipertensivo efectivo? A. Aneurisma artico B. Insuficiencia cardaca congestiva C. Infarto agudo del miocardio D. Insuficiencia renal * E. Accidente vascular 2

1 2

[ENG: sore throat] [ENG: Stroke]

62

Mecanismo de la enfermedad
Estas preguntas debieran evaluar el conocimiento de fisiopatologa en su sentido ms amplio, incluyendo etiologa, patognesis, historia natural, curso clnico, hallazgos asociados, complicaciones, gravedad de la enfermedad y los efectos deseados como los no deseados de intervenciones teraputicas. Las preguntas acerca de mecanismos debieran enmarcarse en un contexto clnico. En general, inicie sus temes acerca de mecanismos con una vieta clnica de un paciente y sus sntomas, signos, historia, exmenes de laboratorio, etc. Luego confeccione una pregunta parecida a una de stas: Cul de los siguientes es la explicacin ms probable de los hallazgos? Cul de las siguientes es la localizacin ms probable de la lesin? Cul de los siguientes es el patgeno ms probable? Cul de los siguientes hallazgos es ms probable que est aumentado o reducido? Una biopsia mostrara con mayor probabilidad lo siguiente:

Una nia de 10 aos desarrolla hematuria macroscpica 14 das despus de haber presentado odinofagia. Su presin arterial es de 170/100 mmHg y presenta edema 2+ en la regin pedia y pretibial. Nitrgeno ureico = 3.2 mg/dL. Cul de las siguientes opciones es la causa ms probable del nivel de la presin arterial? A. Coartacin de la aorta B. Reduccin del factor vasodilatador derivado del endotelio vascular 1 C. Aumento en la produccin de aldosterona D. Aumento en la produccin de catecolaminas * E. Expansin del volumen intravascular

Un hombre de 32 aos de edad presenta secrecin uretral purulenta. El cultivo uretral muestra desarrollo de Neisseria gonorrheae sensible a la penicilina. Una semana despus de terminar el tratamiento con penicilina el paciente vuelve a presentar secrecin uretral. El cultivo vuelve a mostrar N. Gonorheae sensible a la penicilina. Tanto el paciente como su pareja sexual son seronegativos para VIH. Al examinar la pareja del paciente se encuentra una fisura anal y el cultivo uretral no muestra desarrollo de N. gonorrheae. Cul de las siguientes opciones es la causa ms probable de la reaparicin de la infeccin uretral? A. Infeccin concomitante con virus herptico B. Desarrollo de resistencia bacteriana C. Tratamiento inadecuado con penicilina * D. Reinfeccin a travs de la pareja

[ENG: endothelial-derived relaxant factor] (Nota del T: hoy es la prostaglandina E)

Captulo 5. Contenido de las preguntas: Evaluacin de la aplicacin del conocimiento de Ciencias Clnicas

63

Diagnstico
Para esta categora, redacte temes que requieren que los examinandos interpreten hallazgos de la historia clnica, examen fsico, resultados de laboratorio, estudios de imagen 1 y otros estudios para determinar el diagnstico ms probable (diagnstico diferencial) o el prximo paso para llegar a un diagnstico (exmenes diagnsticos). Al redactar preguntas en esta rea piense si desea evaluar la habilidad para integrar conocimiento a travs de varias pasantas o rotaciones clnicas 2 . El tem clsico de diagnstico comienza con una descripcin del paciente (incluyendo la edad, el sexo, sntomas y signos y su duracin, historia, hallazgos del examen fsico, hallazgos en pruebas diagnsticas y de laboratorio) y finaliza con una pregunta: Cul de los siguientes es el diagnstico ms probable? Cul de los siguientes es el paso ms apropiado para llegar a un diagnstico? Cul de los siguientes tiene la mayor probabilidad de confirmar el diagnstico? Un hombre de 52 aos de edad padece de una disnea progresiva con tos productiva y esputo purulento que aparecieron hace dos das. Es fumador de un paquete de cigarrillos desde hace 30 aos. Su temperatura es 37.2 C (99F). Los ruidos respiratorios se escuchan distantes con algunos roncus y estertores 3 . Su recuento de leucocitos es de 9 000/mm3 con recuento diferencial normal. La tincin de Gram del esputo revel numerosos neutrfilos y diplococos Gram negativos. La radiografa de trax muestra hiperinsuflacion pulmonar. Cul de los siguientes es el diagnstico ms probable? A. Asma B. Bronquiectasia * C. Bronquitis D. Embolia pulmonar E. Neumona estreptoccica Una mujer de 28 aos de edad presenta palpitaciones que ocurren aproximadamente una vez a la semana duran 1 a 5 minutos y son regulares y rpidas. Los episodios comienzan y terminan repentinamente y no se asocian a malestar torxico o disnea. No hay historia de problemas cardiacos. Bebe dos o tres tazas de caf al da. Rara vez bebe alcohol y no es fumadora. Su presin arterial es 120/88 mmHg y su pulso es de 96/min y regular. Presencia de mirada fija y retraccin palpebral. El tiroides se palpa firme y aumentado de tamao, 1.5 veces lo normal. Existe un clic mesosistlico en el pex y un soplo protosistlico de grado 2/6 en el borde esternal superior izquierdo. El ECG es normal, excepto por la evidencia de una taquicardia sinusal. Cul de los siguientes es el paso ms apropiado para llegar a un diagnstico? A. Monitoreo por ECG ambulatorio *B. Medicin del nivel de hormona estimulante del tiroides (TSH) en el suero C. Medicin del nivel de catecolaminas en la orina D. MUGA scan E. Ecocardiografa

1 2

[ENG: imaging] [ENG: clerkships] 3 [ENG: ronchi y wheezes]

64

Manejo del paciente 1


Estos temes evalan fundamentos de la atencin aguda y crnica tanto en pacientes intrahospitalarios como en el consultorio externo. Al redactar temes de manejo de pacientes es especialmente importante centrarse en aspectos de la atencin pertinentes al nivel de entrenamiento del examinando (bajo supervisin, supervisin limitada, prctica independiente, subespecialista). Para la seccin 2 del USMLE nos concentramos en niveles de atencin de pacientes apropiados a un interno principiante, sin importar la especialidad. Adems agregamos temes en una situacin de urgencia en el medio de la noche cuando el examinando podra no disponer de ayuda externa. Las preguntas incluyen decisiones de farmacoterapia, decisiones ms generales acerca del tratamiento en que las opciones incluyen una mezcla de farmacoterapia y otros tratamientos y decisiones del manejo del paciente, en donde las opciones incluyen una mezcla de tratamientos y otras cosas tales como pruebas de diagnstico. Preguntas para usar: Cul de las siguientes opciones es el paso inicial o el siguiente paso ms apropiado en el manejo de este paciente? Cul de las siguientes opciones es el manejo ms efectivo en este paciente? Cul de las siguientes opciones es la farmacoterapia ms apropiada? Cul de las siguientes opciones es la primera prioridad en el manejo de este paciente (Ejemplo: en urgencia)? Una mujer de 55 aos de edad, se encuentra hospitalizada con el diagnstico de cirrosis descompensada. Est siendo tratada con espironolactona, elixir de cloruro de potasio, furosemida. Est hiporreactiva 2 e hipotensa pero sin signos de dificultad respiratoria. Presenta hallazgos compatibles con enfermedad heptica crnica, ascitis, y edema perifrico leve. El ECG muestra una frecuencia regular de 55/min, ausencia de ondas P, y el complejo QRS es ancho y aplanado, el segmento ST aplanado y onda T ancha. Cul de los siguientes debiera inyectarse en forma intravenosa? A. Calcio B. Lidocana C. Magnesio D. Suero salino 0.9% * E. Potasio Un nio de 15 aos de edad, previamente en buenas condiciones de salud, sufre dolor periumbilical de tipo calambre. Despus de varias horas el dolor se traslada al cuadrante inferior derecho y se hace constante. Vomita varias veces y es trado a urgencias. Al examen fsico el abdomen est sensible a la palpacin del cuadrante inferior derecho. Las radiografas de trax y de abdomen son normales. El recuento de leucocitos muestra 15 000/mm3. El examen de orina muestra 3 leucocitos por campo. Cul de las siguientes opciones es la medida ms apropiada en el manejo inicial de este paciente? A. Tratamiento de soporte en la casa, volver si el dolor aumenta B. Enema de bario C. Tomografa Axial 3 de abdomen D. Pielografa intravenosa y cistografa * E. Exploracin quirrgica del abdomen 4

1 2

[ENG: Management] [ENG: barely responsive] 3 [ENG: CT scan] 4 Nota del T: hoy la respuesta correcta sera otro procedimiento, como ecotomografa abdominal, para aclarar la causa del abdomen agudo, antes de la exploracin quirrgica

Captulo 5. Contenido de las preguntas: Evaluacin de la aplicacin del conocimiento de Ciencias Clnicas

65

Redactar temes de tpicos difciles


Es comn creer que muchos tpicos no se prestan para ser abordados con el formato de seleccin mltiple. Hemos tenido un xito razonable en generar temes en muchas de estas reas y recomendamos la siguiente estrategia. 1. Despus de identificar un tpico en el cual usted tiene cierta dificultad, revise todas las fuentes de material para el examen y seleccione preguntas del tpico que usted cree aceptables. 2. Identifique los rasgos ms importantes 1 de estos temes y luego trate de desarrollar una plantilla 2 que permita a los profesores redactar temes similares. 3. Para tpicos en que no hay ejemplos de temes, piense en lo que desea evaluar. Vaya ms all de la lista de tpicos mediante el diseo de tareas relacionadas con el tpico y que los estudiantes de medicina deben saber (esto es, cada elemento de la lista debe contener un verbo). Para ilustrar este proceso, los siguientes prrafos esbozan el proceso que se us para redactar preguntas de uno de los exmenes para autorizar la prctica de la medicina en los Estados Unidos (seccin 2 del USMLE) en el tema de tica Mdica y de Jurisprudencia. El esquema de contenidos incluye los siguiente tpicos: 1) Consentimiento y consentimiento informado 3 para el tratamiento de un paciente (por ejemplo, decirlo todo 4 , terapias alternativas, riesgos y beneficios); 2) Relacin mdico-paciente (por ejemplo, decirle al paciente la verdad, confidencialidad, privacidad, autonoma, informar a una agencia sobre la enfermedad; 3) La muerte y el proceso de morir (por ejemplo, diagnosticando la muerte, soporte vital, autopsia, donacin de rganos, eutanasia, suicidio); 4) Asuntos relacionados al nacimiento (por ejemplo, diagnstico prenatal, aborto, conflicto materno-fetal; y 5) Aspectos relacionados a los estudios de investigacin en seres humanos (por ejemplo, consentimiento, placebos, conflicto de inters, poblaciones vulnerables).

Los autores tpicamente redactan preguntas tales como: Cul de las siguientes es la definicin de consentimiento informado? o describen un escenario y preguntan cul principio de tica est ilustrado en el escenario. A continuacin se presenta un ejemplo de pregunta que se us en el pasado.

La base legal para disminuir las restricciones de los abortos en los Estados Unidos puede relacionarse ms estrechamente con A. la legislacin federal B. una determinacin de una corte federal C. la legislacin de un estado D. una determinacin de una corte estatal E. una determinacin de la Asociacin Americana de Medicina

En forma irreverente a este tipo de pregunta se le llama preguntas del tipo a nadie le interesa 5 .

1 2

[ENG: key features] [ENG: template] 3 [ENG: Consent and informed consent] 4 [ENG: full disclosure] 5 [ENG: who cares questions]

66

Revisamos el banco de preguntas y decidimos que estbamos poco interesados en si los estudiantes saban o no las definiciones. En cambio, queramos evaluar si los examinandos podan aplicar principios ticos en sus decisiones relacionadas al cuidado de pacientes. Reunimos a un grupo de redactores de temes que, basndose en las preguntas modelos, procedieron a generar nuevos temes para el examen. Todas las preguntas incluan una vieta de un paciente y requeran del estudiante que indicara lo que debera hacer el mdico en esas circunstancias o le pedan al estudiante que evaluara cun apropiadas eran las acciones del mdico descritas en la vieta. Ninguna pregunta se enfoc en la definicin de trminos. Las siguientes son algunos ejemplos de las preguntas que redactamos.

Una enfermera es hospitalizada y se le realiza una apendicectoma en el hospital donde trabaja. Una semana despus de ser dada de alta, el asistente del director del hospital le pregunta al cirujano cul fue el diagnostico final. Cul de las siguientes opciones es la respuesta ms adecuada que debiera dar el cirujano? A. Responder, porque har que el papeleo del seguro de salud camine ms rpido. B. Responder, porque como empleado del hospital, el ayudante del director tiene acceso a informacin de pacientes. C. Responder, por la posibilidad de que se disemine informacin equivocada acerca de la paciente. D. Declinar de dar una respuesta porque el ayudante del director no es un mdico. *E. Declinar de dar una respuesta, porque la informacin es confidencial.

Un nio de 8 aos de edad, que padece de leucemia linfoblstica aguda, ha sufrido tres recadas en los ltimos 2 aos. El nico tratamiento disponible es una quimioterapia experimental. Sin este tratamiento, es poco probable que el nio sobreviva ms de seis semanas. Se desconoce la probabilidad de sobrevida con este tratamiento. Los padres del nio no quieren administrarle ms tratamientos y desean llevrselo a casa. El nio tambin quiere irse a casa. Cul de las siguientes opciones es el camino ms apropiado a seguir? A. Dar de alta al nio contra el consejo de los mdicos. *B. Dar de alta al nio en forma rutinaria. C. Pedir al juzgado una orden para tratarlo. D. Informar al servicio social acerca de la negligencia mdica de los padres.

Captulo 5. Contenido de las preguntas: Evaluacin de la aplicacin del conocimiento de Ciencias Clnicas

67

68

Seccin III temes de Emparejamiento Ampliado1, 2

La Seccin III aporta la informacin concerniente a los temes de Emparejamiento Ampliado. El Captulo 6 introduce el formato de Emparejamiento Ampliado en el cual se solicita a los examinandos que escojan una mejor respuesta. El Captulo 7 introduce el formato en el cual a los examinandos se les pide que escojan un determinado nmero de opciones, habitualmente ms de una.

[ENG: Extended-Matching Items] No existe una traduccin universalmente aceptada para extended matching items. Incluso para temes de matching stos se traducen de diferentes maneras: temes de trminos pareados, de respuesta por pares, de correspondencia. La decisin de usar emparejamiento ampliado en este manual debe considerarse slo una opcin entre varias, en que ninguna es totalmente satisfactoria.
2

Captulo 6 temes de Emparejamiento Ampliado (Tipo R) 1


Los temes de Emparejamiento Ampliado son temes de seleccin mltiple que se organizan en grupos o conjuntos 2 que usan una sola lista de opciones para todos los temes del conjunto. Un conjunto de Emparejamiento Ampliado bien construido incluye cuatro componentes: 1. Un tema 2. Una lista de opciones 3. Una declaracin introductoria 3 y 4. Al menos dos enunciados de temes 4 como se ilustra ms abajo. Tema: Opciones: Fatiga A. B. C. D. E. F. G. Leucemia aguda Anemia por enfermedad crnica Insuficiencia cardaca congestiva Depresin Infeccin por virus de Epstein-Barr Deficiencia de folato Deficiencia de Glucosa-6-fosfatodeshidrogenasa H. I. J. K. L. M. N. Esferocitosis hereditaria Hipotiroidismo Deficiencia de hierro Enfermedad de Lyme Anemia hemoltica microangioptica Tuberculosis miliar Deficiencia de vitamina B12 (cianocobalamina)

Declaracin introductoria Enunciados:

Seleccionar el diagnstico ms probable para cada paciente con fatiga.

1. Una mujer de 19 aos de edad ha padecido de fatiga, fiebre, y dolor de garganta durante la pasada semana. Tiene una temperatura de 38.3o C (101F), linfadenopata cervical y esplenomegalia. Estudios iniciales de laboratorio muestran un recuento de leucocitos de 5 000/mm3 (80% linfocitos, con abundantes linfocitos atpicos). La aspartato aminotransfera y gamaglucanil transpepeptidasa sricas (AST, GOT) son de 200 U/L. Las concentraciones de bilirrubina srica y fosfatasas alcalinas son normales. Respuesta: E 2. Una nia de 15 aos tiene una historia de dos semanas de evolucin con fatiga y dolor de espalda. Tiene equimosis extensas 5 en la piel palidez y sensibilidad al tacto sobre sus vrtebras y ambos fmures. Un hemograma completo demostr una concentracin de hemoglobina de 7.0 g/dL, leucocitos de 2 000/mm3 con un recuento de plaquetas de 15 000/mm3. Respuesta: A

1
2

[ENG: Extended Matching (R-Type) Items] [ENG: sets] 3 [ENG: lead-in statement] 4 [ENG: item stems] 5 [ENG: widespread bruising]

Captulo 6. temes de Emparejamiento Ampliado (Tipo R)

71

Los temes de Emparejamiento Ampliado se redactan de modo distinto que los temes tradicionales de seleccin mltiple. A menudo, se desarrollan primero el tema, la declaracin introductoria y las opciones, para finalizar con los enunciados. Por ejemplo, si usted desea redactar preguntas relacionadas al diagnstico de fatiga, se comienza con un listado de diagnsticos que producen fatiga. Luego se redacta la vieta 1 para cada una (o por lo menos muchas) de las opciones en la lista. El ejemplo anterior incluye vietas para la infeccin por virus de Epstein-Barr y para leucemia aguda. Se podra redactar temes adicionales para el resto de los diagnsticos de la lista. Para diagnsticos comunes se podra redactar ms de un tem. Las vietas usadas en los ejemplos muestran una extensin moderada, pero tambin podran usarse vietas ms cortas y ms enfocadas. Alternativamente, se podra pedir a los examinandos que identifiquen informacin diagnstica clave, entremezclada con hallazgos secundarios, usando vietas ms extensas

Como evitar errores al redactar temes de Emparejamiento Ampliado para sus exmenes.
Los cuatro componentes (el tema, las opciones, la declaracin introductoria y los enunciados) son esenciales para construir un conjunto de Emparejamiento Ampliado de buena calidad. No deberan usarse conjuntos que no poseen una declaracin introductoria o que tienen declaraciones introductorias inespecficas tales como Encuentre la correspondencia de cada tem con la mejor opcin ya que generalmente stos confunden a los examinandos con tareas a realizar de naturaleza ambigua o inconsistente. El siguiente ejemplo de un conjunto contiene errores ya que las opciones son heterogneas, no hay declaracin introductoria y los enunciados no pueden responderse sin leer las opciones. Las reglas para los temes de Emparejamiento Ampliado son completamente anlogas a aquellas de los temes de una mejor respuesta.

Ejemplo de conjunto de Emparejamiento Ampliado con errores


A. B. C. D. E. F. G. H. es el mareo 2 no tiene efecto sobra las personas aumenta en forma indirecta el CO2 causa la muerte aumenta la sensibilidad a los aromas es una reduccin en la visibilidad esttica, economa, salud productos de la combustin de combustibles fsiles I. J. K. L. M. N. se pueden controlar completamente causan dao ocular y a las plantas son insignificantes aumentan el riesgo de cncer de piel no pueden ser controladas un exceso de enfermedades agudas respiratorias en los nios O. es contrario a la opinin pblica

1. Factores que la gente considera al evaluar la calidad del aire 2. Un efecto principal de la presencia de partculas en el aire 3. Los productos del smog fotoqumico 3

1 2

[ENG: vignette] [ENG: motion sickness] 3 [ENG: photochemical smog]

72

Despus de leer el enunciado en el nmero 1 los examinandos tendrn una idea muy vaga de lo que trata la pregunta. En un esfuerzo para determinar la mejor respuesta, los examinandos debern decidir si es el mareo es mejor que no tiene efecto sobra las personas. Simplemente esta tarea es impracticable. Bajo estas circunstancias, si una opcin no es absolutamente verdadera o falsa, no debe incluirse con el resto de las opciones. Adems el enunciado del tem 1 no est claro ya que la pregunta no puede responderse sin leer las opciones. Tal como en las preguntas de una mejor respuesta, los enunciados deben ser extensos y las opciones cortas. DEBE existir una declaracin introductoria que establezca una relacin entre los temes y las opciones. NO debe haber verbos en las opciones. La regla de cubrir las opciones es tan relevante para los temes de Emparejamiento Ampliado como para los de una mejor respuesta correcta.

Captulo 6. temes de Emparejamiento Ampliado (Tipo R)

73

Ejemplos de declaraciones introductorias y de tpicos para las listas de opciones


Las vietas de pacientes proporcionan una excelente estructura para los enunciados, no solamente en las ciencias clnicas, sino tambin en la evaluacin del conocimiento de ciencias bsicas. Las declaraciones introductorias comienzan generalmente con una frase tal como Para cada uno de los siguientes pacientes. A menudo estos conjuntos se organizan alrededor de motivos de consulta u otro factor que permita una frase introductoria ms especfica tal como Para cada uno de los siguientes pacientes que presenta fatiga, o Para cada uno de los siguientes pacientes que presentan una deficiencia enzimtica. La segunda parte de la declaracin introductoria describe la tarea y el conjunto de opciones: seleccione el diagnstico ms probable, seleccione la protena que tiene la mayor probabilidad de ser defectuosa. Los siguientes son ejemplos de declaracin introductoria y tpicos sugeridos para confeccionar las listas de opciones. Para cada uno de los siguientes pacientes, seleccione [ejemplo: un nervio] que es ms probable que est [anormal/con defectos/deficiente/sin funcionar]. Los conjuntos de opciones podran incluir lista de nervios, lista de enzimas, lista de hormonas, lista de protenas, lista de tipos de clulas, lista de neurotransmisores, lista de procesos patolgicos. Para cada uno de los siguientes pacientes, seleccione el hallazgo que se podra esperar. Los conjuntos de opciones podran incluir una lista de resultados de laboratorio; listas con signos adicionales del examen fsico, resultados de una autopsia, resultados de un examen microscpico de lquidos corporales, tejido muscular o de articulacin, resultados del anlisis de ADN, niveles hormonales. Para cada uno de los siguientes pacientes, seleccione la [causa] ms probable. Los conjuntos de opciones podran incluir listas de mecanismos de enfermedades, medicamentos que podran causar efectos colaterales, lista de medicamentos o clases de medicamentos, agentes txicos, mecanismos hemodinmicos. Para cada uno de los siguientes pacientes, seleccione el [ejemplo: medicamento] que debiera administrarse. Los conjuntos de opciones podran incluir una lista de medicamentos, vitaminas, aminocidos, enzimas, hormonas. Para cada uno de los siguientes pacientes con motivos de consulta, seleccione el diagnstico ms probable. Los conjuntos de opciones podran incluir una lista de diagnsticos, a menudo organizado alrededor de un motivo de consulta tales como afecciones que causan dolor o enfermedades asociadas a fiebre. Para cada uno de los siguientes pacientes, seleccione el paso ms apropiado en el cuidado del paciente. Los conjuntos de opciones podran incluir una lista de terapias farmacolgicas, lista de estudios de laboratorio, alternativas para dar de alta 1 , o las opciones podran incluir un conjunto mixto de tratamientos y estudios adicionales para evaluar si el estudiante puede reconocer cuando ya cuenta con suficiente informacin.
1

[ENG: disposition alternatives]

74

Aspectos adicionales sobre las opciones en los Conjuntos R 1


En general, cualquier asunto que pueda expresarse como una lista puede servir para las opciones en un Conjunto R. A continuacin se presenta una serie de tpicos que se han usado como base para producir listas de opciones. Arterias Nervios Msculos Aminocidos Pptidos Hormonas Enzimas Componentes celulares Tipos de clulas Componentes de la sangre Molculas Cariotipos Protenas Lpidos Patgenos/bacterias/hongos Virus Citoquinas Toxinas Vitaminas/minerales Diagnsticos Tipos de tejido conectivo Estructuras anatmicas Estructuras endocrinas Neurotransmisores Defectos metablicos Enfermedades inmunolgicas Componentes del aparato locomotor Estructuras cardacas Organelos Anomalas congnitas Segmentos de la mdula espinal Componentes del sistema nervioso central Productos de secrecin Componente del extracelular Alternativas de manejo del paciente Medicamentos/clases de medicamentos Procesos patolgicos Estados fisiopatolgicos Anormalidades de los electrolitos Tests diagnsticos

Las listas de opciones debieran ser palabras nicas o frases muy cortas. Deben adems ser homogneas (es decir, diagnsticos nicamente, solamente opciones de manejo de pacientes, solamente sitios anatmicos, nicamente vitaminas, etc.). Pueden adems ser grficos u otro material pictrico que contenga rtulos en reas significativas. En algunos casos, las opciones, especialmente los resultados de laboratorio, se expresan en forma de tablas (ver ms adelante el ejemplo de fisiologa). Incluya todas las opciones relevantes que son apropiadas para los examinandos. Probablemente no es apropiado incluir distinciones sutiles o diagnsticos raros. Para algunos tpicos, podra ser adecuado un nmero bajo de opciones probablemente tan bajo como tres opciones. Para otros se podra requerir hasta 26 opciones (una por cada letra del alfabeto inglsamericano).

[ENG: R-Sets]

Captulo 6. temes de Emparejamiento Ampliado (Tipo R)

75

Redaccin de enunciados
Las vietas de pacientes proporcionan una excelente estructura como enunciados, tanto en ciencias bsicas como en las ciencias clnicas. En las ciencias clnicas, la vieta comnmente muestra la edad del paciente, su gnero, motivo de consulta, y el lugar donde se da el cuidado clnico 1 . Luego se agrega la historia personal, la historia familiar (si es necesario), informacin concerniente al examen fsico y finalmente datos de exmenes de laboratorio (si competen al caso). Segn sea el objetivo del conjunto, las vietas pueden ser breves, presentaciones prototpicas o descripciones ms completas que desafan al examinando para que identifique la informacin clave. Generalmente, estos temes incluyen al menos la edad del paciente, su gnero, el motivo de consulta y la historia pertinente. Los temes destinados a evaluar conocimientos de ciencias bsicas, especialmente en aquellos cursos del primer ao de la carrera de medicina, podran incluir menos descripciones prototpicas detalladas. Cada descripcin de pacientes debe tener la misma estructura que las otras en cada conjunto. Por ejemplo, si la raza, origen tnico y ocupacin son incluidos en un tem, se deben incluir en todos los dems. Asimismo, si se incluye informacin de laboratorio, debe incluirse en todos los temes. Es preferible no mezclar pacientes adultos con nios en un mismo conjunto, ya que a menudo la edad por s misma aporta demasiadas pistas 2 por lo que el examinando elimina un nmero de opciones slo por este motivo. Una ventaja de usar vietas de pacientes es que sirve para asegurar que se est evaluando aplicacin del conocimiento. Estos temes no debieran parecerse a crucigramas, en los cuales tanto las opciones como el enunciado son palabras nicas o frases breves. Debe evitarse la construccin de temes en que se solicita dibujar una raya entre un tem en la columna A y su correspondiente respuesta en una columna B. Es particularmente importante que los temes sean honestos 3 . No hay ninguna razn para usar triquiuelas. Una lista de opciones suficientemente larga asegura que la pregunta presenta una dificultad suficiente como para identificar a los estudiantes con mayores conocimientos sin recurrir a argucias. Tal como en las preguntas de una mejor respuesta tipo A bien construidas, la regla de cubrir las opciones sigue siendo importante. Los estudiantes con conocimientos debieran poder generar una respuesta a la pregunta y luego encontrarla entre la lista de opciones alfabticas. Se debiera preparar un tem para la mayora de las opciones. Incluso, para las opciones ms importantes o ms comunes, se podra preparar ms de un tem. Al construir un examen que evala competencias generales debieran usarse todas las opciones y luego seleccionar slo dos o tres temes para el examen y as evitar enfatizar demasiado un tpico. Luego se guarda el resto de los temes no usados para otros exmenes posteriores. Por el contrario, si se desea examinar un tpico en mayor profundidad, puede incluirse 10 a 20 temes en cada conjunto y calcular un puntaje separado para cada tpico. Al revisar los temes verifique que existe slo una mejor respuesta para cada pregunta. Tambin asegrese que por lo menos hay 4 distractores razonables para cada tem. Para una verificacin final, se recomienda que le pida a un colega que revise los temes (sin mostrarles la respuesta correcta). Si el colega tiene dificultad para encontrar la respuesta correcta se debe modificar la lista de opciones o el tem para eliminar ambigedades.

1 2

[ENG: site of care [ENG: too much cueing]] 3 [ENG: straightforward]

76

Ejemplos de buenos y malos enunciados usando la misma lista de opciones


El siguiente es un buen conjunto para microbiologa. Las opciones son una lista homognea de patgenos que incluye virus y bacterias. Tiene una declaracin introductoria que presenta una tarea clara para el examinando. Tiene dos enunciados que requieren que el estudiante aplique sus conocimientos de ciencias bsicas en microbiologa para reconocer la causa ms probable de la enfermedad de cada paciente. A. B. C. D. E. F. G. H. I. J. K. Adenovirus Aspergillus fumigatus Bacillus anthracis Candida albicans Chlamydia psittaci Coccidioides inmitis Coronavirus Corynebacterium diptheriae Coxiella burnetti Virus coxsackie Virus Epstein-Barr L. M. N. O. P. Q. R. S. T. U. Haemophilus influenzae Histoplasma capsulatum Mycobacterium tuberculosis Mycoplasma pneumoniae Neisseria gonorrheae Neisseria mengitiditis Pneumocystis carinii Rhinovirus Streptococcus pneumoniae Streptococcus pyogenes (grupo A)

Para cada paciente con fiebre, seleccione el patgeno ms probable que est causando la enfermedad. 1. Una nia de 7 aos presenta fiebre alta y dolor de garganta. La faringe est enrojecida, tiene una amgdala derecha aumentada de tamao con exudado blanquecino, y una adenopata submaxilar derecha dolorosa. El cultivo en agar-sangre muestra numerosas colonias hemolticas que son inhibidas con bacitracina. Respuesta: U Un adolescente de 18 aos presenta desde hace una semana fiebre, dolor de garganta, decaimiento y ambas amgdalas aumentadas de tamao, con exudado. Se constata una adenopata linftica cervical y esplenomegalia. Hay linfocitosis con linfocitos atpicos. La serologa del paciente es positiva para anticuerpos heterfilos. Respuesta: K

2.

El siguiente enunciado, desarrollado para el mismo conjunto anterior, evala el recuerdo de datos aislados en lugar de aplicacin del conocimiento. Aparece ms como un crucigrama que como una pregunta a ser usada en una escuela de medicina. 3. Un organismo encapsulado, gram positivo, que habitualmente crece en pares o cadenas cortas. Respuesta: T

Captulo 6. temes de Emparejamiento Ampliado (Tipo R)

77

El siguiente conjunto incluye dos enunciados. El primero requiere que el examinando sintetice la informacin para determinar el diagnstico. El segundo solamente requiere el recuerdo de un dato aislado. A. B. C. D. E. F. G. H. Vitamina A Vitamina B1 Vitamina B2 Vitamina B6 Vitamina C Vitamina D Vitamina E Vitamina K I. J. K. L. M. N. O. P. Biotina Cobre Folato Yodo Hierro Magnesio Niacina Zinc

Para cada paciente con hallazgos clnicos causados por anormalidades metablicas, seleccione la vitamina o mineral que sea ms probable que est involucrada/o 1. Un viudo de 70 aos presenta equimosis, petequias perifoliculares e hinchazn de las encas. Su dieta consiste preferentemente de bebidas de fantasa (cola) y salchichas. Respuesta: E Interviene en la sntesis de un factor de la coagulacin. Respuesta: H

2.

78

El siguiente conjunto incluye dos enunciados. El primero requiere que el examinando sintetice la informacin para determinar el diagnstico de trisoma 21. El segundo tem provee esa informacin. Al desarrollar enunciados se debe decidir el nivel de sntesis que se proveer al examinado. A. B. C. D. Defecto del tabique interauricular Coartacin de la aorta Transposicin completa de los grandes vasos Defecto del cojinete endocrdico E. F. G. H. Ductus arterioso persistente Estenosis de las vlvulas pulmonares Tetraloga de Fallot Defecto del tabique interventricular

Para cada paciente, seleccione el defecto cardaco congnito ms probable. 1. Un recin nacido de 3 090 g, producto de un embarazo de trmino es evaluado encontrndose en el examen fsico: hipotona, un reflejo de succin dbil, pliegues epicnticos prominentes, lengua grande, meiques pequeos y encorvados en ambas manos. Presenta un soplo holosistlico sobre todo el precordio y un frmito a la palpacin sobre el bode esternal izquierdo entre el segundo y cuarto espacio intercostal, sin cianosis. Respuesta: D Un recin nacido con trisoma 21 presenta hendiduras 1 de las vlvulas atrioventriculares derecha e izquierda sin cianosis. Respuesta: D

2.

[ENG: clefts]

Captulo 6. temes de Emparejamiento Ampliado (Tipo R)

79

El siguiente es un ejemplo de un tem con defectos. Este conjunto propone una tarea que es clnicamente al revs. Se le da al examinando una vacuna y se le pide que escoja cul sera el mejor paciente para esta vacuna. Una tarea ms apropiada sera que se le diera un paciente al examinando y luego se le preguntara cul es el mejor paso siguiente para el cuidado del paciente (por ejemplo describiendo un paciente en cada enunciado; usando Para cada paciente, seleccione el paso ms apropiado en su cuidado clnico y usar elementos tales como vacunas, como opciones). Un segundo problema con este conjunto es que no se da suficiente informacin acerca de cada paciente. Por ejemplo, al examinando le gustara conocer la historia de inmunizaciones de un paciente antes de decidir cules inmunizaciones recomendar. Ao de Sexo nacimiento 1980 M 1975 1970 1965 1960 1955 1950 1945 1940 1935 F F F M F F M M F Ocupacin Estudiante Pintor Profesora Abogado Pintor Empleada Enfermera Ejecutivo Chofer Ama de casa Emba- Enfermedades Historia razo de la niez Ninguna Ninguna S S No No No No Ninguna Ninguna Ninguna Sarampin Parotiditis Varicela Sarampin Rubola Se desconoce Alergias

A. B. C. D. E. F. G. H. I. J.

Productos de huevo Ninguna Penicilina Ninguna Ninguna Ninguna Productos de gelatina Diabetes Ninguna Ninguna Ninguna Ninguna Ninguna Hipertensin Ninguna Esplenectoma Ninguna Gastritis Sulfonamidas

Para cada vacuna, seleccione el paciente que representa su uso ms apropiado 1. 2. Vacuna contra el sarampin Vacuna anti meningoccica

80

Resumen de los pasos a seguir al confeccionar temes de Emparejamiento Ampliado


1. Identifique el tema para el conjunto. El tema puede ser un motivo de consulta (por ejemplo, dolor torcico, fatiga), una etapa en el proceso de admisin/alta en el departamento de emergencias), una clase de medicamentos (por ejemplo, agentes antihipertensivos, antibiticos). Redacte la declaracin introductoria para el conjunto. (Por ejemplo, para cada paciente descrito ms abajo, seleccione el diagnstico ms probable). La declaracin introductoria indica la relacin entre los enunciados y las opciones, clarificando la pregunta que se pone a los examinados. Es un componente esencial del conjunto de Emparejamiento Ampliado. Prepare la lista de opciones. La lista de opciones debe contener palabras nicas o frases muy breves. Ordene las opciones alfabticamente a menos que exista otra forma lgica de hacerlo. Redacte los temes. Los temes dentro de un conjunto deben tener una estructura similar. A menudo es apropiado usar las vietas de pacientes. Revise los temes. Asegrese que exista solamente una mejor respuesta para cada pregunta. Tambin verifique que estn presentes por lo menos cuatro buenos distractores para cada tem. Como etapa final, se recomienda pedirle a un colega que revise los temes (sin indicar la respuesta correcta). Si el colega tiene dificultades para indicar la respuesta correcta debe modificarse la lista de opciones o el tem para evitar ambigedades.

2.

3.

4.

5.

Informacin adicional sobre como redactar temes de Emparejamiento Ampliado puede encontrarse en:
Case SM and Swanson DB. Extended-matching items: a practical alternative to free-response questions. Teaching and Learning in Medicine. 1993;5(2):107-115. Case SM, Swanson DB, Woolliscroft JO. Assessment of diagnostic pattern recognition skills in medicine clerkships using a written test. In: Harden R, Hart I, Mulholland H, eds. Approaches to Assessment of Clinical Competence. Norwich, England: Page Brothers; 1992:452-458.

Captulo 6. temes de Emparejamiento Ampliado (Tipo R)

81

Ejemplo de conjuntos de Emparejamiento Ampliado


Ejemplo de conjunto para Anatoma
A. B. C. D. Arteria cerebral anterior izquierda Arteria cerebral anterior derecha Arteria cerebral media izquierda Arteria cerebral media derecha E. F. G. H. Arteria cerebral posterior izquierda Arteria cerebral posterior derecha Arterias lenticuloestriadas izquierdas Arterias lenticuloestriadas derechas

Para cada paciente con anormalidades neurolgicas, seleccione la arteria comprometida con mayor probabilidad. 1. Un hombre diestro de 72 aos presenta debilidad a hiperreflexia de la extremidad inferior derecha, una respuesta plantar en extensin del lado derecho, fuerza normal en el brazo derecho y movimientos faciales normales. Respuesta: A Un hombre diestro de 68 aos, presenta hemiparesia espstica del lado derecho, una respuesta plantar en extensin del lado derecho y una parlisis facial de los dos tercios inferiores de su hemicara derecha. Su lenguaje es fluido y tiene una comprensin normal de rdenes orales y escritas. Respuesta: G

2.

82

Un ejemplo de conjunto para Farmacologa


A. B. C. D. E. F. G. H. I. Acetaminofeno Amiodarona Inhibidores de ACE Aspirina Atenolol Bleomicina Citosina arabinsido Furosemida Metronidazol J. K. L. M. N. O. P. Q. R. Acido nalidxico Nitrofurantona Penicilina Prednisona Procainamida Propranolol Sulfasalazina Tetraciclina Verapamil

Para cada paciente, seleccione la droga ms probable de haber causado un efecto adverso 1. Un hombre de 56 aos que presenta arritmias ventriculares recurrentes ha estado en tratamiento con drogas antiarrtmicas por cinco meses. En la actualidad presenta disnea progresiva, tos y fiebre baja. La eritrosedimentacin est elevada. La radiografa de trax muestra una neumonitis intersticial difusa. Los tests de funcin pulmonar muestran que la capacidad de difusin para el monxido de carbono est disminuida. Respuesta: B Un hombre de 62 aos de edad que sufre de enfermedad obstructiva crnica pulmonar inicia terapia con una droga antihipertensiva. Dos semanas despus se ha empeorado su disnea y presenta sibilancias audibles a distancia. Respuesta: O

2.

Captulo 6. temes de Emparejamiento Ampliado (Tipo R)

83

Ejemplo de conjunto para Fisiologa Hallazgos clnicos (Cules hallazgos adicionales son los ms probables?)
pH 7.15 7.15 7.30 7.40 7.50 7.50 7.50 pO2 mmHg 98 98 56 100 100 100 56 pCO2 mmHg 33 24 80 40 33 24 33 HCO3- mEq/L 11 8 38 25 25 18 25

A. B. C. D. E. F. G.

Para cada paciente que se describe ms abajo, seleccione los valores de gases sanguneos arteriales ms probables. 1. Un hombre de 22 aos presenta una historia de poliuria y polidipsia de tres semanas de duracin. Durante las ltimas 12 horas se agregaron nuseas, vmitos y letargia. Un anlisis de orina (dipstick) muestra glucosa (++++) y cetonas (++++). Respuesta: B Una mujer de 25 aos es trada al servicio de urgencia 12 horas despus de un intento de suicidio. Ingiri aproximadamente 100 tabletas de 500mg de aspirina. Respuesta: F

2.

84

Ejemplo de conjunto para diagnstico


A. B. C. D. Espondilitis anquilosante Infeccin de disco intervertebral Mieloma mltiple Sndrome miofascial E. F. G. H. Osteoporosis Estenosis del canal raqudeo Espondilitis Tuberculosis de la columna vertebral

Para cada paciente con dolor de espalda, seleccione el diagnstico ms probable. 1. Un hombre de 26 aos ha desarrollado en forma paulatina dolor lumbar y rigidez matinal. El dolor se alterna en ambos lados y ocasionalmente se irradia hacia los glteos y la parte posterior de los muslos, hasta las rodillas. Al examen fsico presenta una uvetis anterior aguda, sensibilidad difusa en la regin sacro ilaca y una restriccin de movimiento a nivel de sus caderas. Su eritrosedimentacin es de 40 mm/h. El examen de fijacin de ltex es negativo. Adems presenta una leve anemia hipoproliferativa. Respuesta: A Una mujer de 28 aos presenta dolor vago en las regiones cervical y lumbar con cefalea y restriccin de la movilidad en la regin cervical. Doce horas antes tuvo un accidente automovilstico donde fue embestida desde atrs al ir manejando su coche. Presenta ansiedad y una contractura muscular muy dolorosa en el rea lumbar y sobre la nalga izquierda. Respuesta: D

2.

temes adicionales cubriran los diagnsticos restantes. Los ejemplos de vietas son de una longitud promedio. Podra usarse vietas ms enfocadas. En forma alternativa, podra presentarse una vieta ms larga para desafiar al examinando a identificar la informacin pertinente.

Captulo 6. temes de Emparejamiento Ampliado (Tipo R)

85

Ejemplo de conjunto de diagnsticos simples


A. B. C. D. E. F. G. H. I. J. Aneurisma abdominal Apendicitis Obstruccin intestinal Colecistitis Cncer de colon Estreimiento Diverticulitis Rotura de embarazo ectpico Endometriosis Hernia K. L. M. N. O. P. Q. R. S. T. Litiasis renal Adenitis mesentrica Trombosis de la arteria mesentrica Rotura de quiste ovrico Pancreatitis Enfermedad plvica inflamatoria Ulcera pptica Ulcera pptica perforada Pielonefritis Torsin de quiste ovrico

Para cada paciente con dolor abdominal, seleccione el diagnstico ms probable. 1. Una mujer de 25 aos presenta dolor persistente de inicio repentino e intensidad creciente en la fosa ilaca derecha. Tiene nuseas sin vmitos. Tuvo una defecacin normal poco antes de iniciarse el dolor. El examen fsico muestra una sensibilidad exquisita a la palpacin en esa zona con defensa muscular pero sin rebote. Ruidos intestinales presentes. En el examen de la regin pelviana se encontr una masa sensible de 7 cm en el lado derecho. El hematocrito es 32% y el recuento de leucocitos es de 18 000/mm3. La amilasa srica est en niveles normales. Un test de sangre oculta en las deposiciones result negativo. Respuesta: B Un hombre de 84 aos, residente de una casa de reposo, presenta un dolor poco focalizado en el hipogastrio, intermitente (cada 3-4 hrs), desde hace tres das. No presenta nuseas ni vmitos. No sabe cuando fue su ltima defecacin. En el examen fsico presenta un abdomen blando y una masa algo sensible en el cuadrante inferior izquierdo. El hematocrito es de 28%. El recuento de leucocitos es de 10 000/mm3. Amilasa srica en niveles normales y un examen de sangre oculta en deposiciones positivo. Respuesta: E

2.

86

Ejemplo de conducta mdica: Conducta a seguir


A. B. C. D. E. F. Observacin en el servicio de urgencia Ingresar a ciruga Ingresar para tratamiento mdico Ingresar para una endoscopia Ingresar para una laparoscopia Ordenar estudio de contraste G. H. I. J. K. Ordenar un examen de resonancia magntica Ordenar una tomografa axial computarizada Ordenar una ultrasonografa Alta a domicilio con analgsicos Alta y seguimiento con su mdico personal.

Para cada uno de los siguientes pacientes, seleccione la conducta ms apropiada a seguir.

Los temes podran describir pacientes con apendicitis, embarazo ectpico, endometriosis, enfermedad de Crohn, diverticulitis, absceso plvico, crisis de anemia de clulas falciformes, litiasis renal, torsin de quiste ovrico u otros problemas que comnmente se presentan como emergencias. Otras conductas mdicas podran enfocarse hacia consulta telefnica, decisiones acerca de transferir/dar de alta desde el hospital, etc.

Captulo 6. temes de Emparejamiento Ampliado (Tipo R)

87

Ejemplo de conducta mdica: ordenar exmenes de diagnstico


A. B. C. D. E. F. G. Test de sangre oculta en las deposiciones Glicemia en ayunas Nivel de hemoglobina Nivel de antgeno prosttico especfico Nivel de colesterol srico Nivel de hierro srico Exmenes de funcin tiroidea H. I. J. K. L. M. Test de tolerancia al ejercicio Examen prosttico digital ECG Espirometra Radiografa de trax Sigmoidoscopa

Para cada paciente que acude al mdico para un examen general de salud, seleccione el estudio diagnstico ms adecuado. 1. Hombre de 22 aos que pesa 89 kg (154 lbs) y mide 175 cm (69). Fuma un paquete de cigarrillos diarios desde hace 8 aos. No hace ejercicio. Su ltima evaluacin fue hace 5 aos. Su padre tuvo un infarto del miocardio a los 48 aos. El examen fsico es normal. Respuesta: E Mujer de 28 aos que pesa 70 kg (154 lbs) y mide 173 cm (68). Fuma un paquete de cigarrillos diarios desde hace 12 aos. No hace ejercicio. Su ltima evaluacin fue hace 5 aos, aunque se hizo un Papanicolau hace 9 meses y result normal. Su padre tuvo un infarto del miocardio a los 48 aos y su abuela un cncer de colon a los 62 aos. El examen fsico es normal. Respuesta: E

2.

Ejemplo de lista de opciones para anormalidades de electrolitos


A. B. C. D. Hipocalcemia Hipokalemia Hipomagnesemia Hiponatremia E. F. G. H. Hipercalcemia Hiperkalemia Hipermagnesemia Hipernatremia

Para cada uno de los siguientes pacientes, selecciones la anormalidad electroltica ms probable.

88

Ejemplo de lista de opciones para Pediatra y Ciencias de la Conducta


Habilidades cognitivas/lenguaje A. B. C. D. E. F. G. H. Normal Normal Normal Normal Retraso Retraso Retraso Retraso Habilidades motoras gruesas Normal Normal Retraso Retraso Normal Normal Retraso Retraso Habilidades sociales Normal Retraso Normal Retraso Normal Retraso Normal Retraso

Para cada nio seleccione la mejor descripcin de su desarrollo psicomotor

Ejemplo de listas de opciones para Nutricin/Bioqumica


A. B. C. D. E. F. G. H. Vitamina A Vitamina B1 (tiamina) Vitamina B2 (riboflavina) Vitamina B6 Vitamina C Vitamina D Vitamina E Vitamina K I. J. K. L. M. N. O. P. Folato Biotina Niacina Hierro Magnesio Cobre Zinc Yodo

Para cada nio con una anormalidad metablica, seleccione la vitamina o el mineral que est comprometido con mayor probabilidad. Ver el Apndice B para ejemplos adicionales.

Captulo 6. temes de Emparejamiento Ampliado (Tipo R)

89

Pasos para organizar un grupo para redactar preguntas de Emparejamiento Ampliado


Los siguientes pasos pueden usarse para guiar a grupos que escriban preguntas de contenido clnico de Emparejamiento Ampliado. En nuestra experiencia, algunos grupos se reunieron a la hora de comida siguiendo los pasos que se describen ms abajo y generaron en un primer borrador una docena de temes por lo menos por cada comensal. En otros casos, se program todo un da de trabajo en un sitio alejado del lugar de trabajo habitual, con el objeto de generar temes que estuvieran casi listos para ser usados. El organizador de la reunin 1 debe preocuparse de cmo se puede utilizar mejor el tiempo de cada participante. Si se hace un trabajo previo, se lograr un mayor rendimiento en trminos de cantidad y calidad. Por ejemplo, el rendimiento ser menor si se espera que los participantes generen los tpicos sobre los cules se redactarn las preguntas. Se ahorrar tiempo si se seleccionan los tpicos con antelacin a la reunin (Paso 1 ms abajo). En forma similar, se generarn ms temes si se proporciona un borrador de la lista de opciones para cada conjunto (Paso 3 ms abajo). Se generar un mayor nmero de temes si se proporciona a los asistentes un ejemplo de tem como modelo para cada conjunto (Paso 4 ms abajo). Hay situaciones en que es aconsejable dar la mayor flexibilidad posible a los redactores de temes en cuanto a determinar qu escribir. En estas circunstancias, podra ser mejor proporcionarles uno o ms conjuntos de opciones y luego dejarlos generar uno o ms conjuntos de opciones por su cuenta. Se debe tambin decidir acerca de la composicin de las parejas de redactores de temes. En la mayora de las veces es preferible dejar que los participantes decidan por s mismos acerca de la seleccin de sus compaeros de parejas 2 . En otras, sera mejor asignar las parejas. Los mejores resultados los hemos logrado juntando parejas con intereses similares, pero incluyendo individuos con experiencias ms diversas como revisores crticos. Si el objetivo es redactar temes interdisciplinarios para un examen, entonces una misma lista de opciones puede ser circulada a varias parejas de redactores. Por ejemplo, opciones relacionadas a dolor abdominal podra entregarse a parejas de gineclogos, a una pareja de cirujanos y a una pareja de internistas. Al construir el examen, se podra fusionar uno o ms temes de cada disciplina en un solo conjunto, as requiriendo que los examinandos piensen a travs de varias disciplinas al determinar los diagnsticos ms probables de los pacientes. Tambin se puede ahorrar mucho tiempo si los participantes redactan sus temes en computadores. Esto ahorra muchas horas pues se evita tener que descifrar la escritura de los redactores. Tpicamente, usamos a dos redactores por cada computador (hemos visto como automticamente uno de ellos se pone frente al teclado). Existen varias maneras de abordar la revisin de los temes. Los revisores pueden congregarse alrededor del computador para leer los temes, el autor puede leerlos en voz alta, o se puede imprimir copias para que todos las usen en la sesin de revisin.

1 2

[ENG: item writing party] [ENG: partners]

90

Defina el rea del contenido 1 del examen. Por ejemplo, al redactar un examen para evaluar la habilidad para diagnosticar problemas clnicos comunes, se podra definir esta rea 2 mediante una lista de motivos de consulta. Se ahorrar tiempo si se prepara de antemano esta lista antes de reunirse para redactar temes. Equimosis 3 Fatiga Fiebre Sangramiento gastrointestinal Cefalea Hematemesis/Melena Prurito Ictericia Artralgia

Masa abdominal Dolor abdominal Anemia Dolor en la espalda Dolor en el pecho Confusin Tos Retardo del crecimiento Diarrea Mareo

Adenopatas Anormalidades del movimiento Nusea Palpitaciones Disfuncin sexual Disnea Lesiones de la piel Sncope Cambios de peso Sibilancias

2.

Entrenar a un conjunto de docentes como redactores de temes. Este entrenamiento debe incluir una breve discusin acerca del propsito del examen, algunos ejemplos de temes, y los procedimientos a seguir cuando se redacten los temes. Dividir al grupo en parejas para redactar temes. A cada pareja se le asignan 2 a 4 motivos de consulta. Estas parejas generan (o modifican) una lista de diagnsticos para cada motivo de consulta que les fue asignado y redactan una o ms descripciones de pacientes para los diagnsticos que se incluyeron en la lista de opciones. Hay que esperar que produzcan entre 20 y 50 enunciados redactados por cada pareja (10 a 20 por cada motivo de consulta). A la larga, el uso de computadores ayuda a ahorrar un tiempo considerable.

3.

1 2

[ENG: content domain] [ENG: domain] 3 [ENG: Easy bruising]

Captulo 6. temes de Emparejamiento Ampliado (Tipo R)

91

4.

Enfatizar las siguientes pautas 1 para redactar enunciados. Cada tem debiera describir un paciente con uno de los diagnsticos en la lista de opciones. Se comienza con la edad del paciente, sexo, motivo de consulta, y sitio donde se realiza la atencin mdica, seguido de la historia personal y familiar (si es relevante), informacin acerca del examen fsico y datos de laboratorio (si se proporcionan). Segn sea el propsito del conjunto, las vietas pueden ser presentaciones prototpicas breves o descripciones detalladas que desafan a los examinados para identificar la informacin clave. Cada descripcin de pacientes debe ser similar en estructura a las otras del conjunto. Por ejemplo, si se incluye raza, origen tnico, u ocupacin en uno de los temes, incluya esta informacin en todos los temes. Si se incluyen exmenes de laboratorio, lo mismo.

5.

Junte las parejas en un grupo ms grande para revisar los temes. Una manera es que el autor lea el tem en voz alta y los otros tratan de identificar la respuesta correcta. El grupo revisa la lista de opciones y modifica el tem o la lista de opciones para eliminar cualquiera ambigedad. Otras maneras de realizar este proceso se han descritos anteriormente. Digite 2 en el computador, edite y someta los temes a una revisin externa. Los temes deben revisarse sin identificar la respuesta correcta y despus que estn listos en su forma final. Construir el examen. Seleccione una muestra de temes para cada motivo de consulta, guarde el resto de los temes para otros exmenes posteriores. Los temes pueden transformarse en los del tipo una mejor respuesta agregndoles una pregunta introductoria y las mejores 5 opciones (o ms) de la lista de opciones.

6.

7.

1 2

[ENG: guidelines] [ENG: Type]

92

Formulario para redactar preguntas de Emparejamiento Ampliado 1


Tema: _______________________________________________ (Por ejemplo, motivo de consulta) Declaracin introductoria: _______________________________ (Por ejemplo, Para cada paciente con fiebre, seleccione el diagnstico ms probable)

Opciones A B C D E F G H I J K L M N O P Q R S T U V W X Y Z

(Redacte los temes en pginas separadas)

[ENG: R-Sets]

Captulo 6. temes de Emparejamiento Ampliado (Tipo R)

93

Un ejemplo de cdigo para SPSSX para corregir exmenes 1 de seleccin mltiple, incluyendo los temes de Emparejamiento Ampliado
El siguiente cdigo para SPSSX puede usarse como modelo para corregir un examen hipottico de 100 preguntas de seleccin mltiple. Cada tem puede tener hasta 26 opciones pero solamente una opcin correcta. Se puede cambiar el cdigo para un examen de cualquier nmero de preguntas. Se presume que: las respuestas correctas se encuentran en un archivo llamado KEY.DAT (formato: una identificacin del examen de 8 caracteres, seguido de un espacio y luego de las 100 respuestas correctas. las respuestas del los examinados estn en un archivo llamado RESPONSE.DAT (formato: nmero de identificacin del examinado de 9 caracteres, un espacio, una identificacin del examen de 8 caracteres, un espacio, las respuestas de los examinados a los 100 temes). el resultado del anlisis de los temes muestra la distribucin de las respuestas para cada tem ms el coeficiente de confiabilidad (coeficiente alfa) y los informes del puntaje obtenido por cada examinando con su cdigo de identificacin personal, ms el porcentaje de respuestas correctas y los puntajes estandarizados (que estn en un archivo llamado REPORT.LIS). REPORT.LIS puede importarse a un programa de procesador de palabras para mejorar su apariencia. Para aquellos con experiencia en el uso de SPSSX el cdigo debiera ser fcil de comprender. SPSSX provee algunos recursos para manejo de archivos 2 que algo se parecen a aqullos de las relational data bases (ilustrados por los comandos AGGREGATE y MATCH FILES). Estos agregan la respuesta correcta al registro 3 de cada examinando, calculan el promedio y la desviacin estndar (SD) y luego agregan stos al registro de cada examinando. Es fcil adaptar este cdigo para usarlo en subpuntajes 4 , algoritmos ms complicados, etc.

1 2

[ENG: score] [ENG: file management] 3 [ENG: record] 4 [ENG: subscores]

94

Captulo 6. temes de Emparejamiento Ampliado (Tipo R)

95

Comparacin entre temes de formato de cinco opciones y los de Emparejamiento Ampliado


En varios trabajos destinados a investigar el nmero ptimo de opciones en temes de seleccin mltiple, hemos encontrado que, con otras variables constantes, es mejor tener ms opciones que menos. Basndonos en los temes usados en los exmenes preparados en la NBME, los temes de Emparejamiento Ampliado discriminaban mejor que cualquier otro formato. Lo seguan las preguntas de 5 opciones con una mejor respuesta y las diferentes formas de las preguntas de verdadero/falso eran las peores. En estudios controlados en que se compar en situaciones equivalentes temes de cinco opciones con los de Emparejamiento Ampliado, los de Emparejamiento Ampliado discriminaban mejor que los de 5 opciones. Se puede obtener niveles comparables de reproducibilidad usando un tercio menos temes que los de 5 opciones. Tambin se encontr que, cuando tenan contenidos similares, los temes de Emparejamiento Ampliado eran ms difciles que los de 5 opciones: existe una menor probabilidad de adivinar la respuesta correcta y los redactores de temes, al reducir las opciones a 5, no son capaces de elegir los distractores ms adecuados. La tabla siguiente muestra las respuestas de examinandos a un tem presentado en un formato de 5 opciones y en otro de 15 opciones. En su versin original tena 15 opciones y el autor redujo las opciones a las que crey eran las mejores 5 (B, F, G, J, N). El tem result mucho ms fcil en la versin de 5 distractores (p de 81 vs. 59) 1 y el ndice de discriminacin fue notablemente menor (ste no se muestra). Existe una mayor probabilidad que los examinandos adivinen la respuesta correcta en el formato de 5 opciones, especialmente porque los autores no identifican en forma uniforme los distractores ms destacados (por ejemplo, la opcin D). Los exmenes construidos mediante temes de emparejamiento ampliado tienden a separar a los estudiantes de habilidades ms bajas. La lista de opciones en el formato de Emparejamiento Ampliado les da una mejor oportunidad de demostrar lo que no saben.

Respuesta de los examinandos


Formato 5 opciones 15 opciones 2 A B 81 59 2 C D * 13 1 E F 4 2 G 0 1 0 1 H I J 14 7 2 2 K L M * 8 N 1 0 0 O

Para informacin adicional ver:


Case SM and Swanson DB, Ripkey DR. Comparison of items in five-option and extended-matching format for assessment of diagnostic skills. Academic Medicine. 1994;69(suppl): S1-S3. Case SM and Swanson DB. Extended-matching items: a practical alternative to free-response questions. Teaching and Learning in Medicine. 1993;5(2):107-115. Swanson DB, Case SM. Trends in written assessment: a strangely biased perspective. In: Harden R, Hart I, Mulholland H, eds. Approaches to Assessment of Clinical Competence. Norwich, England: Page Brothers; 1992:38-53.

p se usa para significar porcentaje de respuestas correctas.

96

Desde los A hasta las R y de vuelta1


Aunque las investigaciones realizadas han mostrado en forma consistente una ventaja psicomtrica en el uso del tipo R (Emparejamiento Ampliado) sobre el tipo de tem de cinco opciones, puede haber circunstancias en que usted necesite convertir temes de un formato a otro. Esto puede hacerse fcilmente. Por ejemplo, el siguiente tem se redact como uno de tipo A con cinco opciones. Un paciente con los rasgos fenotpicos de una trisoma (sndrome de Down) tiene 46 cromosomas en cada uno de 100 cariotipos en metafase. Cul es la explicacin ms probable de este hallazgo? A. B. C. Delecin Mosaicismo Mutacin somtica * D. E. Translocacin Trisoma no detectada

Esta pregunta puede transformarse fcilmente en la siguiente de Emparejamiento Ampliado. Una vez que se haya logrado el nuevo formato, puede agregarse nuevos enunciados para expandir el conjunto. A. B. C. Delecin Mosaicismo Mutacin somtica D. E. Translocacin Trisoma no detectada

Para cada paciente con anormalidades genticas, seleccione el patrn gentico que es ms probable que est involucrado. Un paciente con los rasgos fenotpicos de trisoma 21 (Sndrome de Down) presenta 46 cromosomas por cada 100 cariotipos en metafase.

Se puede agregar opciones adicionales para hacer el tem ms difcil A. B. C. D. E. Delecin Marca genmica 2 Mosaicismo Pleiotropa Penetrancia reducida F. G. H. I. Mutacin somtica Translocacin Mutacin no detectada Expresividad variable

Para cada paciente con anormalidades genticas, seleccione el patrn gentico que es ms probable que est involucrado.

Los temes de este conjunto pueden convertirse a los de tipo A con cinco o ms opciones.

1 2

[ENG: As to Rs and Back Again] [ENG: genomic imprinting]

Captulo 6. temes de Emparejamiento Ampliado (Tipo R)

97

98

Captulo 7 temes de Seleccione N1: Una extensin del formato de emparejamiento ampliado 2
El formato Seleccione N puede ser similar al formato tipo A o al de emparejamiento ampliado. La diferencia ms importante es que se le solicita al examinando que escoja 2, 3, 4 o an 5 de las opciones desde una lista. Tal como sucede en los conjuntos de emparejamiento ampliado, la lista de opciones puede contener hasta 26 opciones. Este formato se desarroll para reemplazar temes negativos o temes con dos opciones correctas, particularmente en reas tales como Promocin de la Salud 3 y Prevencin. Los temes podran referirse a varios pacientes con diferentes factores de riesgo y que llegan a la consulta para hacerse un examen de rutina. Se le pide al examinando que seleccione los exmenes de laboratorio o inmunizaciones que debera solicitar para cada paciente. Este formato tambin podra utilizarse en temes de manejo de situaciones de emergencia y donde varios procedimientos podran realizarse simultneamente. Se le pide al examinando que seleccione un nmero especfico de acciones desde una lista de opciones. Las reglas para redactar los temes son las mismas que para los conjuntos de emparejamiento ampliado. Las opciones deben ser breves (normalmente una sola palabra o una frase muy breve); las vietas de pacientes pueden ser ms largas. Si el conjunto pregunta por decisiones en el manejo de pacientes 4 , cada vieta debiera incluir toda la informacin pertinente de la historia clnica y del examen fsico. Tal como en los temes de emparejamiento ampliado el formato se presta para temes que parecen ser muy fciles. Debe evitarse el uso de vietas engaosas o demasiado difciles. El formato de temes de seleccione N est diseado para que se especifique exactamente el nmero de opciones a seleccionar. La base para esta decisin se deriva de la diferencia fundamental entre preguntas de verdadero/falso y las de una mejor respuesta. En las preguntas de verdadero/falso se le pide al examinando que indique todas las respuestas que sean apropiadas. En cambio, en el tipo de una mejor respuesta se le pide al examinando que indique un nmero especfico de respuestas. Al especificar el nmero de opciones a seleccionar se est cambiando la tarea desde una de verdadero/falso a una de mejor respuesta. Los resultados de las investigaciones han mostrado que la manera preferida de otorgar puntajes es mediante puntajes parciales 5 , pero consideraciones logsticas podran impedir su uso. Si se usa el otorgar puntajes mediante el sistema todo o nada 6 , los temes podran ser demasiado difciles y en este caso es mejor pedir que los examinandos elijan solamente dos o tres opciones en lugar de un nmero mayor. Para informacin adicional, ver tambin:
Ripkey DR, Case SM, Swanson DB. A new format for assessing. aspects of clinical competence. Academic Medicine. 1996:71(suppl):S34-S36.

1 2

[ENG: Pick N Items] [ENG: Extended matching] 3 [ENG: Health Maintenance] 4 [ENG: management decisions] 5 [ENG: partial scoring] 6 [ENG: all-or-nothing scoring]

Captulo 7. temes de Seleccione N: Una extensin del formato de emparejamiento ampliado

99

A. B. C. D. E.

Neuropata diabtica Enfermedad de Huntington Sndrome de Wallenberg Encefalopata por plomo Mduloblastoma

F. G. H. I. J.

Esclerosis mltiple Enfermedad de Parkinson Glioma pontino Tabes dorsalis Encefalopata de Wernicke

Un hombre de 50 aos presenta confusin mental, desorientacin y prdida de la memoria reciente que se han iniciado gradualmente. Un frotis de sangre muestra punteado basfilo de los eritrocitos. (Seleccione los dos diagnsticos ms probables)

En este ejemplo, habra desacuerdo acerca de cules diagnsticos seran probables, pero la tarea se aclara si se le solicita al examinando que seleccione los dos diagnsticos mas probables. Las opciones se podran diagramar de la siguiente manera. F C E Diagnstico menos probable D A G H B Diagnstico ms probable

Ejemplo de un conjunto de temes de seleccione N 1


A. B. C. D. E. F. Calcio Fluoruro Acido Flico Hierro Vitamina A Vitamina B1 (tiamina) G. H. I. J. K. Vitamina B6 Vitamina B12 (cianocobalamina) Vitamina C Vitamina D Vitamina E

Para cada nio, seleccione la vitamina o suplemento mineral ms apropiados: 1. Un lactante de 1 mes es trado para ser visto por mdico en un examen de rutina. El lactante ha recibido alimentacin al pecho exclusivamente y el examen es normal. (SELECCIONE DOS SUPLEMENTOS) Respuesta: B, J Una nia de 6 aos padece de fibrosis qustica. No toma medicamentos. (SELECCIONE TRES SUPLEMENTOS) Respuesta: E, J, K

2.

[ENG: Pick N Set]

100

Ejemplo de conjunto temes de seleccione N


A. B. C. D. E. Anlisis y cultivo de lquido cefalorraqudeo Hemocultivo Hemograma Leucocitos en deposiciones Electrolitos plasmticos F. G. H. I. Examen de orina completa Urocultivo Radiografa de abdomen simple Radiografa de trax

Para cada nio con fiebre, seleccione los estudios iniciales apropiados: 1. Una nia de 1 ao, previamente sana, es trada al servicio de urgencia por presentar fiebre de un da de evolucin. La temperatura es de 41 C (105.8 F). No presenta otros sntomas. El resto del examen fsico es normal. (SELECCIONE 4 ESTUDIOS). Respuesta: B, C, G, I Un recin nacido de 10 das, previamente sano, es trado al Servicio de Urgencia por presentar fiebre de dos horas de evolucin. Es producto de un embarazo de trmino y el parto no tuvo complicaciones. Su temperatura es de 39 C (102 F). El examen fsico es normal. (SELECCIONE 6 ESTUDIOS). Respuesta: A, B, C, E, G, I Un nio de 7 aos, portador de anemia de clulas falciformes, es trado al Servicio de Urgencia por presentar fiebre de un da de duracin y dolor en su pecho desde hace una hora. La temperatura es de 39.5 C (103.1 F). Los ruidos respiratorios estn ligeramente disminuidos en la base pulmonar derecha. No presenta dificultad respiratoria. (SELECCIONE TRES ESTUDIOS). Respuesta: B, C, I

2.

3.

Captulo 7. temes de Seleccione N: Una extensin del formato de emparejamiento ampliado

101

HISTORIA CLNICA

Antecedentes personales Sexo: Edad actual: Motivo de consulta: Antecedentes socioeconmicos: Estado civil: Ocupacin: Antecedentes mrbidos: Niez: Inmunizaciones: Chequeo mdico: Alergias: Antecedentes familiares: Padres: Hermanos: Hijos: Hbitos: Alcohol: Tabaco: Ejercicio: Medicamentos:

masculino 28 aos examen de salud soltero programador de computadores obesidad desde que ingres a la escuela todas las de la niez; toxoide tetnico a los 15 aos. Sin otras inmunizaciones. no desde la universidad. no padre, 57 aos, hipertenso, madre, 55 aos, obesa e hipertensa. No No 2-4 cervezas en el fin de semana 5-10 cigarrillos al da entre los 16 y 24 aos. sedentario ninguno

Examen fsico Estatura: Peso: Signos vitales: Presin arterial: Pulso: Respiraciones: Piel: Abdomen: Resto del examen:

178 cm (70) 134 kg (295 lb.) 148/86 mmHg 90/min 16/min eritema en la regin inguinal abundante panculo adiposo dentro de lmites normales

102

1.

Para el paciente del cul se mostr su historia clnica, seleccione las condiciones patolgicas que tiene mayor riesgo de presentar. (SELECCIONE 4 CONDICIONES) A. B. C. D. E. F. Alcoholismo Cncer del colon Enfermedad arterial coronaria Diabetes mellitus Hemocromatosis HIV G. H. I. J. K. L. Hipertensin Hipotiroidismo Osteoartritis Cncer de piel Cncer tiroideo Infeccin de vas urinarias

2. Para el paciente del cual se mostr su historia clnica, seleccione las medidas ms apropiadas para el cuidado de su salud. (SELECCIONE 4 MEDIDAS). A. B. C. D. E. F. G. Hemograma Consejos dietticos Orientacin sobre ejercicio Perfil lipdico en ayunas Inmunizacin para Hepatitis B Serologa para HIV Inmunizacin para influenza H. I. J. K. L. Nitrgeno ureico y creatinina plasmtica. Vacuna de toxoide tetnico Tests de funcin tiroidea Examen de orina Radiografa de trax

Captulo 7. temes de Seleccione N: Una extensin del formato de emparejamiento ampliado

103

104

Seccin IV Temas Adicionales

Esta seccin incluye temas adicionales relacionados con la evaluacin

105

106

Captulo 8 Interpretacin de los resultados del anlisis de temes1


En muchas escuelas, los profesores reciben un informe con los resultados del anlisis de temes despus de un examen con preguntas de seleccin mltiple. Este informe es una excelente fuente de informacin tanto para evaluar la calidad de cada tem como para confirmar si se us la clave correcta. A continuacin se presentan como ejemplos los resultados de cuatro temes que ilustran situaciones frecuentes. Para realizar el anlisis, se dividi a los alumnos que rindieron el examen en dos grupos, uno Alto y otro Bajo, segn los puntajes obtenidos en el examen. Si el nmero de alumnos es pequeo, se recomienda seleccionar el 50% de los puntajes superiores para el grupo Alto y el resto para el grupo Bajo. Si el nmero de alumnos que rinden el examen es grande, entonces se recomienda usar el 25% superior y 25% inferior de los puntajes para los grupos Alto y Bajo respectivamente. Generalmente, el resultado de anlisis de los temes muestra el porcentaje de alumnos que seleccionaron cada opcin. A menudo se incluye tambin a) una medida de la dificultad del tem (por ejemplo, el valor de p, o sea la proporcin de alumnos que respondieron correctamente el tem, y b) una medida de la discriminacin (por ejemplo, el biserial o punto biserial 2 ). Recomendamos fijarse ms en el patrn de respuestas 3 que en el nivel de dificultad o el ndice de discriminacin.

1 2

[ENG: item analysis] [ENG: biserial or point biserial] 3 [ENG: pattern of responses]

Captulo 8. Interpretacin de los resultados del anlisis de temes

107

En cada ejemplo de tem que se presenta a continuacin se muestra el porcentaje de estudiantes que seleccionaron cada opcin. La fila que incluye Total muestra el porcentaje de estudiantes de todo el grupo que seleccionaron cada opcin. Por ejemplo, en el tem #1, el 1% del grupo Alto eligi la opcin A; un 1% eligi la B, un 91% eligi la C, un 4% eligi la D, un 1% eligi la E y un 2% la F. Tambin, el 20% del grupo Bajo eligi la opcin A; 6% la B, etc. El asterisco en la opcin B significa que es la respuesta correcta. tem #1 Grupo Alto Bajo Total A 1 20 9 B* 1 6 2 C 91 51 76 D 4 14 8 E 1 6 3 F 2 3 2

Valor de p (ndice de dificultad): 2

ndice de discriminacin: -0.21

Interpretacin: Este es un tpico ejemplo cuando se comete un error en determinar la clave (respuesta correcta). Si la respuesta correcta fuese B, el tem sera muy difcil (solamente el 2% respondi esta opcin) y el ndice de discriminacin sera negativo. Es muy probable que la respuesta correcta sea C. Se recomienda que un experto en el contenido del examen lo revise para dilucidar esta situacin. Si la respuesta correcta fuese C, el valor de p sera 76 con un ndice de discriminacin de 0.46. Si la respuesta correcta es C, los datos estadsticos del tem son excelentes y no es necesario realizar modificaciones.

tem #2 Grupo Alto Bajo Total A 0 0 0 B 1 1 1 C* 90 60 74 D 3 25 12 E 3 8 7 F 3 6 6

Valor de p (ndice de dificultad): 74

ndice de discriminacin: 0.33

Interpretacin: El 90% del grupo Alto y el 60% del grupo Bajo eligi la respuesta correcta. Estos datos estadsticos son excelentes. Se podran modificar las opciones A y B ya que muy pocos alumnos las seleccionaron.

108

tem #3 Grupo Alto Bajo Total A 44 20 32 B 1 15 7 C* 50 21 34 D 2 22 14 E 1 20 11 F 2 2 2

Valor de p (ndice de dificultad): 34

ndice de discriminacin: 0.30

Interpretacin: El 50% del grupo Alto y el 21% del grupo Bajo seleccion la respuesta correcta. Este tem es muy difcil y es muy probable que no sea apropiado. Un nmero alto eligi la opcin A. Debera revisarse la opcin A para ver si no es igualmente correcta.

tem #4 Grupo Alto Bajo Total A 18 24 22 B 10 24 17 C* 51 21 34 D 17 25 22 E 2 4 3 F 2 2 2

Valor de p (ndice de dificultad): 34

ndice de discriminacin: 0.30

Interpretacin: La distribucin del grupo Alto y la del grupo Bajo es idntica a la del tem #3, en cuanto a la respuesta correcta. A diferencia del tem # 3, en este tem las respuestas errneas de los grupos Alto y Bajo se distribuyen ampliamente entre las opciones incorrectas. De todas maneras sera aconsejable revisar las opciones A, B, y D para ver si estn redactadas correctamente.

Captulo 8. Interpretacin de los resultados del anlisis de temes

109

110

Captulo 9 Como establecer un estndar para aprobar o reprobar1


Definiciones y principios bsicos
Los estndares pueden clasificarse ya sea como relativos o absolutos. Un estndar relativo se basa en el desempeo de todo el grupo que rinde un examen. Los examinandos aprueban o reprueban dependiendo de cun bien se desempean en relacin a los otros examinandos que rinden el examen. Los siguientes son ejemplos de estndares relativos: Aquellos estudiantes con puntajes menores a 1.2 desviaciones estndares bajo el promedio reprobarn el examen. El 20% de los puntajes ms bajos reprobarn el examen. En cambio, un estndar absoluto no compara el desempeo de un examinando con otros que rinden el examen. Los examinandos aprueban o reprueban el examen basado solamente en su propio desempeo, sin considerar el desempeo de los otros que rinden el examen. Es posible que todos los examinandos reprueben o aprueben el examen. El siguiente es un ejemplo de estndar absoluto: Los que respondan correctamente menos del 60% de las preguntas del examen sern reprobados. A menos que existan buenas razones para reprobar un nmero determinado de examinandos, se prefiere un estndar absoluto (basado en el desempeo del examinando) sobre uno relativo (basado en una tasa determinada de reprobacin).

Principios bsicos de la fijacin de estndares2


Independientemente del procedimiento usado, el fijar estndares requiere criterio 3 . Fijar estndares siempre ser arbitrario, pero no debe ser caprichoso. A menos que exista una razn especfica para reprobar un nmero determinado de examinandos (por ejemplo, si solamente existe un cupo limitado disponible o en procesos de seleccin), un estndar basado en que el examinando muestre dominio sobre la materia en el examen es preferible a un estndar basado en una tasa predeterminada de reprobacin.

1 2

[ENG: Establishing a Pass/Fail Standard] [ENG: setting standards] 3 [ENG: judgement]

Captulo 9. Como establecer un estndar para aprobar o reprobar

111

Es conveniente usar mltiples jueces bien informados en el proceso de establecer estndares. Aunque se producirn diferencias de opiniones, stas se mitigarn si se usan varios jueces para reducir las diferencias entre los jueces severos y los benvolos 1 . En algn momento se debe proveer a los jueces de informacin acerca del desempeo de los examinandos. El fijar estndares sin esa informacin podr resultar en estndares no razonables y poco informados 2 .

Una referencia til y prctica para la fijacin de estndares es:


Livingston SA, Zicky MJ. Passing scores: A Manual for Setting Standards of Performance on Educational and Occupational Tests. Princeton, NJ: Educational Testing Service; 1982.

Dos mtodos para establecer estndares basados en juicios 3 acerca de los temes.
Procedimiento de Ebel modificado
Un grupo de jueces discute las caractersticas del examinando que tiene la habilidad mnima para aprobar 4 . Los jueces clasifican los temes en Esencial, Importante, o Indicado 5 . Los jueces mencionan el nmero de temes en cada categora que debera obtener un examinando que tiene la habilidad mnima para aprobar. El estndar para aprobar/reprobar se calcula como el porcentaje de puntos posibles que debiera obtener un examinando con la habilidad mnima para aprobar.

1 2

[ENG: hawk/dove effects] [ENG: uninformed standards and unreasonable results] 3 [ENG: judgement] 4 [ENG: borderline examinee] 5 [ENG: indicated]

112

Procedimiento de Angoff modificado


Un grupo jueces discute las caractersticas del examinando con habilidad mnima para aprobar. Para cada tem del examen, los jueces estiman el porcentaje de examinandos con habilidad mnima para aprobar que responderan la pregunta correctamente. El estndar para aprobar/reprobar el examen es el promedio de los porcentajes de todos los temes.

Variaciones comunes al procedimiento de Angoff


Los jueces pueden o no recibir las respuestas correctas a las preguntas. Los jueces pueden o no recibir el porcentaje de examinandos que respondieron correctamente la pregunta. Despus de un perodo de entrenamiento los jueces pueden continuar ya sea solos o en grupos.

Captulo 9. Como establecer un estndar para aprobar o reprobar

113

Estndares Relativos/Absolutos de Acuerdo: El mtodo de Hofstee1


ltimamente se han desarrollado varios modelos de acuerdo 2 que utilizan las ventajas tanto de los procedimientos de fijacin de estndares relativos como de los absolutos. Uno de los mtodos es el mtodo de Hofstee, aqu descrito: 1. 2. Se les pide a los jueces que examinen una copia del examen. Los jueces indican los siguientes valores que sirven para definir estndares aceptables: El porcentaje mnimo aceptable de examinandos reprobados (tasa mnima de reprobados) 3 . El porcentaje mximo aceptable de examinandos reprobados (tasa mxima de reprobados) 4 . El puntaje mnimo que permitir ser aprobado (puntaje mnimo de aprobacin) 5 . El puntaje mximo que permitir ser aprobado (puntaje mximo de aprobacin) 6 . Despus del examen, se traza una curva mostrando la tasa de reprobacin en funcin del puntaje de aprobacin. (En la figura, la curva se extiende desde el extremo inferior izquierdo al extremo superior derecho). Se trazan los cuatro valores que se obtuvieron en el #2, formando un rectngulo. A menudo se usan los valores de la mediana del grupo. En el ejemplo, la tasa apropiada de reprobacin se juzg que debiera estar entre 0 a 20% (ver lneas horizontales). El puntaje para aprobar/reprobar se juzg debiera estar entre 50 y 60% de respuesta correctas (ver lneas verticales).

3.

4.

5.

Se dibuja una lnea diagonal entre el vrtice superior izquierdo al vrtice inferior derecho del cuadrante. El estndar es el punto de interseccin con la curva (en este caso un poco ms de 55%). Una buena referencia sobre mtodos de acuerdo es:
De Gruijter D. Compromise models for establishing examination standards. Journal of Educational Measurement. 1985; 22:263-269.

1 2

[ENG: Relative/Absolute Compromise Standards: The Hofstee Method] [ENG: compromise models] 3 [ENG: minimum failure rate] 4 [ENG: maximum failure rate] 5 [ENG: minimum passing point] 6 [ENG: maximum passing point]

114

Captulo 10 Comentarios diversos acerca de tpicos de evaluacin


A continuacin presentamos una variedad de comentarios relacionados con la evaluacin. En general los argumentos son especulativos, basados ms bien en experiencias personales que en evidencias obtenidas de resultados de investigaciones. En otras palabras, ms bien reflejan nuestros prejuicios que los resultados de investigaciones. Exmenes con mltiples estaciones 1 (exmenes prcticos, Gymkhanas o carreras de obstculos 2 , OSCEs 3 ) Aunque son difciles de producir e implementar, son muy tiles en las ciencias bsicas, particularmente para evaluar destrezas 4 que no pueden evaluarse mediante exmenes escritos (por ejemplo, habilidad para usar un microscopio, para demostrar un procedimiento de laboratorio). Adems, la reproduccin de ciertos materiales (por ejemplo, radiografas, material pictrico en color) puede ser de alto costo. En estos casos se justifica usar los mtodos de estaciones mltiples para reducir los costos administrativos. Exmenes para llevar a la casa 5 Los exmenes para llevar a la casa pueden constituir una experiencia importante para los estudiantes al estimularlos para la lectura en profundidad acerca de un determinado tpico importante. Desgraciadamente, los estudiantes tienden a producir verdaderos volmenes de material y adems no se est seguro que ellos hayan producido realmente el material entregado. Se puede lograr resultados semejantes mediante la distribucin de un superconjunto 6 de preguntas por adelantado y luego administrarles un subconjunto de preguntas en un examen con tiempo controlado 7 . Exmenes con libro abierto 8 Este tipo de exmenes puede ser til para algunos tipos de preguntas diseadas por los profesores. En este caso no tiene sentido preguntar acerca de datos aislados que pueden encontrarse fcilmente en una pgina de un libro de texto. Por lo tanto, las preguntas que se utilizan para este tipo de examen tienden a enfocarse en la comprensin de conceptos importantes y en principios para aplicar en situaciones problemas.

1 2

[ENG: Multiple Station Exams] [ENG: Steeplechases] 3 OSCE= Objective Structured Clinical Examination 4 [ENG: hands-on skills] 5 [ENG: take-home exams] 6 [ENG: superset] 7 [ENG: timed test] 8 [ENG: Open-Book tests]

Captulo 10. Comentarios diversos acerca de tpicos de evaluacin

115

Exmenes cortos y frecuentes versus exmenes poco frecuentes 1 Cuando los exmenes son poco frecuentes, cada evento es de alta importancia. A veces los estudiantes incluso no van a clases para prepararse para el prximo examen. Se considera que esto no es deseable. En el caso de exmenes cortos y frecuentes, es difcil saber si los estudiantes estn estudiando en profundidad el contenido adecuado. Aunque les exige mucho tiempo a los profesores, los exmenes frecuentes reducen la importancia de cada evento y ayuda a los estudiantes a juzgar mejor su progreso. En general, parece preferible realizar exmenes frecuentes, aunque de una u otra forma los estudiantes no van a estar contentos. Mantener los exmenes bajo seguridad o dejarlos en manos de los estudiantes 2 Los exmenes pueden guiar el aprendizaje de los estudiantes y por lo tanto si se permite que los estudiantes se queden con las preguntas despus de un examen, esto servir para reforzar los objetivos del curso (si los exmenes reflejan stos). Sin embargo, el preparar buenas preguntas para un examen toma mucho tiempo. Por lo tanto, con el pasar del tiempo, la calidad de las preguntas puede ir deteriorndose si los profesores deben preparar preguntas nuevas cada vez que se realiza el curso. Es mejor enfrentar este dilema entregando a los estudiantes una muestra de preguntas bien confeccionadas para guiar el aprendizaje y al mismo tiempo mantener un banco de preguntas bajo condiciones de seguridad para ser usado repetidamente. Debe tomarse en cuenta que es difcil mantener la seguridad total ya que los estudiantes generalmente memorizan las preguntas y luego las reproducen para ser usadas por sus compaeros. Uso de exmenes acumulativos 3 El uso de tests acumulativos significa producir exmenes de todo el contenido del curso hasta la fecha del examen. De esta manera se estimula el lograr una interrelacin entre los contenidos estudiados, particularmente si se hace esto entre tpicos abordados hace un tiempo con los ms frescos. El uso de exmenes que cubren slo el contenido abordado desde el examen anterior tiende a estimular el estudio de tpicos en forma aislada, lo que podra resultar en una prdida de la comprensin de las relaciones entre los tpicos. Por cuanto los estudiantes pueden no lograr un buen puntaje en una serie de exmenes porque no dominaron los conceptos bsicos de la materia, los exmenes acumulativos pueden motivarlos para corregir sus deficiencias. Uso de exmenes integradores a travs de varios cursos 4 Tal como los exmenes acumulativos, los exmenes integradores usan material de varios cursos permitiendo a los estudiantes ver las interrelaciones entre disciplinas y tpicos. Esto debera ayudar a la retencin de conceptos por largo tiempo y a la aplicacin de conceptos de ciencias bsicas a situaciones clnicas. En general, se requiere que tanto profesores de ciencias bsicas como de clnicas trabajen en conjunto para este tipo de exmenes. Aunque esta manera de producir exmenes exige tiempo, el esfuerzo realizado en conjunto puede resultar en mejores preguntas y adems lograr discusiones tiles entre los profesores acerca de qu materia debera incluirse en el programa de estudios.

1 2

[ENG: Frequent short quizzes versus infrequent tests] [ENG: Keeping tests secure versus permitting students to retain them] 3 [ENG: cumulative tests] 4 [ENG: Use of Integrative, Cross-Course Tests]

116

Apndice A El cementerio de los formatos de temes del NBME

117

118

Apndice A El cementerio de los formatos de temes del NBME


El primer examen realizado por el National Board se administr en 1916. Tuvo una duracin de cinco das y consisti en pruebas escritas, orales, de laboratorio y de clnica. Despus de la Primera Guerra Mundial, el National Board design una comisin para visitar Inglaterra, Escocia y Francia para estudiar mtodos de evaluacin utilizados en los exmenes para autorizar la prctica mdica. En 1922, el National Board administr un nuevo examen. La Parte I consisti en tres das de examen de ensayo en ciencias bsicas. La Parte II fue dos das de examen de ensayo en las principales ciencias clnicas. La Parte III fue un da de examen oral al lado del enfermo. Los exmenes continuaron de esa forma hasta la dcada del 50 cuando se hicieron populares los tests de seleccin mltiple. En 1951, el National Board, con la colaboracin de Educational Testing Services, desarroll un estudio de tres aos de duracin para comparar las pruebas de ensayo con las de seleccin mltiple. Las Partes I y II fueron convertidas a preguntas de seleccin mltiple en 1953. La Parte III fue revisada durante los primeros aos de la dcada de los 60, donde se introdujeron nuevos formatos y se eliminaron otros. temes Tipo A El primer examen consisti fundamentalmente en De las siguientes, el agente profilctico ms efectivo temes del tipo A involucrando recuerdo de datos, para la prevencin de las recurrencias de la fiebre como el siguiente ejemplo (al lado). reumtica es: Cuarenta aos despus de la primera aplicacin del A. cido acetilsaliclico examen con temes de seleccin mltiple, el B. acido para-aminobenzoico National Board ha ampliado el alcance de los temes C. hormona adrenocorticotrpica tipo A para evaluar habilidades de razonamiento y D. cortisona resolucin de problemas mediante la inclusin de E. sulfadiazina vietas clnicas en los enunciados de los temes. Actualmente, el tipo A sigue siendo el formato de tem ms comnmente usado en los exmenes. Muchos otros formatos de temes que fueron desarrollados durante este perodo han sido eliminados. Estos formatos (llamados por una letra de acuerdo al orden en su origen) se describen en las pginas siguientes en el llamado cementerio de formatos de temes. Los temes mostrados en esta seccin fueron obtenidas de:
Hubbard JP, Clemens WV. Multiple-Choice Examinations in Medicine. Philadelphia PA: Lea & Febiger; 1961.

Crticas a estos formatos fueron obtenidas de:


Hubbard JP. Measuring Medical Education. Philadelphia PA: Lea & Febiger; 1971. Hubbard JP. Levit EJ. The National Board of Medical Examiners: The First Seventy Years. NBME; 1985.

Y tambin de varios artculos de Morton publicados en Federation Bulletin en 1985 y 1986. Apndice A 119

temes Tipo B Los temes tipo B eran temes de emparejamiento (o de trminos pareados) que consistan en una lista de encabezamientos identificados por una letra seguida de una lista de palabras o frases identificados por nmeros. Se instrua al examinando que seleccionara el encabezamiento que estaba mejor asociado con cada palabra o frase. Como cada respuesta poda ser usada ms de una vez o ninguna vez, este tipo de pregunta no poda ser respondido mediante eliminacin. Los temes tipo B fueron creados para ampliar el alcance de un examen de preguntas de seleccin mltiple permitiendo evaluar varios temas relacionados en una sola serie de preguntas. A diferencia de los formatos de emparejamiento usados hoy, los temes de tipo B no incluan una declaracin introductoria; por consiguiente las preguntas resultaban a veces confusas. Aunque se obtenan buenos resultados usando este tipo de temes, su uso ha sido recientemente eliminado en la medida que el formato de emparejamiento ampliado 1 se ha utilizado ms extensamente (ver captulo 6).

Ejemplo de tem Tipo B


INSTRUCCIONES: Cada uno de los conjuntos de temes de esta seccin consiste en una lista de 3 a 5 opciones identificadas con una letra (algunas de las opciones pueden ser grficos o figuras) seguidas de una serie de preguntas numeradas. Para cada una de los temes numerados, seleccione LA opcin identificada con letra que est ms directamente asociada con ella y llene el crculo correspondiente a la letra en la hoja de respuestas. Cada opcin identificada con una letra puede ser seleccionada una vez, ms de una vez o ninguna vez. A. B. C. D. E. 1. 2. 3. 4. 5. Coartacin de la aorta Ductus arterioso persistente Tetraloga de Fallot Anillo vascular Atresia de la tricspide Se beneficiara con una anastomosis pulmonar-sistmica La forma ms comn de cardiopata congnita ciantica Se corrige quirrgicamente mediante reseccin y anastomosis trmino terminal Posible causa de disfagia en lactantes y preescolares Hipertensin en los brazos e hipotensin en las piernas

[ENG: extended matching]

120

temes Tipo D Los temes tipo D eran temes complejos de emparejamiento (o de trminos pareados) en que cada tem consista de tres trastornos funcionales (identificados con una letra) y cinco situaciones (en una lista numerada). Se solicitaba al examinando 1) seleccionar el trastorno funcional o categora en que se relacionaba con cuatro de las cinco situaciones y 2) indicar la nica situacin que no perteneca a esta categora. Se estimaba que estos temes requeran un conocimiento discriminatorio de un nmero similar de factores. Sin embargo las preguntas de tipo D eran difciles de escribir y las instrucciones eran confusas. Adems, no discriminaban entre los estudiantes que saban y los que no saban.

Ejemplo de tem Tipo D


INSTRUCCIONES: Hay dos respuestas a cada una de las siguientes preguntas. En el lado izquierdo hay tres categoras identificadas con letras. Exactamente 4 de los 5 aspectos sealados en la lista del lado derecho estn relacionadas de alguna forma con UNA de esas categoras. (1) En la lnea correspondiente de la hoja de respuestas ennegrezca el espacio bajo la letra de la categora a la cul pertenecen estos cuatro temes. (2) Luego ennegrezca el espacio bajo el nmero del tem de la derecha que NO pertenece a la misma categora que los otros cuatro. A. Eosinofilia de importancia diagnstica B. Plasmacitosis de importancia diagnstica C. Linfocitosis de importancia diagnstica 1. 2. 3. 4. 5. Triquinosis Mieloma mltiple Sindrome de Lffler Enfermedad de Hodgkin Esquistosomiasis

Apndice A

121

temes Tipo K Los temes tipo K eran los temes de formato verdadero/falso mltiple ms usados por el National Board. Consistan en un enunciado seguido de cuatro opciones, una o ms de las cuales poda ser correcta. Se pensaba que los temes tipo K evaluaban en mayor profundidad el conocimiento o la comprensin de diferentes aspectos de una enfermedad, un proceso, o un procedimiento y requeran que el examinando estuviera familiarizado con diferentes aspectos acerca de un tpico dado. Sin embargo, los temes tipo K fueron criticados como muy complicados, requiriendo del examinando tener constantemente el cdigo de respuesta presente. Adems, la combinacin de posibles respuestas introduca pistas al contestarlas, lo que disminua la validez y la confiabilidad del test. Era difcil de escribir buenas preguntas, sin ambigedad, de verdadero/falso. Debido a que estos temes requeran solamente que se usaran hechos absolutamente falsos o verdaderos, los temes tipo K no podan ser usados para evaluar el juicio clnico, con excepcin de comparaciones (por ej. "La droga X es mejor que la droga Y para el tratamiento de la enfermedad K"). Los temes tipo K eran ms difciles y discriminaban menos que otros tipos de preguntas. Adems, eran menos eficientes que otros formatos de temes de seleccin mltiple y la confiabilidad relativa por unidad de tiempo de una prueba era menor.

Ejemplo de tem Tipo K


Instrucciones resumidas A Slo 1, 2 y 3 B Slo 1 y 3 C Slo 2 y 4 D Slo 4 E Todas son correctas

1.

Un nio que sufre una exacerbacin aguda de su fiebre reumtica generalmente tiene: (1) una eritrosedimentacin elevada (2) un intervalo PR prolongado (3) un ttulo elevado de antiestreptolisina O (4) ndulos subcutneos

122

temes Tipo C Los temes tipo C eran similares a las temes tipo B en apariencia, pero en la tarea que se solicitaba al examinando stos se comportaban como temes mltiples de verdadero/falso. Las preguntas tipo C consistan en una lista de encabezamientos identificados con una letra seguidos de una lista de palabras o frases numeradas. Para cada tem identificado con un nmero, los examinandos deban decidir si la opcin A era verdadera, si B era verdadera, si ambas eran verdaderas (opcin C), o si ninguna era verdadera (opcin D). Se usaba este tipo de tem para comparar y contrastar dos enfermedades, signos y sntomas, hallazgos de laboratorio, etc. Los temes tipo C son iguales a las tipo K en su nivel de dificultad. El problema principal con los temes de tipo C era decidir en qu medida algo deba ser verdadero para ser seleccionado. Si, por ejemplo, algo estaba asociado con A y B, pero estaba ms fuertemente relacionado con A, el examinando deba decidir si la respuesta correcta era si slo A o si ambos, A y B, eran verdaderos. Con asociaciones relativamente dbiles, los examinandos deban decidir si la asociacin era lo suficientemente fuerte como para tomarla en cuenta, o si ninguna es verdadera era la opcin correcta. Estos juicios no estaban relacionados con el conocimiento mdico, pero en cambio forzaban al examinando a adivinar acerca de las intenciones de quienes escribieron los temes.

Ejemplo de tem tipo C


INSTRUCCIONES: Cada conjunto de preguntas de emparejamiento (o de trminos pareados) consiste en una lista de cuatro opciones identificadas con una letra seguida de varios temes identificados con un nmero. Para cada tem identificado con un nmero, seleccione UNA opcin identificada con una letra que est ms directamente asociada con ella y llene el crculo que contiene la letra en la hoja de respuestas. Cada opcin identificada con una letra puede ser seleccionada una vez, ms de una vez o ninguna vez. A. B. C. D. 1. 2. 3. 4. Malaria por Plasmodium vivax Malaria por Plasmodium falciparum Ambas Ninguna

Una combinacin de primaquina y cloroquina es el tratamiento de eleccin para el ataque agudo Los ataques clnicos se suprimen mediante la ingestin de cloroquina una vez a la semana mientras se est en el rea endmica Se sana permanentemente mediante el tratamiento con cloroquina La infeccin se previene con la ingestin de cloroquina una vez a la semana

Apndice A

123

temes Tipo E Las temes tipo E eran preguntas de mltiple verdadero/falso que estn basadas en el anlisis de relaciones. Los alumnos que conocieron los temes de tipo E se refieren a ellos como las preguntas de Verdadero, Verdadero y No relacionado. Los temes tipo E consisten en una frase con dos partes principales: una afirmacin y una razn para esa afirmacin. El examinando deba seleccionar A si la afirmacin y la razn eran verdaderas y la razn era una explicacin correcta para la afirmacin; B si tanto la afirmacin como la razn eran verdaderas, pero la razn no era una explicacin correcta para la afirmacin; C, si la afirmacin era verdadera pero la razn era una aseveracin falsa; D, si la afirmacin era falsa pero la razn era un enunciado verdadero; E, si tanto la afirmacin como la razn eran aseveraciones falsas. Se pensaba que se requera de buenas habilidades de razonamiento y de entender principios bsicos para responder el tem correctamente. Sin embargo, los temes tipo E eran difciles de construir y los examinandos los encontraban confusos.

Ejemplo de tem Tipo E


Instrucciones resumidas A B C D E Verdadero Verdadero Verdadero Falso Falso Verdadero Verdadero Falso Verdadero Falso La razn es una explicacin correcta La razn NO es una explicacin correcta

Afirmacin 1. Al Herpes simple se le considera generalmente como una infeccin autgena PORQUE

Razn los pacientes que reciben terapia mediante fiebre 1 frecuentemente desarrollan herpes. la leche de vaca tiene un mayor contenido de calcio.

2. La leche de vaca es preferible a la leche materna en la alimentacin infantil

PORQUE

[ENG: fever therapy]

124

temes Tipo H Los temes tipo H eran temes de comparacin que consistan en pares de aseveraciones describiendo dos entidades a ser comparadas en un sentido cuantitativo. El examinando deba seleccionar A, si A era mayor que B; B si B era mayor que A; y C si las dos eran aproximadamente iguales. Aunque generalmente se aceptaba que las preguntas que dependen de la memorizacin de cantidades cuantitativas absolutas deben ser limitadas en su nmero, los temes tipo H se consideraban tiles en esas instancias donde se pensaba que el recuerdo de informacin cuantitativa era importante. El problema para los examinandos estaba en decidir cun grande deba ser la diferencia para ser considerada importante para la respuesta.

Ejemplo de tem Tipo H


INSTRUCCIONES: Los siguientes pares de frases describen dos entidades que deben ser comparadas cuantitativamente. En la lnea correspondiente de la hoja de respuestas ennegrezca el espacio bajo A. si (A) es mayor que (B) B. si (B) es mayor que (A) C. si las dos son iguales o son aproximadamente iguales

1. (A) (B)

La dosis teraputica usual de epinefrina La dosis teraputica usual de efedrina

2. (A) (B)

Expectativa de vida en un paciente con un glioblastoma del lbulo occipital Expectativa de vida en un paciente con un glioblastoma del lbulo frontal

Apndice A

125

temes Tipo I Los temes tipo I eran similares a los temes tipo H. Consistan en pares de frases que describen condiciones o cantidades que podran variar entre s. Se instrua al examinando que deba seleccionar A si las dos frases se relacionaban directamente (ej. Un aumento en la primera se acompaaba de un aumento en la segunda o una disminucin de la primera iba acompaada de una disminucin de la segunda); B si las frases se relacionaban en forma inversa (ej. Un aumento en la primera iba acompaada de una disminucin en la segunda o si una disminucin en la primera iba acompaada de un aumento en la segunda); o C si los cambios eran independientes uno de otro.

Ejemplo de tem Tipo I


INSTRUCCIONES: Cada uno de los siguientes pares de frases describe condiciones o cantidades que pueden o no estar relacionadas. En la hoja de respuesta ennegrezca en la lnea correspondiente bajo: A si el aumento en la primera est acompaado por un aumento en la segunda o si la disminucin en la primera est acompaado de una disminucin en la segunda. B si el aumento en la primera est acompaado por una disminucin en la segunda o si la disminucin de la primera est acompaado de un aumento en la segunda. C si los cambios en la primera no necesariamente estn acompaados de cambios en la segunda. 1. (A) (B) 2. (A) (B) Volumen urinario Gravedad especfica de la orina Concentracin de protena en el plasma Presin osmtica coloide del plasma

Ni los formatos tipo H ni los I fueron particularmente populares. Existan mayores posibilidades de adivinar la respuesta correcta debido a que las opciones eran menores que en otros tipos de temes. Adems, los temes tendan a centrarse en detalles de menor importancia en lugar de conceptos cientficos.

126

En sus series del Federation Bulletin, Morton (1985-1986) seal que l crea que los diversos tipos de temes fueron incluidos en exmenes para autorizar la prctica como mdico simplemente para agregar variedad a un examen muy largo. El NBME revis los resultados de las investigaciones que se haban realizado en los distintos tipos de preguntas de seleccin mltiple usadas durante los primeros 25 aos, desde que el examen con preguntas de ensayo se convirti en un examen con preguntas de seleccin mltiple. Como consecuencia de esto, se redujo la variedad de los tipos de temes a los A, B, C, G, K, X y M. A mediados de la dcada de los 80 se realiz una nueva revisin. En aquel momento, el consenso general fue que cuatro tipos bsicos de temes aseguraban una variedad suficiente para evaluar el conocimiento especfico importante para otorgar el certificado del National Board. Estos cuatro tipos bsicos incluyeron los temes tipo A, B, C y K. Los tipos G (grupo de temes de tipo A), tipo N (grupo de temes de tipo K), y tipo M no fueron consideradas por ms tiempo como formatos independientes. Recientemente, la variedad de tipos de temes fue nuevamente revisada. Los exmenes actuales incluyen slo los temes de los tipos A y R. Algunas de las medidas tomadas para mejorar los exmenes incluyen: concentrarse en tipos de temes que sean psicomtricamente bien fundados, instruir a los redactores de temes en tcnicas de composicin de temes, centrarse en la toma de decisiones clnicas ms que en temes de memoria y probar las nuevas preguntas antes de ser empleadas en un examen formal 1 .

[ENG: pretesting]

Apndice A

127

128

Apndice B Ejemplos de plantillas para escribir temes, temes, preguntas introductorias y listas de opciones para Ciencias Bsicas y Clnicas

130

Apndice B Ejemplos de plantillas para escribir temes, temes, preguntas introductorias y listas de opciones para Ciencias Bsicas y Clnicas
Anatoma Macroscpica
Plantilla para temes de Anatoma Macroscpica Enunciado del tem 1 (vieta de un paciente): (describa un paciente con un problema) Pregunta introductoria: En cul de las siguientes estructuras/procesos es ms probable encontrar un defecto? Opciones: (lista de estructuras/procesos)

Un hombre de 65 aos tiene dificultad para levantarse desde una posicin sentada y enderezar su tronco, pero no tiene dificultad para flectar sus muslos. Cul de los siguientes msculos podra estar daado con mayor probabilidad? A.* Glteo Mayor D. Psoas ilaco B. Glteo Menor E. Obturador Interno C. Posterior del muslo

Un hombre de 30 aos presenta prdida de la sensacin de temperatura y dolor en el lado izquierdo de su cara y en el lado derecho del cuerpo desde el cuello hacia abajo, parlisis parcial del paladar blando, laringe y faringe del lado izquierdo y ataxia en su lado izquierdo. Este sndrome est ms probablemente relacionado con la trombosis de cul de las siguientes arterias A. Basilar D. Cerebelar superior derecha B.* Cerebelar derecha posterior inferior E. Cerebelar superior izquierda C. Cerebelar izquierda posterior inferior

[ENG: Item Stem]

Apndice B

131

Inserte una vieta del paciente describiendo un paciente con un problema. Cul de las siguientes lesiones de rodilla es la ms probable? A. Condromalacia rotuliana F. Osteocondritis disecante B. Dislocacin (tibiofemoral) G. Osteosarcoma C. Fractura rotuliana H. Bursitis prerotuliana D. Quiste ganglionar I. Artritis sptica E. Tumor seo de clulas gigantes J. Rotura de meniscos

Inserte una vieta del paciente describiendo un paciente con un problema. En cul de los siguientes vasos debiera ser inyectado el medio de contraste durante la fluoroscopia para visualizar el sitio de la anormalidad? A. Arteria celaca D. Arteria mesentrica superior B. Arteria ilaca interna E. Arteria renal C. Arteria mesentrica inferior F. Vena porta

Inserte una vieta del paciente describiendo un paciente con un problema. Cul de las siguientes enfermedades gastrointestinales es la causa ms probable de estos hallazgos? A. Esofagitis por candidiasis F. Estenosis pilrica B. Diverticulitis G. Enteritis regional C. Hernia hiatal H. Absceso subfrnico D. Ulcera pptica I. Colitis ulcerosa E. Enterocolitis pseudomembranosa

Versin de datos aislados Cul de las siguientes reas est perfundida por la arteria cerebelar posterior inferior?

Versin de Aplicacin del Conocimiento Un hombre de 62 aos desarrolla ataxia de sus extremidades izquierdas, sndrome de Horner, nistagmo, prdida de sensacin trmica y dolor en la cara. Cul de las siguientes arterias est ocluida con mayor probabilidad?

132

Ciencias de la Conducta
Inserte una vieta describiendo la edad de un nio y lo que puede hacer. Cul de los siguientes describe mejor el nivel de desarrollo? Habilidades de Lenguaje Cognitivo Normal Normal Normal Normal Atrasado Atrasado Atrasado Atrasado Habilidades Motoras Generales Normal Normal Atrasado Atrasado Normal Normal Atrasado Atrasado Habilidades Sociales Normal Atrasado Normal Atrasado Normal Atrasado Normal Atrasado

A. B. C. D. E. F. G. H.

Un hombre de 55 aos ingresa al Servicio de Urgencia debido a un fuerte dolor en el pecho. El paciente se ve calmado. Su temperatura es 37 C, su pulso es 68/minuto y su presin arterial 130/74 mmHg. Cul de las siguientes preguntas es la ms apropiada para ser preguntada primero? A. B. C. D. E. F. . Tiene usted una historia de enfermedad cardaca o infarto al miocardio? Ha tenido usted alguna vez dolor en el pecho antes? Cunto dur su ltimo dolor en el pecho? El dolor en el pecho es penetrante o sordo? Qu me puede contar de su dolor al pecho? Dnde se localiza su dolor en el pecho?

Apndice B

133

Una mujer de 35 aos visita a su mdico despus de palparse una masa en una de sus mamas. Ella llora mientras le cuenta al mdico que su madre muri de cncer mamario. Cul de las siguientes respuestas, de parte del mdico, es la ms apropiada? A. Me doy cuenta que al encontrar esa masa usted se intranquiliz. B. Yo dudo que esa masa sea un cncer mamario. C. Dgame cmo se sinti cuando encontr esta masa? D. No hay razn para preocuparse hasta practicar una biopsia de la masa E. El tratamiento para el cncer mamario ha mejorado mucho desde que su madre muri por cncer mamario Versin de datos aislados Cul de los siguientes caracteriza mejor la angustia de separacin? Versin de Aplicacin del Conocimiento Un nio de 8 aos necesita ser halagado para ir a la escuela y cuando va se queja frecuentemente de cefalea severa o dolores abdominales. Algunas veces su madre tiene que llevarlo a casa por sus sntomas. En las noches el nio trata de dormir con sus padres. Cuando lo envan a su habitacin, el nio dice que hay monstruos en su closet. Cul de los siguientes explica mejor esta conducta? A. Conducta apropiada a la edad B. Esquizofrenia infantil C.* Angustia de separacin D. Fobia social E. Desorden Psictico Compartido

134

Bioqumica
Inserte una vieta describiendo un paciente con un problema. Cul de las siguientes enzimas digestivas o cofactores es ms probable que est involucrada? A. Amilasa F. Lipasa B. Quimiotripsina G. Pepsina C. Colipasa H. Sacarasa D. Enteroquinasa I. Tripsina E. Lactasa

Inserte una vieta describiendo un paciente con un problema. Cul de los siguientes resultados de laboratorio es ms probable? A. Hipercalcemia E. Hipernatremia B. Hipocalcemia F. Hiponatremia C. Hipermagnesemia G. Hiperkalemia D. Hipomagnesemia H. Hipokalemia

Versin de datos aislados La porfiria aguda intermitente es el resultado de un defecto en la va biosinttica para: A. Colgeno B. Corticosteroides C. cidos grasos D. Glucosa E. Heme F. Tiroxina

Versin de Aplicacin del Conocimiento Un hombre de 33 aos, saludable hasta hace 6 meses, inici episodios de debilidad moderada y dolor abdominal severo sin diarrea. Una ta y un primo han tenido episodios similares. Durante un episodio su abdomen se distendi y los ruidos intestinales diminuyeron. El examen neurolgico mostr una leve debilidad en la parte superior de sus brazos. Estos resultados sugieren un defecto en la va de sntesis de cul de los siguientes elementos? A. Colgeno B. Corticosteroides C. cidos grasos D. Glucosa E. * Heme F. Tiroxina (T4)

Apndice B

135

Versin de datos aislados

Versin de Aplicacin del Conocimiento

Cul de las siguientes caracteriza a la - Una mujer de 45 aos presenta rpido aumento de tamao de su hgado asociado con deterioro de la funcin heptica y cirrosis a Fetoproteina? causa de un episodio de hepatitis. La concentracin srica de cul de los siguientes estar anormal con mayor probabilidad? A. B. C. D. E. 1-antitripsina Antgeno carcinoembrionario Gonadotropina corinica -Fetoprotena Gastrina

Versin sin vieta Un desorden metablico de carbohidratos heredado, caracterizado por una concentracin de glicgeno heptico anormalmente aumentado, de estructura normal y sin un aumento detectable de la glicemia despus de administracin oral de fructosa. Estas dos observaciones sugieren que la enfermedad es el resultado de la ausencia de cul de las siguientes enzimas? A. B. C.* D. E. Fructoquinasa Glucoquinasa Glucosa-6-fosfatasa Fosfoglucomutasa UDPG-glicgeno transglucosilasa

Versin con vieta Un lactante de 6 meses de edad presenta hepatomegalia. La evaluacin para enfermedades metablicas muestra glicgeno heptico anormalmente aumentado, de estructura normal y sin un aumento detectable de la glicemia despus de administracin oral de fructosa. Estas dos observaciones sugieren que la enfermedad es el resultado de la ausencia de cul de las siguientes enzimas? A. B. C.* D. E. Fructoquinasa Glucoquinasa Glucosa-6-fosfatasa Fosfoglucomutasa UDPG-glicgeno transglucosilasa

136

Recuerdo de Hechos Aislados Cul de las siguientes vitaminas o minerales est involucrada en la sntesis de factores de coagulacin?

Aplicacin de Conocimiento Un viudo de 70 aos presenta equimosis, petequias e hinchazn gingival. Su dieta consiste bsicamente de bebidas Cola y salchichas. Cul de las siguientes vitaminas o minerales es ms probable que est deficiente en este paciente? I. J. K.* L. M. N. O. P. Biotina Cobre Folato Yodo Hierro Magnesio Niacina Zinc

A. B. C. D. E. F. G. H.

Vitamina A Vitamina B1 Vitamina B2 Vitamina B6 Vitamina C Vitamina D Vitamina E Vitamina K


B B B

Otras opciones (beriberi; kwashiorkor, pelagra; raquitismo; escorbuto) parecen menos apropiadas.

En la va metablica ramificada, cada paso est catalizado por una enzima diferente. La enzima inhibida ms severamente por el producto V es: A. A B.* B C. C D. D E. E

(A) I

(B) II (E)

(C) III

(D) IV

IV

Una mujer de 25 aos presenta 8 semanas de embarazo. Su ingesta oral de calcio es inadecuada. Si ella no recibe un suplemento Cul de las siguientes ser la fuente primaria de calcio del feto en su desarrollo? A.* Hueso esponjoso 1 B. Hgado C. Glndulas paratiroides D. E. Tbulos renales Intestino delgado

[ENG: cancellous bone]

Apndice B

137

Gentica
Un nio recin nacido presenta hemlisis severa e insuficiencia circulatoria. Su madre de 26 aos de edad es Rh negativa y tuvo previamente dos abortos en el tercer trimestre. La condicin actual del nio pudo haber sido prevenida por la administracin de cul de los siguientes anticuerpos a la madre? A. IgG Anti-RhD durante el ltimo embarazo B.* IgG Anti-RhD al final de cada uno de los dos primeros embarazos C. IgM Anti-RhD durante el ltimo embarazo D. IgM Anti-RhD al final del primer embarazo

Los genes de un cromosoma bacteriano tienen las siguientes frecuencias de asociacin en la transferencia de conjugacin, x e y, 25% de las veces; z e y, 50% de las veces. Si el orden gentico correcto es x-y-z, En qu porcentaje aproximado de las veces, x y z sern transferidos juntos? A. 1% B. 5% C. 13% D. 20% E.* 40%

La deficiencia de cul de las siguientes enzimas puede ser la causa ms probable de hiperamonemia hereditaria caracterizada por un EEG muy anormal y una concentracin elevada de amonio sanguneo? A. Asparagina sintetasa B.* Carbamil Fosfato Sintetasa-I C. Fumarasa D. Glutamato-Oxaloacetato aminotransferasa E. Glutaminasa

138

Histologa/Biologa Celular
Inserte la descripcin de una accin. Cul de los siguientes organelos celulares est ms directamente involucrado? A. Complejo de Golgi E. Envoltura nuclear B. Lisosoma F. Grnulos secretorios (cimgenos) C. Peroxisoma G. Retculo endoplasmtico rugoso D. Mitocondria H. Retculo endoplasmtico liso

Vieta de laboratorio
Varias clulas contiguas estn marcadas intracelularmente con un colorante fluorescente que no puede cruzar la membrana celular. Una clula es iluminada experimentalmente con una luz que destruye el colorante, pero luego de un corto tiempo esta clula recupera la fluorescencia. Esta recuperacin se explica mejor por la presencia de cul de las siguientes estructuras entre la clula iluminada y su vecina fluorescente? A. Lmina Basal B. Desmosomas (maculae adherentes) C.* Uniones de brecha 1 D. E. Glicosaminoglicanos Uniones fuertes 2 (zonulae occludentes)

1 2

[ENG: Gap junctions] [ENG: Tight junctions]

Apndice B

139

Microbiologa
Plantilla para temes de Microbiologa: Enunciado del tem (vieta del paciente): (describa un paciente con un problema) Pregunta introductoria: Es ms probable una infeccin con cul de los siguientes organismos? Opciones: (lista de patgenos)

En un banquete, el men inclua pollo frito, papas fritas, guisantes, chocolate, eclairs de chocolate y caf. A las 2 horas de haber cenado, la mayora de los asistentes presentaron nuseas, vmitos y dolor abdominal. Cul de los siguientes organismos se encontrar con mayor probabilidad en un gran nmero en el anlisis del alimento contaminado? A. Escherichia coli B. Proteus mirabilis C. Salmonella typhimurium D.* Staphylococcus aureus E. Streptococcus faecalis F. Enterococcus

Inserte una vieta describiendo un paciente con un problema. Cul de las siguientes toxinas es ms probable que est involucrada en la patognesis? A. B. C. D. E. Toxina botulnica Toxina difterica Toxina pertussis Toxina shiga Toxina tetnica F. G. H. I. Enterotoxina del Clera Enterotoxina del Clostridium difficile Enterotoxina termoestable de Escherichia coli Enterotoxina de Staphylococcus aureus

140

Inserte una vieta describiendo un paciente con un problema. teraputico ms apropiado? A. B. C. D. E. F. G. Aciclovir Anfotericina B Eritromicina Ganciclovir Gentamicina Ketoconazol Miconazol H. I. J. K. L. M.

Cual de los siguientes es el agente

Pamoato de pirantel Piracinamida Rifampicina Trimetoprim-sulfametoxazol Vancomicina Zidovudina (AZT)

tem de hecho aislado: Cul de los siguientes patgenos es un organismo gram-positivo que crece usualmente en pares de cadenas cortas? (la misma lista de opciones que abajo): Respuesta: T tem de aplicacin de conocimiento: Una nia de 7 aos tiene fiebre y odinofagia. Presenta enrojecimiento farngeo, amgdala derecha inflamada con exudado cremoso y una linfoadenopata dolorosa submandibular derecha. En el cultivo farngeo en placas de agar sangre se desarrollan numerosas colonias beta hemolticas pequeas que son inhibidas por bacitracina. Cul de los siguientes patgenos es ms probable que cause su enfermedad? A. B. C. D. E. F. G. H. I. J. K. Adenovirus Aspergillus fumigatus Bacillus anthracis Candida albicans Chlamidia psittaci Coccidoides immitis Coronavirus Corynebacterium diphtheriae Coxiella burnetti Virus Coxsackie Virus Epstein Barr L. M. N. O. P. Q. R. S. T. U. Hemophilus influenza Histoplasma capsulatum Micobacterium tuberculosis Mycoplasma pneumoniae Neisseria gonorrheae Neisseria meningitidis Pneumocistis carinii Rhinovirus Streptococcus pneumoniae Streptococcus pyogenes (grupo A)

Apndice B

141

Ejemplos de tem Un nio de 2 aos ha tenido infecciones recurrentes desde los 6 meses de edad. La concentracin de complemento srico, funcin fagoctica y bactericida de neutrfilos es normal. Un examen de piel con antgenos de cndida gener 2 cm de induracin a las 48 horas. Cul de las siguientes opciones provee la mejor explicacin para el momento de inicio de la enfermedad a los 6 meses de edad? A. B. C. D. E. Desarrollo de una infeccin viral Exposicin a una bacteria rara Prdida de inmunidad pasiva de la madre Defecto de maduracin del timo Transferencia transplacentaria de anticuerpos IgM

Cul de los siguientes exmenes de laboratorio estar anormal con mayor probabilidad? A. B. C. D. E. Actividad mieloperoxidasa Razn de linfocitos T CD4/CD8 Concentracin de cloruro en la piel Receptores Fc en los macrfagos Concentracin de IgG srico

142

Neurociencia (Neuroanatoma y Neuropatologa)


Plantilla para temes de Neuroanatoma: Enunciado del tem (vieta del paciente): (describa un paciente con un problema) Declaracin introductoria: En cul de las siguientes localizaciones es ms probable que se presente una anormalidad? Opciones: (lista de sitios)

Inserte una vieta describiendo un paciente con un problema. Cul de los siguientes nervios craneanos es el sitio ms probable de la lesin? A. Nervio olfatorio G. Nervio facial B. Nervio ptico H. Nervio vestibulocloclear C. Nervio oculomotor I. Nervio glosofarngeo D. Nervio troclear J. Nervio vago E. Nervio trigmino K. Nervio accesorio espinal F. Nervio abducens L. Nervio hipogloso

Inserte una vieta describiendo un paciente con una anormalidad neurolgica. Cul de las siguientes ramas del plexo braquial es ms probable que est afectada? A. Axilar G. Musculocutneo B. Dorsal escapular H. Radial C. Supraescapular I. Torxico largo D. Subescapular superior J. Toracodorsal E. Subescapular inferior K. Cubital F. Mediano

Apndice B

143

Inserte una vieta describiendo un paciente con un problema. sistema motor est involucrado con mayor probabilidad? A. Ganglio basal F. B. Hemisferio cerebelar G. C. Vermis cerebelar H. D. Relevo motor del tlamo I. E. rea premotora

Cul de los siguientes componentes del Corteza motora sensitiva Colculo superior/area pretectal rea motora suplementaria Cuerno Ventral

Inserte una vieta describiendo un paciente con un problema. La causa ms probable es la oclusin de cul de las siguientes arterias? A. Cerebral anterior izquierda E. Cerebral posterior izquierda B. Cerebral anterior derecha F. Cerebral posterior derecha C. Cerebral media izquierda G. Lentculo estriada izquierda D. Cerebral media derecha H. Lentculo estriada derecha

Inserte una vieta describiendo un paciente con un problema. Cul de los siguientes es el diagnstico ms probable? A. Esclerosis lateral amiotrfica E. Poliomielitis B. Enfermedad de Huntington F. Polineuropata C. Esclerosis mltiple G. Retinitis pigmentosa D. Enfermedad de Parkinson

144

Ejemplos de un conjunto de temes Una mujer de 58 aos, inconciente, es llevada al Servicio de Urgencia despus de sufrir un colapso en un centro de compras local. Su familia informa que ella se senta bien en la maana pero que desarroll una cefalea de intensidad creciente. Tiene antecedentes de hipertensin arterial y fibrilacin auricular, por lo que recibe medicamentos antihipertensivos y anticoagulantes orales. Al examen fsico: presin arterial 220/130 mmHg. Presenta apnea alternada con hiperpnea y responde solo a estmulos dolorosos con extensin postural de brazo y pierna derecha. El fondo de ojo muestra edema de papila que compromete el disco ptico izquierdo. Las pupilas son 3.0/7.0 (derecha/izquierda) sin reaccin a la luz en la izquierda y con una preferencia de mirada a izquierda. Presenta hiperreflexia difusa, mayor en lado derecho y signo de Babinski bilateral. 1. Con cul de las siguientes estructuras del lado izquierdo que presente una lesin es ms consistente la presencia de una pupila izquierda no reactiva y dilatada? A. Nervio ptico D. Protuberancia B. Tracto ptico E. Colculo superior C.* Nervio oculomotor Con una lesin en cul de las siguientes reas del cerebro izquierdo es ms consistente la postura en extensin del brazo derecho? A. Telencfalo D. Protuberancia B. Diencfalo E. Bulbo Raqudeo C.* Cerebro medio Cul de los siguientes describe mejor su perfil respiratorio? A.* Cheyne-Stokes B. Hiperventilacin Neurognica Central C. Apnustica 1 D. Atxico Cual de los siguientes sndromes de herniacin es ms consistente con su presentacin clnica? A. Giro cingulado detrs de la hoz B.* Uncus del lbulo temporal a travs del tentorio C. Diencfalo a travs de la cisura tentorial D. Tronco enceflico a travs de la cisura tentorial E. Amgdalas cerebelares a travs del foramen magno

2.

3.

4.

[ENG: Apneustic]

Apndice B

145

Patologa
Inserte la descripcin de una accin. Cul de los siguientes organelos celulares est ms directamente involucrado? A. Complejo de Golgi E. Envoltura nuclear B. Lisosomas F. Grnulos secretorios (cimgenos) C. Peroxisomas G. Retculo endoplasmtico rugoso D. Mitocondria H. Retculo endoplasmtico liso

Inserte una vieta describiendo un paciente con un problema. Cul de los siguientes se encontrar con ms probabilidad en una biopsia de tejido renal? A. Vasculitis necrotizante aguda F. Granulomas B. Depsitos de amiloide G. Neutrfilos intersticiales C. Necrosis cortical H. Glomeruloesclerosis nodular D. IgA glomerular mesangial I. Tbulos regenerativos E. IgG glomerular perifrica J. Cristales de cido rico

Inserte una vieta describiendo un paciente con un problema. Cul de las siguientes anormalidades endocrinas est presente con mayor probabilidad? A. Adenoma cortical adrenal F. Feocromocitoma B. Carcinoma cortical adrenal G. Adenoma de clula de islote pancretico C. Hiperplasia adrenal H. Adenoma paratiroideo D. Necrosis adrenal I. Carcinoma paratirodeo E. Neuroblastoma adrenal J. Hiperplasia paratiroidea

Un hombre de 32 aos muere 4 das despus de fracturarse el fmur en un accidente automovilstico. El examen de la autopsia de su cerebro revela petequias esparcidas por toda la materia blanca. Qu otro hallazgo adicional es ms probable que se encuentre en su autopsia? A. Sndrome de dificultad respiratoria del adulto B. Injuria por contusin cerebral C.* Embolia grasa D. Septicemia E. Hematoma subdural

146

La autopsia de una mujer de 24 aos de edad muestra una pleuritis, engrosamiento membranoso de la pared capilar glomerular, anillos concntricos de colgeno alrededor de las arteriolas esplnicas y excrecencias en la cara inferior de la vlvula mitral. Cul de los siguientes hallazgos es ms probable en la sangre de esta mujer? A.* Anticuerpo antinuclear B. Concentracin aumentada de C3 C. Linfocitosis D. Gamapata monoclonal E. Cultivo bacteriano positivo

Un paciente con hepatitis B muere 9 das despus del inicio de los sntomas. Cul de los siguientes hallazgos es ms probable en el examen microscpico de su hgado? A. Vacuolizacin grasa difusa con necrosis mnima B. Fibrosis difusa con nodularidad C. Necrosis zonal perifrica limitada D.* Necrosis hepatocelular difusa E. Infiltracin neutroflica difusa en lbulos

Apndice B

147

Mecanismos Patolgicos
Plantilla para temes de Mecanismos: Enunciado del tem (vieta de un paciente): (describa un paciente con un problema) Declaracin introductoria: Cul de los siguientes mecanismos es la causa ms probable de los hallazgos en el paciente? Opciones: (lista de mecanismos)

Inserte una vieta describiendo un paciente con un problema. Cul de las siguientes es la causa ms probable de la enfermedad miocrdica? A. Toxicidad alcohlica B. Amiloidosis cardiaca C. Fibrosis endomiocrdica D. Hemocromatosis E. Cardiomiopata hipertrfica F. Endocarditis de Lffler G. Miocarditis postviral H. Sarcoidosis I. Tripanosomiasis sudamericana (enfermedad de Chagas) J. Deficiencia de vitamina B1 (tiamina) Inserte una vieta describiendo un paciente con ictericia su ictericia? A. Enfermedad heptica alcohlica B. Reaccin a droga C. Sndrome de Dubin-Johnson D. Sndrome de Gilbert E. Ictericia hemoltica Cul de las siguientes es la causa ms probable de F. G. H. I. Atresia ductal intraheptica Ictericia obstructiva Cirrosis biliar primaria Hepatitis viral

148

Inserte una vieta describiendo un paciente con una anormalidad hemosttica. Cul de las siguientes es la causa ms probable de la anormalidad hemosttica? A. B. C. D. Coagulacin intravascular diseminada aguda Deficiencia de factor V (proacelerina) Deficiencia de factor VII (proconvertina) Hemofilia A E. F. G. H. Hemofilia B Prpura trombocitopnico idioptico Tromboembolismo pulmonar Enfermedad de Von Willebrand

Inserte una vieta describiendo un paciente con un problema. gastrointestinal ms probable? A. B. C. D. E. Esofagitis candidal Diverticulitis Hernia hiatal Ulcera pptica Enterocolitis pseudomembranosa F. G. H. I.

Cul de las siguientes es la lesin

Estenosis pilrica Enteritis regional Absceso subfrnico Colitis ulcerosa

Apndice B

149

Diagnstico Patolgico
Plantilla para temes de Diagnstico: Enunciado del tem (vieta del paciente): (describe un paciente con un problema) Declaracin introductoria: Cul de los siguientes es el diagnstico ms probable? Opciones: (lista de diagnsticos)

Inserte una vieta describiendo un paciente con una anormalidad histolgica. Cul de las siguientes es la enfermedad tiroidea ms probable? A. Tiroiditis autoinmune de Hashimoto B. Enfermedad de Graves C. Mixedema despus de tratamiento con I131 D. Tiroiditis subaguda E. Carcinoma tiroideo bien diferenciado

Inserte una vieta describiendo un paciente con un problema. Cul es el diagnstico ms probable? A. Leucemia linfoblstica aguda B. Leucemia mielognica aguda C. SIDA D. Leucemia linfoctica crnica E. Sarcoma de Ewing F. Linfoma folicular G. Enfermedad de Hodgkin H. Prpura trombocitopnico idioptico I. Mieloma mltiple J. Policitemia vera K. Histiocitosis X L. Linfoma de clulas T

150

Inserte una vieta describiendo un paciente con una enfermedad cardaca. Cul de los siguientes es el diagnstico ms probable? A. B. C. D. E. Miocarditis viral aguda Enfermedad de Chagas Miocardiopata dilatada Miocarditis de clulas gigantes Hemocromatosis F. G. H. I. J. Miocardiopata hipertensiva Miocardiopata hipertrfica Miocardiopata isqumica Enfermedad de Pompe Miocardiopata restrictiva

Inserte una vieta describiendo un paciente con un problema. Cul de los siguientes tipos celulares est ms probablemente involucrado con la lesin? A. B. C. D. Basfilo Clula endotelial Eosinfilo Linfocito E. F. G. H. Clula mesotelial Monocito Neutrfilo Clula plasmtica

Inserte una vieta describiendo un paciente con una enfermedad renal. Cul es el diagnstico ms probable? A. B. C. D. Glomerulonefritis postestreptoccica aguda Esclerosis segmental focal Sndrome de Goodpasture Nefropata por IgA E. F. G. H. Nefritis por Lupus Glomerulonefritis membranoproliferativa Glomerulonefritis membranosa Nefropata de cambio mnimo

Un hombre de 21 aos ha perdido peso y presenta una diarrea intermitente. El enema baritado y la colonoscopa muestran mltiples lceras y cambios inflamatorios extendindose desde el recto hasta el colon medio transverso. Las biopsias de sitios mltiples muestran inflamacin crnica y aguda restringida a la mucosa. Cul de los siguientes es el diagnstico ms probable? A. B. C.* D. Gastroenteritis asociada a SIDA Amebiasis Enfermedad de Crohn Colitis asociada a Clostridium difficile E. F. G. H. Colitis asociada a Escherichia coli Colitis isqumica Gastroenteritis por salmonella Colitis ulcerativa

Apndice B

151

Una mujer de 38 aos presenta una insuficiencia cardaca congestiva, contracciones ventriculares prematuras y episodios repetidos de taquicardia ventricular. Su presin arterial es normal. Su corazn esta marcadamente agrandado. No hay soplos cardacos y la angiografa coronaria es normal. Cul de los siguientes es el diagnstico ms probable? A. Fiebre reumtica aguda B. Fibroelastosis congnita C. Pericarditis constrictiva D.* Infarto al miocardio E. Cardiomiopata primaria

Un hombre de 74 aos de edad presenta dolor abdominal clico en su cuadrante inferior izquierdo, leucocitosis y fiebre. No presenta diarrea ni constipacin. Cul de los siguientes es el diagnstico ms probable? A. Carcinoma del colon sigmoideo B.* Diverticulitis C. Poliposis adenomatosa familiar D. Colitis ulcerativa E. Adenoma velloso del recto superior

152

Fisiopatologa
Plantilla para temes de Fisiopatologa Hallazgos adicionales Enunciado del tem (vieta del paciente): (describe un paciente con un problema) Declaracin introductoria: Cul de los siguientes hallazgos adicionales es ms probable? Opciones: (lista de hallazgos)

Los exmenes de laboratorio de un hombre con edema muestran una concentracin srica normal de complemento y una concentracin srica aumentada de colesterol. El anlisis urinario muestra proteinuria (4+), 0.5 eritrocitos por campo y varios cilindros hialinos. Cul de los siguientes hallazgos es ms probable en una biopsia renal? A. Glomerulonefritis postestreptoccica (proliferativa) aguda B. Glomerulonefritis membrano proliferativa C.* Glomerulonefritis membranosa D. Enfermedad de cambio mnimo E. Glomerulonefritis rpidamente progresiva

Un joven de 16 aos est siendo evaluado por su ictericia. Los estudios de laboratorio muestran actividad enzimtica heptica normal, test de antiglobulina directa negativa, concentracin de hemoglobina corpuscular media aumentada y aumento de la fragilidad osmtica de eritrocitos. Cul de los siguientes tipos de eritrocitos es ms probable encontrar en un frotis de sangre perifrica? A. Ovalocito D. Clula en diana B. Esquistocito E. Clula en forma de lgrima C.* Esferocito

Apndice B

153

Una mujer de 50 aos asintomtica, portadora de hipertensin arterial, presenta excrecin aumentada de catecolaminas urinarias. Una TAC muestra una masa suprarrenal. Cul de los siguientes hallazgos es ms probable en el examen microscpico de la masa? A. Neoplasia benigna de corteza D. Neoplasia maligna de mdula adrenal B.* Neoplasia benigna de mdula E. Hiperplasia difusa de corteza adrenal C. Neoplasia maligna de corteza F. Hipoplasia difusa de mdula adrenal

Inserte una vieta describiendo un paciente con un problema. Cul de los siguientes es el hallazgo pulmonar ms probable en la autopsia? A. Mltiples burbujas sobre la superficie con una consistencia blanda esponjosa B. Exudado viscoso, espumoso y sanguinolento que proviene de la superficie cortada C. Ndulos fibrticos difusos de apariencia en espiral que reemplazan el parnquima normal D. Lbulo inferior izquierdo con consistencia similar al hgado, que contiene fibrina y neutrfilos en sus alvolos E. De apariencia normal; con gran cogulo laminado y enroscado en forma de Y, que casi llena ambas arterias pulmonares.

Inserte una vieta describiendo un paciente con un problema. Cul de los siguientes grupos de resultados es ms probable?
Volumen Circulatorio Volumen del Lquido Efectivo Extracelular Volumen Plasmtico Excrecin Urinaria de Na+

A. B. C. E.

disminuido disminuido disminuido aumentado

disminuido aumentado aumentado aumentado

disminuido disminuido aumentado aumentado

disminuido disminuido disminuido aumentado

Inserte una vieta describiendo un paciente con un problema. Cul de las siguientes enzimas digestivas o cofactores es ms probable que est involucrado/a? A. Amilasa F. Lipasa B. Quimotripsina G. Pepsina C. Colipasa H. Sacarasa D. Enteroquinasa I. Tripsina E. Lactasa

154

Inserte una vieta describiendo un paciente con un problema. Cul de los siguientes resultados de laboratorio es ms probable? A. Hipercalcemia E. Hipernatremia B. Hipocalcemia F. Hiponatremia C. Hipermagnesemia G. Hiperpotasemia D. Hipomagnesemia H. Hipopotasemia

Inserte una vieta describiendo un paciente con un problema. Cul de los siguientes es el perfil de funcin tiroidea ms probable? Tiroxina (T4) A. B. C. D. E. F. G. normal Resina de captura de T3 normal normal normal normal Hormona TSH normal normal T3 normal normal normal

Inserte una vieta describiendo un paciente con hallazgos electrocardiogrficos. Cul de las siguientes arritmias cardacas es ms probable que est presente? A. Fibrilacin auricular F. Contracciones auriculares prematuras B. Taquicardia paroxstica auricular G. Contracciones ventriculares prematuras C. Bloqueo incompleto de primer grado H. Arritmia sinusal D. Bloqueo incompleto de segundo grado I. Fibrilacin ventricular E. Bloqueo completo de tercer grado J. Taquicardia paroxstica ventricular

Apndice B

155

Inserte una vieta describiendo un paciente con anormalidad cardaca. (Lactante de 4 semanas con un fuerte soplo sistlico y frmito palpatorio sistlico; en todos los otros aspectos el lactante se presenta sano, no hay cianosis) Cul de las siguientes enfermedades congnitas es ms probable que est presente? . A. Defecto del tabique auricular E. Ductus arterioso persistente B. Coartacin de la aorta F. Estenosis de vlvula pulmonar C. Transposicin completa de los grandes G. Tetraloga de Fallot vasos H. Defecto del tabique ventricular D. Defecto del cojinete endocrdico

156

Farmacologa
Plantilla para Efectos Farmacolgicos Adversos: adivina mi droga Enunciado del tem (vieta del paciente): (describe un paciente con un efecto adverso a una droga) Declaracin introductoria: Cul de las siguientes drogas es ms probable que est recibiendo el paciente? Opciones: (lista de drogas)

Un paciente en el servicio de Urgencia no logra recordar cul pldora para el corazn est tomando. Dice que tiene como un campanilleo en los odos. Su frecuencia cardaca es superior a 80 latidos por minuto. El ECG muestra un intervalo PR y complejos QRS prolongados. Cul de las siguientes drogas est tomando el paciente con mayor probabilidad? A. Digoxina B. Lidocana C. Fenitona D. Propranolol E.* Quinidina

Inserte una vieta describiendo un paciente con el efecto adverso de una droga. Cul de las siguientes drogas es ms probable que est causando el efecto adverso? A. Acetominofeno J. cido Nalidxico B. Amiodarona K. Nitrofurantona C. Inhibidores de la ECA L. Penicilina D. Aspirina M. Prednisona E. Atenolol N. Procainamida F. Bleomicina O. Propranolol G. Citosina Arabinosido P. Sulfasalasina H. Furosemida Q. Tetraciclina I. Metronidazol R. Verapamilo

Apndice B

157

La droga Y tiene un volumen de distribucin (Vd) de 75L en hombres jvenes y viejos. En hombres jvenes tiene un clearance de 15 L/hora, 50% del cual ocurre a travs del hgado y el otro 50% a travs de los riones. Para los hombres jvenes la posologa es de 100 mg cada 6 horas. Cul de las siguientes posologas producir esencialmente la misma concentracin sangunea en un hombre de edad avanzada cuyo clearance de creatinina es la mitad que la del hombre joven, pero con funcin heptica normal? A. 75 mg cada 3 horas B.* 75 mg cada 6 horas C. 75 mg cada 9 horas D. 100 mg cada 3 horas E. 100 mg cada 6 horas F. 100 mg cada 9 horas

Una mujer de 24 aos, sexualmente activa, reporta un aumento de secrecin vaginal. En el examen plvico se aprecia una descarga espumosa verde. Un examen microscpico del flujo mostr organismos unicelulares motiles de 10-30 micrones de largo. Cul de las siguientes caractersticas de estos microorganismos podra explicar mejor el porqu esta infeccin puede ser efectivamente tratada con metronidazol? A. Aerobio facultativo B. Microaeroflico C. Aerbico estricto D.* Anaerobio estricto

Plantilla para tem de Sitios / Mecanismos de Accin - Farmacologa Enunciado del tem (vieta del paciente): (describa un paciente que requiere una farmacoterapia) Declaracin introductoria: Con cul de los siguientes sitios / mecanismos de accin sera efectiva con mayor probabilidad una droga? Opciones: (lista de mecanismos de accin) o (listas de sitos de accin)

158

Farmacoterapia
Plantilla para temes de Farmacoterapia: Enunciado del tem (vieta del paciente): (describa un paciente que requiere farmacoterapia) Declaracin introductoria: La administracin de cul de las siguientes drogas es ms apropiada? Opciones: (lista de drogas)

Un hombre de raza negra de 40 aos tiene un severo y repentino dolor de cabeza acompaado de mareos y vmitos. Su presin arterial es 260/130 mmHg; presenta encefalopata grado IV y retinopata. La administracin de cul de las siguientes drogas es ms apropiada? A. Agonistas alfa G. Naloxona B. Bloqueadores alfa H. Vasodilatadores directos C. Inhibidores de la ECA I. Agentes inotrpicos negativos D. Agonistas beta J. Diurticos tiacida E. Bloqueadores beta K. Vasoconstrictores F. Glicsidos cardacos

Inserte una vieta describiendo a un paciente que requiere farmacoterapia. Cul de las siguientes drogas es ms apropiada como terapia inicial? A. Adenosina G. Naloxona B. Aspirina H. Prednisona C. Cafena I. Propranolol D. Adrenalina J. Quinidina E. Insulina K. Estreptoquinasa F. Lidocana

Apndice B

159

Inserte una vieta describiendo un paciente que requiere farmacoterapia. Cul de las siguientes drogas es ms apropiada como terapia inicial? A. Carbamazepina F. Carbonato de litio B. Dextroanfetamina G. Fenobarbital C. Etosuccimida H. Primidona D. Haloperidol I. Propranolol E. L-dopa/carbidopa J. Piridostigmina

Inserte una vieta describiendo un paciente que requiere farmacoterapia. Cul de las siguientes drogas es ms apropiada como terapia inicial? A. Atropina F. Leche de magnesia B. Subsalicilato de bismuto G. Misoprostol C. Cimetidina H. Omeprazol D. Difenoxilato I. Ranitidina E. Caoln J. Sucralfato

Inserte una vieta describiendo un paciente que requiere farmacoterapia. Cul de los siguientes diurticos es ms apropiado como terapia inicial? A. Inhibidor de la anhidrasa carbnica B. De tbulo proximal o de asa C. Ahorrador de potasio no-esteroidal D. Osmtico E. Ahorrador de potasio esteroidal F. Tiacida G. Xantina

160

Fisiologa
Un paciente anestesiado est siendo ventilado mecnicamente. Los valores de gases sanguneos arteriales son normales. Si la ventilacin se disminuye, cul de los siguientes describira mejor los cambios de pCO2 y pH? pCO2 Arterial Disminuido Disminuido Disminuido Aumentado Aumentado Aumentado pH disminuido aumentado sin cambios disminuido aumentado sin cambios

A. B. C. D.* E. F.

Inserte una vieta describiendo a un paciente con un problema. Cul de las siguientes sustancias est ms probablemente involucrada? A. Vasopresina E. Bradiquinina B. Aldosterona F. Calcitonina C. Angiotensina G. Hormona paratiroidea D. Pptido natriurtico auricular H. Renina

Un hombre de 22 aos con una historia de 3 semanas de poliuria, polifagia y polidipsia, presenta durante las ltimas 12 horas nuseas, vmitos y disminucin de la respuesta a estmulos. El anlisis de orina muestra glucosa +4 y cetonas +4. Cul de los siguientes resultados de gases arteriales presentar el paciente con mayor probabilidad? pH 7.15 7.15 7.30 7.40 7.50 7.50 7.50 pO2 (mmHg) 98 94 56 100 100 100 56 pCO2 (mmHg) 33 28 80 40 33 24 33 HCO3- (mEq/L) 11 8 38 25 25 18 25

A. B.* C. D. E. F. G.

Apndice B

161

Inserte una vieta describiendo a un paciente con un problema. Cul de las siguientes sustancias humorales est ms probablemente involucrada? A. Acetilcolina G. Prostaciclina (PGI2) B. Adenosina H. Prostaglandina E2 C. Aldosterona I. Prostaglandina F2 D. Bradiquinina J. Serotonina E. Adrenalina K. Tromboxano A2 F. Noradrenalina

162

Conjunto de temes integradores


Plantilla para temes con indicaciones integradoras: Microbiologa / Farmacologa Enunciado de un tem (vieta del paciente): (describa un paciente con un problema) Declaracin introductoria: Cul de los siguientes patgenos es ms probable que sea el responsable de la infeccin? Opciones: (lista de patgenos) Declaracin introductoria: Con cul de las siguientes drogas se obtendr el mejor efecto teraputico? Opciones: (lista de drogas)

Apndice B

163

Un hombre de 40 aos con SIDA presenta historia de 1 semana de fiebre baja y letargo. La temperatura es 38 C, pulso 90/min y presin arterial 110/70 mmHg. Existe una resistencia pasiva a la flexin del cuello. Los resultados de laboratorio del lquido cefalorraqudeo mostraron: Presin de apertura: 210 mmHg Color Pajizo Recuento de leucocitos 400/mm3 (100% linfocitos) Recuento de eritrocitos 50/mm3 Glucosa 30 mg/dL Protena 100 mg/dL Preparacin de tinta china Levaduras encapsuladas 1. Cul de los siguientes patgenos es ms probable que produzca la infeccin? A. Blastomyces B. Cryptococcus C. Histoplasma D. Toxoplasma gondii E. Treponema pallidum Cul de las siguientes drogas es la ms apropiada para el tratamiento? A. Anfotericina B. Ketoconazol C. Miconazol D. Nistatina E. Trimetoprim sulfametoxazol

2.

164

Ejemplo de Conjunto de temes Una mujer de 34 aos presenta una diarrea acuosa desde hace 4 das. Dos meses atrs present mononucleosis infecciosa. Abusa de drogas intravenosas y es seropositiva para el VIH. El examen fsico muestra signos de deshidratacin y debilidad muscular marcada. 1. Cul de los siguientes hallazgos anormales de laboratorio es ms probable en esta paciente? D.* Aumento en la concentracin srica de A. Disminucin en la concentracin srica Na+ de K+ B. Disminucin en la concentracin srica E. Aumento en el pH srico de Ca2+ C. Aumento en la concentracin srica de HCO3En la evaluacin del curso de la diarrea, cul de los siguientes estudios es ms apropiado? A. Biopsia del colon para identificar Giardia lamblia B. Cultivo de la mucosa oral para Candida albicans C. Biopsia duodenal para identificar Entamoeba histolytica D. Aspirado gstrico para identificar Micobacterium avium-intracelullare E.* Muestra de deposiciones para identificar Cryptosporidium Estudios posteriores para evaluar la infeccin por VIH muestran una razn de linfocitos T helper/supresor de 0.3. Cul de las siguientes acciones del virus del Sida explica mejor esta razn? A. Induccin de proliferacin de linfocitos T helper B. Induccin de proliferacin de linfocitos T supresores C.* Infeccin de las clulas con el receptor CD4 D. Infeccin de macrfagos E. Estimulacin de la sntesis de leucotrienos

2.

3.

Apndice B

165

Plantillas para temes Integradores de Patologa / Farmacologa: Enunciado del tem (vieta del paciente): (describe un paciente con un problema) Declaracin introductoria: Cul de los siguientes es el diagnstico ms probable? Opciones: (lista de diagnsticos) Siguiente declaracin introductoria: La administracin de cul de las siguientes drogas es ms apropiada? Siguiente lista de opciones: (lista de drogas)

Ejemplo de temes 1. Un hombre de 62 aos con dependencia alcohlica es admitido al hospital para una reseccin prosttica transuretral. A la maana siguiente, mientras es trasladado a la sala de operaciones, presenta dos convulsiones generalizadas de 5 minutos de duracin. El examen neurolgico no muestra anormalidades focales. Cul de los siguientes diagnsticos es ms probable? A.* Abstinencia alcohlica C. Convulsin compleja parcial B. Sndrome de Korsakoff D. Encefalopata de Wernicke

2.

El manejo clnico ms apropiado de este paciente es la administracin intravenosa de cul de las siguientes drogas? A. Diazepam D. Fenitona B. Haloperidol E. Acido Valproico C. Fenobarbital

166

Ejemplo de conjunto de temes Un nio de 2 aos presenta infecciones recurrentes desde los 6 meses de edad. La concentracin del complemento srico, la funcin fagoctica y la actividad antibacteriana de los neutrfilos son normales. Un test drmico para antgeno de Candida muestra 2 cm de induracin a las 48 horas. Cul de los siguientes proporciona la mejor explicacin del momento de inicio de la enfermedad a los 6 meses de edad? A. Desarrollo de una infeccin viral B. Exposicin a una bacteria rara C. Prdida de la inmunidad pasiva desde la madre D. Defecto de maduracin del timo E. Transferencia de anticuerpos IgM transplacentarios

Cul de los siguientes tests de laboratorio est anormal con mayor probabilidad? A. La actividad de la mieloperoxidasa B. La razn CD4/CD8 de linfocitos T C. La concentracin de cloruros en el sudor D. Los receptores FC en macrfagos E. La concentracin srica de IgG

Plantillas para temes de Fisiologa / Farmacologa Enunciado del tem (vieta del paciente): (describa un paciente con un problema) Declaracin introductoria: A lograr cul de los siguientes efectos est dirigida la droga ms apropiada O cul de los siguientes efectos es el ms probable que tenga la administracin de {especificar una droga}? Opciones: (lista de efectos fisiolgicos)

Apndice B

167

Ejemplos de listas de opciones para la redaccin de temes de Ciencias Clnicas


Componentes de la vieta de un paciente Edad, Sexo (ejemplo, hombre de 45 aos) Lugar de atencin (ejemplo, acude al servicio de urgencia) Motivo de consulta (ejemplo, por presentar cefalea) Duracin (ejemplo, continuo durante los ltimos 2 das) Historia del paciente (con Historia Familiar?) Hallazgos del Examen Fsico Resultados (+/-) de los exmenes estudios diagnsticos Tratamiento inicial (+/-), hallazgos subsecuentes

Masa Abdominal A. B. C. D. E. F. G. H. I. Absceso abdominal Leucemia linfoctica aguda Carcinoma de colon Carcinoma de ovario Quiste del coldoco Estreimiento Tumor desmoide Distensin de vejiga Hepatoma J. K. L. M. N. O. P. Q. R. Hernia Quiste hidatdico Enfermedad intestinal inflamatoria Neuroblastoma Seudoquiste pancretico Rin poliqustico Fibromas uterinos Vlvulo Tumor de Wilms

Para cada paciente con masa abdominal, seleccione el diagnstico ms probable:

168

Dolor Abdominal A. B. C. D. E. F. G. H. I. J. Aneurisma abdominal Apendicitis Obstruccin intestinal Colecistitis Cncer de colon Estreimiento Diverticulitis Ruptura de embarazo ectpico Endometriosis Hernia K. L. M. N. O. P. Q. R. S. T. Litiasis renal Adenitis mesentrica Trombosis arterial mesentrica Ruptura de quiste ovrico Pancreatitis Enfermedad inflamatoria plvica lcera pptica lcera pptica perforada Pielonefritis Torsin

Para cada paciente con dolor abdominal, seleccione el diagnstico ms probable:

Estado Mental Alterado A. B. C. D. E. F. G. H. I. Absceso cerebral Episodio de isquemia transitoria Deshidratacin Sobredosis/Toxicidad de drogas Encefalopata heptica Hipercalcemia Hiperglicemia Hiperkalemia Encefalopata hipertensiva J. K. L. M. N. O. P. Q. R. Hipoglicemia Hiponatremia Hipotiroidismo Demencia multi-infarto Demencia degenerativa primaria tipo Alzheimer Convulsiones Sepsis Uremia Encefalopata de Wernicke

Para cada paciente con estado mental alterado, seleccione el diagnstico ms probable:

Apndice B

169

Anemia A. B. C. D. E. F. G. H. Leucemia mielobstica aguda Anemia aplstica Leucemia linfoctica crnica Anemia hemoltica inmune inducida por droga Malaria por Falciparum Deficiencia de folato Deficiencia de la glucosa 6-fosfato deshidrogenasa Esferocitosis hereditaria I. J. K. L. M. N. O. P. Anemia por deficiencia de hierro Metstasis en la mdula sea Anemia hemoltica microangioptica Mieloma mltiple Mielofibrosis Anemia de clulas falciformes Talasemia menor Deficiencia de vitamina B12 (cianocobalamina)

Para cada paciente con anemia, seleccione el diagnstico ms probable:

Dolor de espalda A. B. C. D. Espondilitis anquilosante Infeccin de disco intervertebral Mieloma mltiple Dolor miofascial E. F. G. H. Osteoporosis Estenosis del canal raqudeo Espndilolisis Tuberculosis espinal

Para cada paciente con dolor de espalda, seleccione el diagnstico ms probable:

170

Conducta Extraa (o anormal) /alteracin conductual A. B. C. D. E. F. G. H. I. Acidosis Psicosis aguda Hematoma subdural agudo Intoxicacin alcohlica Ingestin / Sobredosis de cocana Delirium tremens Depresin Hipoglicemia Hipoxia J. K. L. M. N. O. P. Q. Ingestin de LSD Simulacin Meningitis Sndrome de Reye Psicosis por esteroides Hemorragia subaracnoidea Convulsin lbulo temporal Sobredosis de antidepresivos tricclicos

Para cada paciente con conducta extraa, seleccione el diagnstico ms probable:

Ndulos mamarios / Molestias A. B. C. D. E. F. G. Quiste mamario Necrosis de grasa mamaria Fibroadenoma mamario Galactocele Ginecomastia Carcinoma mamario inflamatorio Papiloma intraductal H. I. J. K. L. M. Lipoma mamario Mastodinia Carcinoma mamario metastsico Enfermedad de Paget de la mama Mastitis puerperal Sndrome de Tietze

Para cada paciente con problemas mamarios, seleccione el diagnstico ms probable:

Apndice B

171

Dolor Torcico A. B. C. D. E. F. G. Angina de pecho Fractura compresiva espinal Aneurisma disecante de la aorta Espasmo esofgico Esofagitis Herpes zoster Hiperventilacin H. I. J. K. L. M. Infarto miocrdico Dolor de msculo pectoral Pericarditis Neumona Neumotrax Embolismo pulmonar

Para cada paciente con dolor torcico, seleccione el diagnstico ms probable:

Diarrea A. B. C. D. E. F. G. H. I. Amebiasis Gastroenteritis bacteriana Sndrome carcinoideo Carcinoma del colon Pancreatitis crnica Enfermedad de Crohn Infeccin por Cryptosporidium Diverticulitis Sndrome de vaciamiento gstrico rpido J. K. L. M. N. P. Q. R. Linfoma gstrico Colitis isqumica Abuso de laxantes Colitis pseudomembranosa Sndrome de intestino corto O. Esprue Adenoma velloso Gastroenteritis viral Sndrome de Zollinger-Ellison

Para cada paciente con diarrea, seleccione el diagnstico ms probable:

172

Fatiga A. B. C. D. E. F. G. Leucemia aguda H. Esferocitosis hereditaria Anemia: enfermedad crnica I. Hipotiroidismo Insuficiencia cardaca congestiva J. Deficiencia de hierro Depresin K. Enfermedad de Lyme Infeccin por virus de Epstein-Barr L. Tuberculosis miliar Deficiencia de folato M. Deficiencia de vitamina B12 Deficiencia de la glucosa 6-fosfato deshidrogenasa
B

Para cada paciente con fatiga, seleccione el diagnstico ms probable:

Fiebre / Adultos A. B. C. D. E. F. G. H. I. Apendicitis Intoxicacin por aspirina Celulitis Enfermedad de Crohn Gastritis Enfermedad de Hodgkin Mononucleosis infecciosa Meningitis Sobredosis de narcticos J. K. L. N. O. P. Q. R. Pancreatitis Neumona Prostatitis M. Embolismo pulmonar Pielonefritis Sinusitis Tuberculosis Infeccin del tracto urinario Infeccin respiratoria viral

Para cada paciente con fiebre, seleccione el diagnstico ms probable:

Apndice B

173

Fiebre en Nios A. B. C. D. E. F. G. H. I. Leucemia linfoctica aguda Meningitis bacteriana Varicela Fiebre por drogas Gastroenteritis Artritis reumatoidea juvenil Enfermedad de Kawasaki Mastoiditis Sarampin J. K. L. M. O. P. Q. R. Osteomielitis Otitis media Neumona neumoccica Pielonefritis N. Fiebre reumtica Rosola Rubola Sinusitis Infeccin estreptoccica

Para cada paciente con fiebre, seleccione el diagnstico ms probable:

Sangramiento Gastrointestinal A. B. C. D. E. F. G. H. I. Amebiasis Angiodisplasia del colon Colitis por Clostridium difficile Carcinoma del colon Carcinoma del esfago Carcinoma del estmago Coagulopata Diverticulitis Epistaxis J. K. L. M. N. O. P. Q. R. Vrices esofgicas Hemorroides Enfermedad inflamatoria intestinal Colitis isqumica Sndrome de Mallory-Weiss Ulcera pptica Esofagitis por reflujo Infeccin por Salmonella Infeccin por Shigella

Para cada paciente con sangramiento intestinal, seleccione el diagnstico ms probable:

174

Cefalea A. B. C. D. E. F. G. H. I. Meningitis asptica Meningitis bacteriana Absceso cerebral Metstasis cerebral Oclusin cerebrovascular Cefalea en racimos Cefalea inducida por drogas Disfuncin de la articulacin temporomandibular Crisis hipertensiva J. K. L. M. N. O. P. Q. Migraa Tumor cerebral primario Seudo tumor cerebral Endocarditis bacteriana subaguda Hemorragia subaracnoidea Arteritis temporal Cefalea tensional Neuralgia del trigmino

Para cada paciente con dolor de cabeza, seleccione el diagnstico ms probable:

Dolor de extremidades inferiores A. B. C. D. E. F. G. H. I. Insuficiencia arterial aguda Enfermedad de Buerger Causalgia Celulitis Insuficiencia arterial crnica Ectasia venosa crnica Dermatomiositis Neuropata diabtica Congelacin J. K. L. M. N. O. P. Q. R. Sndrome de Leriche Linfedema Obstruccin de la vena cava inferior Osteomielitis lcera por presin Enfermedad de Raynaud Tromboflebitis Venas varicosas Insuficiencia venosa

Para cada paciente con dolor de extremidades inferiores, seleccione el diagnstico ms probable:

Apndice B

175

Debilidad de Extremidades A. B. C. D. E. F. G. H. I. Plexopata braquial Subluxacin C1-2 Subluxacin C7-T1 Encefalopata Absceso epidural Fractura de la columna cervical Sndrome de Guillain-Barr Neuropata por plomo Migraa J. K. L. M. N. O. P. R. Esclerosis mltiple Miastenia gravis Osteoartritis Poliomielitis Estado postictal Isquemia de disco cervical fracturado Ataque cerebrovascular transitorio Q. Hematoma subdural Metstasis vertebral

Para cada paciente con debilidad de una extremidad, seleccione el diagnstico ms probable:

Nusea / Vmito A. B. C. D. E. F. G. H. I. Bulimia nerviosa Toxicidad por digoxina Obstruccin al vaciamiento gstrico Gastroenteritis Gastroparesia Hepatitis Enfermedad inflamatoria Intestinal Hemorragia intracraneana Laberintitis J. K. L. N. O. P. Q. R. Meningitis Migraa Infarto miocrdico M. Carcinoma pancretico Pancreatitis Embarazo Vlvulo sigmoideo Obstruccin del intestino delgado Sncope vasovagal

Para cada paciente con nauseas y vmitos, seleccione el diagnstico ms probable:

176

Problemas Respiratorios A. B. C. D. E. F. G. H. I. Reaccin alrgica Neumona por aspiracin Bronquiectasia Carcinoma pulmonar Enfermedad pulmonar obstructiva crnica Insuficiencia cardiaca congestiva Fibrosis qustica Efectos secundarios a drogas Asma inducida por ejercicio J. K. L. M. N. O. P. Q. R. Cuerpo extrao Reflujo gastroesofgico Estenosis mitral Asma ocupacional Pleuresa Embolismo pulmonar Virus sincicial respiratorio Sarcoidosis Tuberculosis

Para cada paciente con sntomas respiratorios, seleccione el diagnstico ms probable:

Shock A. B. C. D. E. F. G. H. I. Crisis addisoniana Anafilaxis Fibrilacin auricular Bloqueo aurculo-ventricular Hemorragia de lcera pptica Sndrome carcinoideo Taponamiento cardaco Cardiomiopata Deshidratacin J. K. L. M. N. O. P. Q. Fractura de la pelvis Infarto al miocardio Embolia pulmonar Ruptura de aneurisma artico abdominal Ruptura de absceso del apndice Ruptura de un embarazo ectpico Ruptura de quiste ovrico Obstruccin del intestino delgado

Para cada paciente en shock, seleccione el diagnstico ms probable:

Apndice B

177

Disnea A. B. C. D. E. F. G. H. I. Absceso pulmonar Sndrome de dificultad respiratoria del adulto Anemia Asma Enfermedad pulmonar obstructiva crnica Insuficiencia cardaca congestiva Ruptura diafragmtica Cuerpo extrao va area Hiperventilacin J. K. L. M. N. O. P. Q. R. Espasmo larngeo Mesotelioma Miastenia gravis Tumor de Pancoast Derrame pleural Neumotrax Hipertensin pulmonar primaria Embolismo pulmonar Fibrosis pulmonar

Para cada paciente con disnea, seleccione el diagnstico ms probable:

Problemas Urinarios A. B. C. D. E. F. G. H. Cistitis aguda Retencin urinaria aguda Carcinoma de vejiga Carcinoma de prstata Diabetes inspida Diabetes mellitus Hipercalcemia Cistitis intersticial I. J. K. L. M. N. O. Vejiga neurognica Polidipsia psicognica Hiperplasia prosttica Prostatitis Pielonefritis Carcinoma de clula renal Estenosis ureteral

Para cada paciente con problemas urinarios, seleccione el diagnstico ms probable:

178

Sntomas Urinarios A. B. C. D. E. F. G. H. I. Retencin urinaria aguda Fstula vesical Carcinoma de vejiga Cistitis Efecto de droga Endometriosis Gonorrea Nefrosis hemoglobinrica Menstruacin J. K. L. M. N. O. P. Q. Mioglobinuria Nefrolitiasis Neumaturia Prostatismo Carcinoma renal Incontinencia por estrs Sfilis Trauma

Para cada paciente con sntomas urinarios, seleccione el diagnstico ms probable:

Sangramiento Vaginal /Dolor A. B. C. D. E. F. G. H. I. Desprendimiento de placenta Vaginitis bacteriana Sangramiento penetrante 1 Vaginitis por candida Carcinoma cervical Condiloma acuminado Embarazo ectpico Cncer endometrial Cuerpo extrao vaginal J. K. L. M. N. O. P. Q. Cervicitis gonoccica Embarazo molar Menstruacin normal Placenta previa Amenaza de aborto Tricomoniasis Fstula vesicovaginal Carcinoma vulvar

Para cada paciente con sangramiento o dolor, seleccione el diagnstico ms probable:

[ENG: breakthrough bleeding]

Apndice B

179

Flujo vaginal / Prurito Vaginal A. B. C. D. E. F. G. H. Vaginitis alrgica Vaginitis atrfica Vaginitis bacteriana Sangramiento penetrante Vaginitis por Candida Carcinoma cervical Condiloma acuminado Cncer endometrial I. J. K. L. N. O. P. Cuerpo extrao vaginal Cervicitis gonoccica Menstruacin normal Placenta previa M. Amenaza de aborto Tricomoniasis Fstula vesicovaginal Carcinoma vulvar

Para cada paciente con flujo vaginal o prurito, seleccione el diagnstico ms probable:

Sibilancias A. B. C. D. E. F. G. H. I. Angioedema Asma Bronquiectasia Sndrome carcinoideo Enfermedad pulmonar obstructiva crnica Insuficiencia cardiaca congestiva Fibrosis qustica Plipos endobronquiales Epiglotitis J. K. L. M. N. O. P. Q. R. Reflujo gastroesofgico Cuerpo extrao vas areas Bocio Disquinesia larngea Neumona Edema pulmonar Embolismo pulmonar Infeccin del virus sincicial respiratorio Crup viral

Para cada paciente con sibilancias, seleccione el diagnstico ms probable:

180

Anda mungkin juga menyukai